s s c e monthly currentaffair(august) · ssce 8981426494/8296260082 downloaded from:- page 1 s s c...

165
SSCE 8981426494/8296260082 Downloaded from :- www.onlinessce.com Page 1 S S C E Monthly CurrentAffair(August) Contact : 08981426494/08296260082 Visit us: - www.onlinessce.com Daily Current Affairs 1,August,2018 1) Govt launched eAksharayan Software in how many languages? Ans:- 7 Explanation:- E-Aksharayan is free, desktop software for converting scanned printed Indian Language documents into a fully editable text format in Unicode encoding. The software supports editing in seven Indian languages - Hindi, Bangla, Malayalam, Gurmukhi, Tamil, Kannada & Assamese. 2) Which of the following country will build the intercontinental ballistic missiles? Ans:- North Korea Explanation:- Washington Post reported that North Korea seems to be building one or two new liquid-fueled intercontinental ballistic missiles at a factory at Sanumdong. It also said that the advancement of the missiles is unknown. North Korea has produced the country's first missiles which are capable of reaching the United States. This is the latest evidence that suggests the ongoing activity at North Korea's nuclear and missile facilities despite of the Singapore Summit in June between the US

Upload: others

Post on 22-Sep-2020

9 views

Category:

Documents


0 download

TRANSCRIPT

Page 1: S S C E Monthly CurrentAffair(August) · SSCE 8981426494/8296260082 Downloaded from:- Page 1 S S C E Monthly CurrentAffair(August) Contact : 08981426494/08296260082 Visit us: - www

SSCE 8981426494/8296260082 Downloaded from :- www.onlinessce.com Page 1

S S C E Monthly

CurrentAffair(August)

Contact : 08981426494/08296260082

Visit us: - www.onlinessce.com

Daily Current Affairs 1,August,2018

1) Govt launched eAksharayan Software in how many

languages?

Ans:- 7

Explanation:- E-Aksharayan is free, desktop software for

converting scanned printed Indian Language documents into a fully editable text format in Unicode encoding. The software

supports editing in seven Indian languages - Hindi, Bangla,

Malayalam, Gurmukhi, Tamil, Kannada & Assamese.

2) Which of the following country will build the

intercontinental ballistic missiles?

Ans:- North Korea

Explanation:- Washington Post reported that North Korea seems

to be building one or two new liquid-fueled intercontinental ballistic missiles at a factory at Sanumdong. It also said that the

advancement of the missiles is unknown. North Korea has produced the country's first missiles which are capable of

reaching the United States. This is the latest evidence that suggests the ongoing activity at North Korea's nuclear and missile

facilities despite of the Singapore Summit in June between the US

Page 2: S S C E Monthly CurrentAffair(August) · SSCE 8981426494/8296260082 Downloaded from:- Page 1 S S C E Monthly CurrentAffair(August) Contact : 08981426494/08296260082 Visit us: - www

SSCE 8981426494/8296260082 Downloaded from :- www.onlinessce.com Page 2

President Donald Trump and the North Korean leader Kim Jong-

un.

3) Which network company signed world's first big 5G deal

worth $3.5 billion?

Ans:- Nokia

Explanation:- Finland's Nokia will supply T-Mobile, the third biggest U.S. mobile carrier, with $3.5 billion in next-generation 5G

network gear, marking the world's biggest 5G deal so far. In February, T-Mobile was working both with Nokia and rival

network vendor Ericsson of Sweden to build out 5G networks in

30 U.S. cities during 2018.

4) This city hosted the 2018 Unified Commanders' Conference

(UCC).

Ans:- New Delhi

Explanation:- The Unified Commanders' Conference (UCC) for the year 2018 has started at New Delhi on July 30 to review external

security challenges as well as the country's preparedness. The 2-day annual conference will provide a platform for discussions at

the Apex Level on all 'Joint Issues' among the three Services & Ministry of Defence. The scope of this conference is to enable the

forum to facilitate joint stock taking of the previous year's performance of all Services & Departments of Ministry of Defence

and jointly chart the course for the next year. The conference will be attended by Defence Minister Nirmala Sitharaman, Minister of

State for Defence Subhash Ramrao Bhamre, National Security Advisor Ajit Doval, the Chiefs of Army, Navy and Air Force among

others.

5) Which country has been designated as a nodal centre for preparing flash-flood forecasts to Asian nations by the World

Meteorological Organization (WMO)?

Ans:- India

Page 3: S S C E Monthly CurrentAffair(August) · SSCE 8981426494/8296260082 Downloaded from:- Page 1 S S C E Monthly CurrentAffair(August) Contact : 08981426494/08296260082 Visit us: - www

SSCE 8981426494/8296260082 Downloaded from :- www.onlinessce.com Page 3

Explanation:- The World Meteorological Organization (WMO) has

designated India as a nodal centre for preparing flash-flood forecasts to Asian nations including Vietnam, Sri Lanka, Myanmar

and Thailand. That means the Indian Meteorological Department (IMD) under Ministry of Earth Sciences (MoES) will have to

develop a customized weather model that can issue advance warning of floods. Using a combination of satellite mapping and

ground-based observation, this system - called the Flash Flood Guidance System - aims to provide forecasts six hours in advance.

Like India, several south-east Asian countries depend on the

monsoon and are prone to its vagaries. The proposed model will provide forecasts by computing the likelihood of rainfall and the

soil moisture levels to warn of possible floods.

6) Name the Bengali author who has passed away, recently.

Ans:- Ramapada Chowdhury

Explanation:- Acclaimed Bengali writer Ramapada Chowdhury

(95), whose story 'Abhimanyu' was made into a Hindi movie 'Ek doctor ki maut', has passed away in Kolkata, West Bengal on July

29, 2018. 'Abhimanyu' was based on the life and work of Dr Subhas Mukhopadhyay who created India's first and the world's

second test-tube baby in 1978. Director Tapan Sinha turned the story into a award winning Hindi film 'Ek doctor ki maut' in 1990.

Mrinal Sen's Hindi film 'Ek din achanak' (1989) was also based on Chowdhury's story 'Beej'. Chowdhury had penned famous literary

works such as 'Pratham Prahar' (1954), 'Banpalashir padabali' (1960), 'Ekhoni' (1969), 'Kharij', 'Bari badle jay' (1988),

'Abhimanyu' (1982). His last book 'Har.

7) Which team won the WSF-World junior squash team

championship 2018?

Ans:- Egypt

Explanation:- The Egypt squash team has won the boy's title at the 6th WSF-World junior squash team championship 2018 by

defeating England by 2-0 in Chennai, Tamil Nadu on July 29. Egypt's Marwan Tarek, who lost the individual championship

Page 4: S S C E Monthly CurrentAffair(August) · SSCE 8981426494/8296260082 Downloaded from:- Page 1 S S C E Monthly CurrentAffair(August) Contact : 08981426494/08296260082 Visit us: - www

SSCE 8981426494/8296260082 Downloaded from :- www.onlinessce.com Page 4

final, defeated Nick Wall of England in straight games. Czech

Republic and USA finished third and secured the bronze medal.

Host India finished 11th overall.

8) Which rank did India obtain at UN E-Government Index

recently?

Ans:- 96

Explanation:- India jumped 22 places to 96th rank in the top 100

of the United Nations E-Government Development Index (EGDI) 2018. Denmark, with an index value of 0.9150, topped the 2018

E-Government Development Survey. The E-Government survey is

released by the United Nations in every two years. The 2018 edition was titled as 'Gearing E-Government to Support

Transformation towards sustainable and resilient societies'. India, which was ranked 118 in 2014, jumped 11 places to be ranked 96

in 2018.

9) Which of the following is the only state in India that has an

NRC (National Register of Citizens)?

Ans:- Assam

Explanation:- Assam is the only state in India that has an NRC. In a bid to identify the illegal Bangladeshi migrants, the

government released the final draft of National Register of Citizens

(NRC) list with names of 28,983,677 citizens in Assam.

10) Which player enters pre-quarterfinals of World Badminton

Championships at Nanjing in China?

Ans:- Kidambi Srikanth

Explanation:- Kidambi Srikanth enter pre-quarterfinals of World

Badminton Championships at Nanjing in China. Srikanth defeated Pablo Abian Vicen of Spain 21-15, 12-21, 21-14. In Women's

singles, Olympic silver medalist PV Sindhu will face Fitriani Fitriani of Indonesia in the second round of the championships.

Saina Nehwal already entered the pre-quarterfinals defeated

Page 5: S S C E Monthly CurrentAffair(August) · SSCE 8981426494/8296260082 Downloaded from:- Page 1 S S C E Monthly CurrentAffair(August) Contact : 08981426494/08296260082 Visit us: - www

SSCE 8981426494/8296260082 Downloaded from :- www.onlinessce.com Page 5

Turkey's Aliye Demir In Men's doubles, pair of Chirag Shetty and

Satwiksairaj Rankireddy and Sumeeth Reddy and Manu Attri will

also play their round-of-16 matches.

11) Who of the following conferred with the Rajiv Gandhi

Sadbhavana Award 2018?

Ans:- Shri Gopalkrishna Gandhi

Explanation:- The advisory committee of the Rajiv Gandhi

National Sadbhavana Award has decided to award the Former West Bengal Governor Gopalkrishna Gandhi, this year, with the

Rajiv Gandhi Sadbhavana Award 2018. He was chosen by the

committee for his outstanding contribution for promoting communal harmony, peace and goodwill. The award carries a

citation and a cash award of Rs.10 lakh. The award will be presented on 20th August 2018 on the birth anniversary of former

Prime Minister Rajiv Gandhi. Former recipients of the Rajiv Gandhi Sadbhavana Award: Mother Teresa, Ustad Bismillah

Khan, Mohammed Yunus, Lata Mangeshkar, Sunil Dutt, Dilip Kumar, Kapila Vatsyayan, Teesta Setalvad, Swami Agnivesh, K.R.

Narayanan, Ustad Amjad Ali Khan, Muzaffar Ali and Shubha

Mudgal.

12) RBI increased the policy repo rate by 25 basis points to

__________.

Ans:- 6.5%

Explanation:- The Reserve Bank of India (RBI) increased the repo

rate by 25 basis points to 6.5% and the reverse repo rate to 6.25%. This is the third bi-monthly policy review of the 2018-2019

financial year. The marginal standing facility rate and the bank rate have been adjusted to 6.75%. This decision was made by the

Monetary Policy Committee headed by RBI Governor Urjit Patel. Repo rate is the rate at which the RBI lends money to commercial

banks in the event of a shortfall of funds.

13) Which team entered quarter finals of 2018 Women's

Hockey World Cup?

Page 6: S S C E Monthly CurrentAffair(August) · SSCE 8981426494/8296260082 Downloaded from:- Page 1 S S C E Monthly CurrentAffair(August) Contact : 08981426494/08296260082 Visit us: - www

SSCE 8981426494/8296260082 Downloaded from :- www.onlinessce.com Page 6

Ans:- India

Explanation:- India stormed into the quarterfinals of the Women's Hockey World Cup beating Italy 3-0 in London. In their last

playoffs, Indian women produced a stellar performance to ease past Italy. Two penalty corner goals from Neha Goyal and

Vandana Katariya in 45th and 55th minute helped India register the emphatic win. India will meet in Ireland. Meanwhile, in the

second crossover match of the day, hosts England beat Korea 2-0 to set up a quarterfinal date with the Netherlands. The semi-finals

will be played on Saturday. The Final is slated for Sunday.

14) Which ministry signed MoU With Ministry Of Electronics

And Information Technology recently?

Ans:- Ministry of Home Affairs

Explanation:- Ministry of Health Affairs signed MoU With Ministry

Of Electronics And Information Technology recently.

15) Which bank awarded as the best performing bank in CLSS

schemes?

Ans:- HDFC Bank

Explanation:- HDFC has been awarded the best performing

primary lending institution in Credit Linked Subsidy Scheme (CLSS) for the EWS (Economically Weaker Section) and LIG (Low

Income Group) category. The award was given by Prime Minister Narendra Modi. CLSS was introduced in June 2015 under

Pradhan Mantri Awas Yojana ( PMAY) for home loans to customers

from the EWS as well as LIG.

Daily Current Affairs 2,August,2018

Page 7: S S C E Monthly CurrentAffair(August) · SSCE 8981426494/8296260082 Downloaded from:- Page 1 S S C E Monthly CurrentAffair(August) Contact : 08981426494/08296260082 Visit us: - www

SSCE 8981426494/8296260082 Downloaded from :- www.onlinessce.com Page 7

1) Who has become the first Asian to swim UK-France

channel?

Ans:- Prabhat Koli

Explanation:- Prabhat Koli (19) from Maharashtra, who crossed

the English Channel when he was 16, has become the first Asian to swim the choppy water from Jersey (an Island in the UK) to

France in cold weather. The long-distance sea swimmer covered 25km in 6 hours and 54 minutes. He started off from La Coupe

Point of Jersey, behind St Catherine's backwater and touched the

shore of St Germain Plage on the Normandy coast of France. As part of his seven seas challenge, Prabhat is now training to cross

the North Channel in Ireland in first week of August.

2) Who won the 2018 RBC Canadian Open, recently?

Ans:- Dustin Johnson

Explanation:- Dustin Johnson won the 2018 RBC Canadian Open at the Glen Abbey Golf Club in Oakville in Canada. Dustin

Johnson shot a 6-under par 66 and won his first RBC Canadian Open at 23-under par. He is currently in world number one

position. Now he has become the first player after Tiger Woods (2005-09) with at least three wins in three consecutive seasons.

Dustin Johnson was awarded $1,116,000 for the win. Byeong Hun

An and Whee Kim secured second place.

3) Which Union Ministry unveiled a blueprint of Deep Ocean

Mission (DOM) to explore deep ocean?

Ans:- Union Ministry of Earth Science

Explanation:- The Centre drew a five-year plan and allocated Rs.

8000 crore to explore the ocean and its resources. It would focus on deep sea mining, under water robotics, underwater vehicles

and ocean climate change. The goals would be achieved by : establishing offshore desalination plant for tidal energy,

developing a submersible vehicle that can travel to atleast 6,000 metres with three people on board. 10 percent of the recovery from

these reserves have been presumed to meet the energy requirement of India for the next 100 years with the presence of

Page 8: S S C E Monthly CurrentAffair(August) · SSCE 8981426494/8296260082 Downloaded from:- Page 1 S S C E Monthly CurrentAffair(August) Contact : 08981426494/08296260082 Visit us: - www

SSCE 8981426494/8296260082 Downloaded from :- www.onlinessce.com Page 8

380 million metric tonnes of polymetallic nodules in the Central

Indian Ocean Basin.

4) Which social network has announced that it is to introduce

a time management tool which will be like a daily reminder?

Ans:- Facebook

Explanation:- Facebook (and Instagram) has announced that it is to introduce a time management tool which will be like a daily

reminder. This tool will help people to manage their time by restricting them in spending time on the platforms. Using this tool

the users can set a time limit to be spent on the app. They will be alerted when they reach the time limit. It is also introducing an

option to deactivate notifications on smartphones.

5) China successfully launched __________ satellite, an optical remote sensing satellite, as part of its high-resolution Earth

observation project?

Ans:- Gaofen-11

Explanation:- The Gaofen-11 satellite was launched on a Long

March 4B rocket from the Taiyuan Satellite Launch Center in northern Shanxi Province, China. It was the 282nd flight mission

on a Long March carrier rocket. This satellite will be useful in land survey, urban planning, road network design, agriculture, and

disaster relief. It will also provide data for the Belt and Road

Initiative (BRI). China introduced the Gaofen project in 2010.

6) With which country India sign Pact on Financial And

Technical Cooperation?

Ans:- Germany

Explanation:- India and Germany have signed an agreement on financial and technical cooperation focusing on sustainable urban

development and renewable energy. The government-to-government Umbrella Agreement on Financial Cooperation and

Technical Co-operation 2017 is worth about 5,000 thousand crore rupees. The agreement under the Indo-German Bilateral

Development Cooperation was signed between Joint Secretary in

the Finance Ministry, Sameer Kumar Khare, and German

Ambassador to India, Martin Ney, in New Delhi.

Page 9: S S C E Monthly CurrentAffair(August) · SSCE 8981426494/8296260082 Downloaded from:- Page 1 S S C E Monthly CurrentAffair(August) Contact : 08981426494/08296260082 Visit us: - www

SSCE 8981426494/8296260082 Downloaded from :- www.onlinessce.com Page 9

7) India and USA Military Cooperation Meeting begins in

__________.

Ans:- New Delhi

Explanation:- India and USA Military Cooperation Meeting begins

in New Delhi. The meeting was co-chaired by Lt Gen Satish Dua, Chief of Integrated Defence Staff to the chairman, Chiefs of Staff

Committee (CISC) and Lt Gen Bryan Fenton, Deputy Commander, Indo-Pacific Command. External Minister Sushma Swaraj and

Defence Minister Nirmala Sitharaman would meet their US

counterparts in 2 plus 2 meeting in September in New Delhi. The main aim of the meeting is on strengthening defense and strategic

ties.

8) Former FTII director John Sankaramangalam passes away.

He belongs to the state of;

Ans:- Kerala

Explanation:- Filmmaker and former director of Film and

Television Institute of India (FTII), John Sankaramangalam, died due to age related ailments in Thiruvalla, Kerala. John

Sankaramangalam was 84 years old. He hailed from Pathanamthitta in Kerala. He had received two National and four

Kerala State Film Awards. He had directed several experimental movies and documentaries. He had secured a diploma in direction

and screenplay writing from FTII. Later, he became the director of

FTII.

9) Which become the first State in the country to implement

the national policy on biofuels?

Ans:- Rajasthan

Explanation:- Rajasthan has become the first State in the country

to implement the national policy on biofuels unveiled by the Centre this year. The desert State will lay emphasis on increasing

production of oilseeds and establish a Centre for Excellence in Udaipur to promote research in the fields of alternative fuels and

energy resources. The policy on biofuels seeks to help farmers dispose of their surplus stock in an economic manner and reduce

the country's oil import dependence.

Page 10: S S C E Monthly CurrentAffair(August) · SSCE 8981426494/8296260082 Downloaded from:- Page 1 S S C E Monthly CurrentAffair(August) Contact : 08981426494/08296260082 Visit us: - www

SSCE 8981426494/8296260082 Downloaded from :- www.onlinessce.com Page 10

10) The US elevated this country's status in the export control

regime and designated it as a Strategic Trade Authorization-1

(STA-1) country.

Ans:- India

Explanation:- The US has elevated India's status in the export control regime and designated it as a Strategic Trade

Authorization-1 (STA-1) country. This will allow India to buy more advanced and sensitive technologies from America. India is the

only South Asian country to be on the list. Currently there are 36

countries on STA-1 list. Other Asian countries designated as STA-

1 are Japan and South Korea.

11) BEML Ltd and this firm signed pact for making mining

equipment.

Ans:- HEC

Explantion:-BEML, HEC sign pact for making mining equipment. The MoU will result into additional Rs 150-200 crore revenue for

both PSUs. It will delivery time, reduce working capital and create capability for high capacity equipment. The mining products could

be co-branded and will come under 'Make in India' initiative. Coal

India is a major client for these mining equipment.

12) Which of the following company has been acquired by the

Elgi Equipments Limited, recently?

Ans:- Pulford Air and Gas

Explanation:- Elgi Equipments Limited, an Indian manufacturer

of air compressors and automotive equipment, has acquired Pulford Air and Gas, an Australian company, at a total cost of

Australian $11.04 million. Sydney based Pulford is one of the largest distributors of industrial air compressors in Australia. Elgi

has now acquired F.R. Pulford & Son Pty Limited and its wholly-owned subsidiary Advanced Air compressors Pty Ltd. Jairam

Varadaraj, MD, Elgi Equipments, said Elgi expects Rs. 60 crore

revenue in Australia this financial year because of the acquisition.

13) Which state government announched to up 'iHub' (the

intelligent hub) for research and scientific learning?

Ans:- Andhra Pradesh

Page 11: S S C E Monthly CurrentAffair(August) · SSCE 8981426494/8296260082 Downloaded from:- Page 1 S S C E Monthly CurrentAffair(August) Contact : 08981426494/08296260082 Visit us: - www

SSCE 8981426494/8296260082 Downloaded from :- www.onlinessce.com Page 11

Explanation:-The Andhra Pradesh government will set up 'iHub'

(the intelligent hub), a first-of-its kind global research centre for scientific learning with an initial investment of Rs 245 crore

spread over five years. For this purpose, the state government has already collaborated with UNESCO Mahatma Gandhi Institute of

Education for Peace and Sustainable Development (MGIEP). Some of the social and economic objectives of iHub will be to create

50,000 future-ready skilled manpower, reduced school dropouts and capture the EdTech market. The initial staff will be about 40

International researchers in disciplines such as neuro-science,

education, psychology, digital instructional design, educational technology, Artificial Intelligence, data sciences, and

gamingamong others. The centre will also issue "Global

Certification" for digital educational content.

14) World Ranger Day was celebrated all over the world on;

Ans:- July 31

Explanation:- World Ranger Day was celebrated all over the world

on July 31st. The World Ranger Day was initiated by International Ranger Federation (IRF). It is jointly promoted by The Thin Green

Foundation, the charity arm of IRF. Ranger Day was first celebrated in 2007 on the anniversary of IRF's foundation day. .

This day is celebrated to honour Rangers killed or injured during duty. It also honours the work of Rangers in safeguarding earth's

natural treasures and cultural heritage.

15) This firm has been awarded the 'Marketer of the Year' award by the International Advertising Association of India

(IAAI)

Ans:- Amul

Explanation:- Gujarat Co-operative Milk Marketing Federation

Ltd. (GCMMF), the marketers of the popular Amul brand of milk and dairy products, was awarded the 'Marketer of the Year' award

by the International Advertising Association of India (IAAI). The award was received by RS Sodhi, the Managing Director (MD) of

GCMMF, on behalf of the 36 lakh milk famers and producers of Gujarat. Amul is the brand worth Rs 41,000 crore and is the

biggest cooperative owned by 36 lakh farmers of Gujarat. Amul is

Page 12: S S C E Monthly CurrentAffair(August) · SSCE 8981426494/8296260082 Downloaded from:- Page 1 S S C E Monthly CurrentAffair(August) Contact : 08981426494/08296260082 Visit us: - www

SSCE 8981426494/8296260082 Downloaded from :- www.onlinessce.com Page 12

not only known for its cooperative structure and farmer's faith,

but is also known for its marketing and advertizing strategies.

Daily Current Affairs 3,August,2018

1) Which of the following test did the Navy recently perform

on the Shore Based Test Facility (SBTF) in Goa?

Ans:- LCA Naval Prototype 2

Explanation:- The indigenous Light Combat Aircraft (LCA), Naval

Prototype 2 (NP2) successfully landed on the Shore Based Test Facility (SBTF) in Goa. This trial is a milestone towards completion

of CCT trials of LCA Navy. The Arrestor Hook System (AHS) for

Ship Deck operations of LCA Naval version was designed and developed by Aircraft Research and Design Centre (ARDC) wing of

Hindustan Aeronautics Limited (HAL). The combined effort by HAL, Aeronautical Development Agency (ADA), Center for Military

Airworthiness & Certification (CEMILAC), Regional Director Aeronautical Quality Assurance (RDAQA) and Navy led to the

success. Navy will conduct a series of such trials and the Naval

LCA is expected to be on INS Vikramaditya by the end of 2018.

2) Which state government launched 'Mukhya Mantri-Yuva

Nestham' for the unemployed youth in the state?

Ans:- Andra Pradesh

Explanation:- The Andhra Pradesh government has launched 'Mukhya Mantri-Yuva Nestham', a scheme through which an

allowance of Rs 1000 per month will be provided to unemployed youth in the state. About 12 lakh youths in the age group of 22-35

years will get the benefit of the scheme. The scheme will be extended to all those eligible even if there are more than one

beneficiary in a family.

3) Indian-origin prison guard, who won MasterChef Australia

2018 is __________.

Ans:- Sashi Cheliah

Explanation:- The 39-year-old Sashi Cheliah from Adelaide earned 93 out of 100 points to beat fellow contestant Ben Borsht,

who scored 77 out of 100. He earned a prize money of $250,000 (almost Rs 1.7 crore). Cheliah has served in the Singapore police

Page 13: S S C E Monthly CurrentAffair(August) · SSCE 8981426494/8296260082 Downloaded from:- Page 1 S S C E Monthly CurrentAffair(August) Contact : 08981426494/08296260082 Visit us: - www

SSCE 8981426494/8296260082 Downloaded from :- www.onlinessce.com Page 13

force for over a decade and then relocated to Australia, where he

started working as a prison guard in a women's prison.

MasterChef Australia 2018 is the popular culinary reality show.

4) Who will organice the Investors Conference for the Holistic

Development of Islands?

Ans:- NITI Aayog

Explanation:- NITI Aayog is set to host an Investors' Conference

for the Holistic Development of Islands, at Pravasi Bharatiya Kendra in New Delhi. This Conference will be inaugurated by

Amitabh Kant, CEO, NITI Aayog. It is expected to gain investment for the sustainable development of eco-tourism projects in

Andaman & Nicobar and Lakshadweep islands. The Conference is organized by NITI Aayog along with the respective UT

Administration. NITI Aayog has been mandated to look over the Holistic Development of Islands program, along with the respective

UT administration and State Governments.

5) Who taken charge as a Director (Finance) of Hindustan

Aeronautics Ltd (HAL)?

Ans:- C B Ananthakrishnan

Explanation:- C B Ananthakrishnan took charge as Director (Finance) of Hindustan Aeronautics Ltd (HAL). He joined HAL as

Chief Manager (Finance) at Helicopter Division, Bengaluru, in 2004. He was also HAL nominee Director in the Board of three

joint ventures of HAL.

6) Recently, which department join hands with SBI for Rs 25k

cr loan?

Ans:- NHAI

Explanation:- NHAI will ink a pact with State Bank of India (SBI)

for a long-term unsecured loan to the tune of Rs 25,000 crore. This is SBI's highest long-term unsecured loan to any entity and

also the largest ever loan sanctioned to NHAI. NHAI is getting an unsecured loan from SBI for 10 years with 3 years of moratorium

on repayments.

7) \Who of the following wins 6th Hungarian Formula 1 Grand

Prix, recently?

Page 14: S S C E Monthly CurrentAffair(August) · SSCE 8981426494/8296260082 Downloaded from:- Page 1 S S C E Monthly CurrentAffair(August) Contact : 08981426494/08296260082 Visit us: - www

SSCE 8981426494/8296260082 Downloaded from :- www.onlinessce.com Page 14

Ans:- Lewis Hamilton

Explanation:- Lewis Hamilton of Mercedes won the Hungarian

Grand Prix for the 6th time, at the Hungaroring circuit, in Hungary. This is the 67th career victory for Lewis Hamilton.

Daniel Ricciardo was named Driver of the Day by Formula One for

finishing at fourth position after starting from the 12th position.

8) Which company signed MoU with Karur Vysya Bank to provide financial assistance to commodity traders and

farmers?

Ans:- StarAgri Finance

Explanation:- To provide financial assistance to commodity traders and farmers, StarAgri Finance signed MoU with private

lender Karur Vysya Bank. Micro-lending would be provided to traders and farmers in Tamil Nadu, Karnataka, Andhra Pradesh

and Maharashtra. The assistance would be provided against the

commodities stored in warehouses of StarAgri Finance.

9) Which firm makes history by becoming first US company to

reach $1 trillion market value?

Ans:- Apple

Explanation:-Apple Inc. became the first U.S.-based company with a market value of $1 trillion, four decades after it was co-

founded by Steve Jobs in a Silicon Valley garage and later revolutionized the worlds of computing, music and mobile

communications. The iMac desktop computer, the iPod, iPhone,

and iPad were smash hits. Apple CEO - Tim Cook.

10) Who laid the foundation stone of the 2nd Phase of

construction of NEIAH in Shillong?

Ans:- Shri Shripad Yesso Naik

Explanation:- Union Minister of State (Independent Charge) for

AYUSH Shri Shripad Yesso Naik laid the foundation stone of the 2nd Phase of Construction of North East Institute of Ayurveda &

Homeopathy (NEIAH) at NEIAH Campus in Mawdiangdiang, Shillong. The estimated cost of the construction is Rs 40.80 crore.

The estimated cost of the construction is Rs 40.80 crore. It will

Page 15: S S C E Monthly CurrentAffair(August) · SSCE 8981426494/8296260082 Downloaded from:- Page 1 S S C E Monthly CurrentAffair(August) Contact : 08981426494/08296260082 Visit us: - www

SSCE 8981426494/8296260082 Downloaded from :- www.onlinessce.com Page 15

have residential accommodation for students, doctors and staff

and guest house.

11) On which date, the Earth Overshoot Day was observed?

Ans:- August 1

Explanation:- Earth Overshoot Day 2018 was observed on Aug

1st. Earth Overshoot Day is an initiative of Global Footprint Network. Global Footprint Network is an international research

organization that is changing how the world measures and manages its natural resources. The date of Earth Overshoot Day is

calculated from data of Global Footprint Network. Earth Overshoot Day marks the date when we all have consumed more from nature

than our planet can renew in the entire year.

12) Reliance Jio partnered with which bank for digital

services?

Ans:- SBI Bank

Explanation:- RIL's telecom subsidiary Reliance Jio partnered with State Bank of India (SBI) to extend the next generation of

digital banking and payments services to customers. MyJio platform will enable SBI YONO's digital banking features and

solutions for a seamless, integrated and superior customer experience. It will also extend the financial services capabilities of

SBI and Jio Payments Bank.

13) The Metro Rail Corporation of this city have signed an

MoU with BSNL for extending benefits in customer services.

Ans:- Lucknow

Explanation:- Lucknow Metro Rail Corporation (LMRC) and BSNL

have signed an MoU for extending benefits in customer services. BSNL will use the smart card services of LMRC for collection of

post-paid bills. It will also involve sale of pre-paid products like recharge vouchers and electronic top-up at LMRC Ticket Office

Machine (TOM) station counters.

14) Who launched the Dedicated Portal FOCAL For Cargo

Owners And Shippers?

Ans:- IWAI

Page 16: S S C E Monthly CurrentAffair(August) · SSCE 8981426494/8296260082 Downloaded from:- Page 1 S S C E Monthly CurrentAffair(August) Contact : 08981426494/08296260082 Visit us: - www

SSCE 8981426494/8296260082 Downloaded from :- www.onlinessce.com Page 16

Explanation:- The Inland Waterways Authority of India (IWAI) has

launched a dedicated portal to connect cargo owners and shippers with real-time data on the availability of vessels. The portal has

been named as the Forum of Cargo Owners and Logistics Operators (FOCAL). This is designed by the IT Department and the

traffic wing of IWAI.

15) This state made Yoga mandatory for all Colleges and

Universities, recently.

Ans:- Karnataka

Explanation:- The Karnataka government has made Yoga mandatory for all colleges and universities in the State. It was

decided at a meeting of Vice-chancellors and other officials of state universities which was chaired by State Higher Education Minister

G T Devegowda. According to the officials, all government, aided and private degree colleges in the state must conduct yoga for 20-

minutes. It also said that a proposal to recruit 5,000 teaching staff has been approved by the Chief Minister and the Finance

Department of the State.

Daily Current Affairs 4,August,2018

1) Which state government approves over Rs 7,000 crore for

water projects?

Ans:- Maharashtra

Explanation:- Maharashtra state minister informed that

Maharashtra govt. has approved over Rs 7,000 crore to fulfill the drinking water projects in scarcity hit regions. The government

has approved Rs 7,952 crore to finish 6,624 water projects in 10,583 villages under the National Rural Drinking Water Scheme.

The government has aimed to provide water supply to all regions and villages that are perennially drought-hit and also to finish the

project in Konkan, Nashik, Aurangabad, Amravati, Nagpur and

Page 17: S S C E Monthly CurrentAffair(August) · SSCE 8981426494/8296260082 Downloaded from:- Page 1 S S C E Monthly CurrentAffair(August) Contact : 08981426494/08296260082 Visit us: - www

SSCE 8981426494/8296260082 Downloaded from :- www.onlinessce.com Page 17

Pune divisions. Around 6,500 projects have already been

completed in the last four years by spending Rs 5,500 crore.

2) Who has been appointed as Secretary in the Ministry of

External Affairs?

Ans:- Vijay Thakur Singh

Explanation:- Ms. Vijay Thakur Singh is currently India's envoy to Ireland and will take over as Secretary (EAST) from October 1

after superannuation of Preeti Saran on September 30. She is a

1985 batch officer of the Indian Foreign Service.

3) This ministry and Skill Development and Entrepreneurship

join hands to train one lakh RAC Service Technicians.

Ans:- Ministries of Environment, Forest and Climate Change

Explantion:- A Memorandum of Understanding was signed between the Ministries of Environment, Forest and Climate

Change and Skill Development and Entrepreneurship took place up skilling and certification of 100,000 RAC service technicians.

This is under under the Skill India Mission - Pradhan Mantri Kaushal Vikas Yojana (PMKVY). It was attended by representatives

of UN Environment, UNDP, Deutsche Gesellschaft fur Internationale Zusammenarbeit (GIZ),etc. Skill training will

happen under HCFC Phase-out Management Plan (HPMP) Stage-I being implemented by the Ozone Cell, MoEFCC and Electronic

Sector Skill Council of India (ESSCI).

4) Which company inks pact with Telangana govt?

Ans:- Tech Mahindra

Explanation:- The Telangana State Information Technology,

Electronics and Communication department (ITE&C) signed an MoU with Tech Mahindra to launch India's first Blockchain

district in Telengana at the International Blockchain Congress 2018. It will have process development for the emerging

technology with innovative infrastructure and facilities. This will

Page 18: S S C E Monthly CurrentAffair(August) · SSCE 8981426494/8296260082 Downloaded from:- Page 1 S S C E Monthly CurrentAffair(August) Contact : 08981426494/08296260082 Visit us: - www

SSCE 8981426494/8296260082 Downloaded from :- www.onlinessce.com Page 18

start the growth of Indian blockchain start-ups and companies.

The blockchain district will provide the platform and technology assistance to all the incubators in it. It will be a centre of

excellence and an incubator.

5) Which state government has launched a 360 degree nation-

wide publicity campaign publicity campaign for 2018 men's

hockey World Cup?

Ans:- Odisha

Explanation:- The Government of Odisha has launched a 360

degree nation-wide publicity campaign "Heartbeats for Hockey" to

attract fans for 2018 men's hockey World Cup. The movement attempts to get India behind the game by way of music,

entertainment, fashion, pure sport and panel discussions. Each time anyone holds the custom-made hockey stick, named 'Heart

Beat', their heart beat will be recorded as a pledge to support hockey. The 'Heart Beat' will take a round of the country through

'Hockey Adda', a curated on-ground discussion platform for hockey players and experts. The event will take place at the

Kalinga Stadium in Bhubaneswar from November 28 to December 16. The state is the title partner for the hockey event and Odisha

Tourism is the host partner. The maximum players in the national

team are also from Odisha.

6) India & __________ to cooperate in bamboo sector in Tripura.

Ans:- Japan

Explanation:- A team from the Japanese Embassy in New Delhi and stakeholders in the Bamboo handicraft sector of Tripura took

part in a workshop called 'Bridging Japan and India by Bamboo'. The Japanese team, led by Kenko Sone, Minister, Economic and

Development, Embassy of Japan, India.

7) Who of the following won the 2018 International Contributions Award given by the Association for Educational

Communications and Technology (AECT), USA?

Page 19: S S C E Monthly CurrentAffair(August) · SSCE 8981426494/8296260082 Downloaded from:- Page 1 S S C E Monthly CurrentAffair(August) Contact : 08981426494/08296260082 Visit us: - www

SSCE 8981426494/8296260082 Downloaded from :- www.onlinessce.com Page 19

Ans:- K Anvar Sadath

Explanation:- K. Anvar Sadath, the vice chairman and executive director of the Kerala Infrastructure and Technology for Education

(KITE), has won the 2018 International Contributions Award given by the Association for Educational Communications and

Technology (AECT), USA. He has been chosen for the award to recognize his outstanding contribution to the field of educational

communication and technology in an international setting. With this, he has become the first Indian to be honoured with the

International Contributions award. The award will be presented at 2018 AECT International Convention function to be held in

Kansas City, Missouri, on October 26, 2018. The AECT is a premier professional association for instructional design and

educational technology.

8) India successfully test-fires Advanced Air Defence

interceptor missile from __________.

Ans:- Abdul Kalam Island

Explanation:- India test-fired the endo-atmospheric Advanced Air Defence (AAD) interceptor missile from the Abdul Kalam Island off

the Odisha coast. It is a part of the country's ballistic missile defence (BMD) shield against a multiple target scenario. The

Defence Research and Development Organisation (DRDO) test-fired the AAD interceptor from a canister against multiple

simulated targets. The interceptor is a 7.5-meter long single stage

solid rocket propelled guided missile.

9) How many amendments of GST Laws were approved in a Cabinet meeting chaired by Prime Minister Narendra Modi on 1st

August 2018?

Ans:- 46

Explanation:- In a Cabinet meeting chaired by Prime Minister Narendra Modi amendments of GST Laws were approved. It has

46 amendments. It includes hiking threshold limit to Rs.1.5 crore, allowance of employers to claim input tax credit on food, transport

Page 20: S S C E Monthly CurrentAffair(August) · SSCE 8981426494/8296260082 Downloaded from:- Page 1 S S C E Monthly CurrentAffair(August) Contact : 08981426494/08296260082 Visit us: - www

SSCE 8981426494/8296260082 Downloaded from :- www.onlinessce.com Page 20

and insurance provided to employees under any law. It also has

modification of reverse charge mechanism, separate registration for companies having different business verticals, cancellation of

registration, new return filing norms and issuance of consolidated

debit/credit notes covering multiple invoices.

10) Who announced to set up the floating solar power plants?

Ans:- MRPL

Explanation:- Mangalore Refinery and Petrochemicals Ltd (MRPL) is setting up the floating solar power plants on the three water

storage reservoirs on its premises. Floating solar power plants

would not only generate around 6 MW of energy but also minimise water evaporation from the reservoirs. The 6 MW of solar power is

already being tapped from the rooftop panels. Also, MRPL would be distributing 31,867 LED bulbs to 8,000 families, angadwadis,

schools, community halls and shops in Bala, Kalavar, Permude, Kuthethoor, Jokatte, Thokur, Soorinje, Delanthabettu, Chelyar

and Madya villages, at a cost of around Rs.43 lakh. These bulbs

being would have a warranty period of two years.

11) Which Indian-Australian mathematician has bestowed

with the prestigious 2018 Fields medal?

Ans:- Akshay Venkatesh

Expklanation:- Akshay Venkatesh, a renowned Indian-Australian

mathematician, has won the 2018 Fields Medal for his profound contributions to an exceptionally broad range of subjects in

mathematics. Venkatesh is one of four winners of mathematics' top honour known as the Nobel prize for math. The Fields medals

are awarded every four years to the most promising mathematicians under the age of 40. Each winner receives a

15,000 Canadian-dollar cash prize. The other three winners are: Caucher Birkar, a Cambridge University professor of Iranian

Kurdish origin; Germany's Peter Scholze, who teaches at the University of Bonn and Alessio Figalli, an Italian mathematician at

ETH Zurich.

Page 21: S S C E Monthly CurrentAffair(August) · SSCE 8981426494/8296260082 Downloaded from:- Page 1 S S C E Monthly CurrentAffair(August) Contact : 08981426494/08296260082 Visit us: - www

SSCE 8981426494/8296260082 Downloaded from :- www.onlinessce.com Page 21

12) Gita Mittal is appointed as new Chief Justice of __________

high court.

Ans:- Jammu and Kashmir

Explanation:- Chief Justice Gita Mittal has been appointed as the

Chief Justice of Jammu and Kashmir High Court (HC). She is the acting Chief Justice of Delhi HC. She is the first woman Chief

Justice of J&K HC. Justice B.D. Ahmed, the State's last Chief Justice, was retired on March 15, 2018. Patna High Court Chief

Justice Rajendra Menon has been appointed as the Chief Justice

Delhi HC.

13) This country becomes World's 2nd biggest stock market.

Ans:- Japan

Explanation:- As a result of China's trade war with the US and its campaign to cut debt, the value of China's stock market fell to

$6.09 trillion, losing its position as the World's second biggest stock market to Japan ($6.17 trillion). The U.S. has the world's

largest stock market at just over $31 trillion. China's stock market overtook Japan's in 2014 and soared to an all-time high of over

$10 trillion in 2015.

14) What is the current repo rate, as per the 3rd bi-monthly

policy, released by RBI?

Ans:- 6.50

Explanation:- The Monetary Policy Committee (MPC) of the Reserve Bank of India (RBI) hiked policy repo rate by 25 basis

points to 6.50% in its third bi-monthly monetary policy review of

2018-19. Repo rate is the rate at which the RBI lends money to commercial banks in the event of any shortfall of funds. Reverse

repo rate, the rate at which the Reserve Bank borrows money from commercial banks within the country, was adjusted to 6.25%.

Accordingly, the Marginal Standing Facility (MSF) rate and the Bank Rate too stand adjusted at 6.75%. This time too, RBI has

maintained neutral stance with an objective of achieving the

Page 22: S S C E Monthly CurrentAffair(August) · SSCE 8981426494/8296260082 Downloaded from:- Page 1 S S C E Monthly CurrentAffair(August) Contact : 08981426494/08296260082 Visit us: - www

SSCE 8981426494/8296260082 Downloaded from :- www.onlinessce.com Page 22

medium-term target for headline inflation of 4 per cent while

supporting growth.

15) Name of the Indian player who becomes the second Indian

cricketer to smash a hundred in women's T20 cricket.

Ans:- Smriti Mandhana

Explanation:- Indian women's cricket team opener, 22-year-old, Smriti Mandhana scored 102 runs off 61 balls for Western Storm

against Lancashire Thunder, slamming 12 fours and four sixes. The first Indian who scored century in women's T20 cricket is

Mithali Raj (100* for Railways, 2017).

Daily Current Affairs 5,August,2018

1) According to a Morgan Stanley report, the GDP growth of

India is:

Ans:- 7.5%

Explanation:- India's economy is expected to clock GDP growth of

7.5% in this financial year according to a Morgan Stanley report. The growth recovery will remain robust, supported initially by

consumption and exports. In the January-March quarter, India's gross domestic product (GDP) grew at the fastest pace in seven

quarters at 7.7% on robust performance by manufacturing and service sectors as well as good farm output. India's economy is

expected to clock GDP growth of 7.5% in this financial year according to a Morgan Stanley report. The growth recovery will

remain robust, supported initially by consumption and exports. In the January-March quarter, India's gross domestic product (GDP)

grew at the fastest pace in seven quarters at 7.7% on robust performance by manufacturing and service sectors as well as good

farm output.

2) How many Indian companies are ranked in the Fortune's

World's Largest Companies?

Ans:- 7

Page 23: S S C E Monthly CurrentAffair(August) · SSCE 8981426494/8296260082 Downloaded from:- Page 1 S S C E Monthly CurrentAffair(August) Contact : 08981426494/08296260082 Visit us: - www

SSCE 8981426494/8296260082 Downloaded from :- www.onlinessce.com Page 23

Explanation:- Seven Indian companies have been listed in

Fortune's 500 list 2018 of world biggest corporations. Indian Oil Corporation (IOC) is the highest ranked company from India

followed by RIL which jumped 55 places. The US-based retail company, Walmart has topped the list. IOC ranked 137th in 2018

with a significant jump from 168th position from 2017. RIL is the top private sector company in India. It ranked 148th in 2018 from

203rd from 2017. ONGC ranked 197th in 2018 which not even featured in 2017. SBI got 216th rank 1 place higher than 217th in

2017.

3) The researchers from this country creates world's first

single-chromosome yeast.

Ans:- Chinese

Explanation:- Researchers from China have claimed to have

created the world's first single-chromosome yeast while not affecting the majority of its functions. The researchers involved in

the creation are molecular biologists from the Centre for

Excellence in Molecular Plant Sciences in China.

4) Commerce and Industry Minister Suresh Prabhu launched a

logo and tagline for Geographical Indications (GI) to increase awareness about intellectual property rights (IPRs) in the

country. The slogan for the GI tag is;

Ans:- Invaluable Treasures of Incredible India

Explanation:- Commerce and Industry Minister Suresh Prabhu

launched a logo and tagline for Geographical Indications (GI) to increase awareness about intellectual property rights (IPRs) in the

country. The slogan for the GI tag is- 'Invaluable Treasures of Incredible India'. A GI product is primarily an agricultural, natural

or a manufactured product (handicraft and industrial goods)

originating from a definite geographical territory

5) Who of the following beat a record of Olympian Michael

Phelps at the Far West International Swimming

Championship?

Page 24: S S C E Monthly CurrentAffair(August) · SSCE 8981426494/8296260082 Downloaded from:- Page 1 S S C E Monthly CurrentAffair(August) Contact : 08981426494/08296260082 Visit us: - www

SSCE 8981426494/8296260082 Downloaded from :- www.onlinessce.com Page 24

Ans:- Clark Kent Apuada

Explanation:- Ten-year-old swimmer Clark Kent Apuada of California placed first in all seven events at the Far West

International Swimming Championship and broke Olympian Michael Phelps record. In the 100 meter butterfly, he broke a

record set by Olympian Michael Phelps at the same meet back in

1995. He broke the records by 1.1 seconds.

6) Who won the gas distribution network auctions, recently?

Ans:- Gautam Adani's Group

Explanation:- Gautam Adani's group won the gas distribution network auctions. It gained rights to sell Compressed Natural Gas

(CNG) in 11 cities, including Allahabad. In the country's biggest City Gas Distribution (CGD) Adani won 6 cities on its own and

another 5 in joint ventures with state-owned Indian Oil Corp

(IOC).

7) Bharti Airtel plans to provide optical fibre network to which

state?

Ans:- Tamil Nadu

Explanation:- Bharti Airtel plans to provide optical fibre network

in Tamil Nadu. Being the telecommunications provider, it aims to improve network connectivity in the State. Currently, Tamil Nadu

has a fibre network for 14,000 km. With Bharti Airtel's plan for a rollout of 3,000 km of fresh optic fibre, it would touch 17,000 km

in 2021. The company has planned to invest Rs. 24,000 crore to Rs. 26,000 crore pan-India in mobile network sites, capacities,

fibre rollout and digitalisation among others.

8) This department joined hands with Invest India to offer its

three-month programme 'Integrate to Innovate' for startups in

the sector.

Ans:- DIPP

Page 25: S S C E Monthly CurrentAffair(August) · SSCE 8981426494/8296260082 Downloaded from:- Page 1 S S C E Monthly CurrentAffair(August) Contact : 08981426494/08296260082 Visit us: - www

SSCE 8981426494/8296260082 Downloaded from :- www.onlinessce.com Page 25

Explanation:- The commerce and industry ministry joined hands

with Invest India to offer its three-month programme 'Integrate to Innovate' for startups in the sector. This programme would be

carried out by the department of industrial policy and promotion (DIPP). The selected start-ups will receive a cash prize grant of up

to Rs 5 lakh along with an opportunity to pilot their product with corporates. The corporates would offer them access to technology,

technical and commercial mentorship and access to customers through networks. Innovators across various stages of energy life-

cycle : generation, transmission and distribution, storage and

consumption and in multiple sectors such as households, farm,

industry, infrastructure, building, utility and transport are invited.

9) Which country got warning from UNICEF about the

outbreak of cholera ?

Ans:- Yemen

Explanation:- World Health Organization (WHO) warned Yemen

may be on the verge of another cholera epidemic, which could be deadlier than previous ones because of widespread malnutrition in

the war-torn country. Since April 2017 to February 2018, more than 1,060,000 suspected cases of cholera and 2300 deaths have

been reported in Yemen.

10) Which payment bank has been bared by the RBI from

enroling new customers?

Ans:- Fino Payments Bank

Explanation:- Reserve Bank of India observed that a few Fino Payments Bank accounts had deposits in excess of the stipulated

amount of ?1 lakh. According to RBI's operating guidelines for payments banks, a customer can make deposits of up to ?1 lakh

per account in a year. Such banks cannot accept deposits beyond this limit. RBI has imposed similar restrictions on Paytm

Payments Bank (on June 20) and Airtel Payments Bank (in

January) which are also not able to on-bard new customers.

Page 26: S S C E Monthly CurrentAffair(August) · SSCE 8981426494/8296260082 Downloaded from:- Page 1 S S C E Monthly CurrentAffair(August) Contact : 08981426494/08296260082 Visit us: - www

SSCE 8981426494/8296260082 Downloaded from :- www.onlinessce.com Page 26

11) Which of the following is the first foreign lender to get

RBI approval for subsidiary?

Ans:- SBM Group

Explanation:- Mauritius-based SBM Group has received Reserve

Bank of India's approval to operate in the country through a wholly-owned subsidiary route. It is the first foreign lender to

receive such a licence after local incorporation was introduced in

2013.

12) Who was conferred with the Hindi Sahitya Academy's

Shalaka Samman for 2017-18?

Ans:- Javed Akhtar

Explanation:- Hindi Academy Delhi has given its highest Shalaka

Samman for 2017-18 to bollywood lyricist and poet Javed Akhtar. He is a father of Actor-filmmaker Farhan Akhtar. The scriptwriter,

lyricist and poet has five National Awards for Best Lyricist to his name for works such as Border, Refugee and Lagaan. The lyricist

has also been honoured with the Padma Shri in 1999 and the Padma Bhushan in 2007. The lyricist has also been honoured

with the Padma Shri in 1999 and the Padma Bhushan in 2007.

13) For the first time, Supreme court to get __________ serving

Women judges.

Ans:- 3

Explanation:- The Supreme Court will have three serving women judges for the first time, after the Centre cleared the elevation of

Madras High Court Chief Justice Indira Banerjee. The other two

judges are R Banumathi, appointed in August 2014, and Indu Malhotra, appointed in April 2018. Notably, SC will have a total of

22 male judges including two new male appointees.

14) The name of World's 1st Hindi-Speaking Robot is

__________.

Ans:- Rashmi

Page 27: S S C E Monthly CurrentAffair(August) · SSCE 8981426494/8296260082 Downloaded from:- Page 1 S S C E Monthly CurrentAffair(August) Contact : 08981426494/08296260082 Visit us: - www

SSCE 8981426494/8296260082 Downloaded from :- www.onlinessce.com Page 27

Explanation:- Ranchi-based software developer Ranjit Srivastava

is developing the world's first Hindi-speaking humanoid robot, claimed to cost just Rs50,000 so far. Named Rashmi, the AI-

powered robot can also speak Bhojpuri, Marathi, and English, and is equipped with facial expressions and recognition systems. He

was inspired to create Rashmi after seeing Hong Kong-made

humanoid 'Sophia' in 2016.

15) Who has become the fourth youngest debutant in men's

ODI cricket, recently?

Ans:- Rohit Kumar Paudel

Explanation:- Nepalese cricketer Rohit Kumar Paudel, aged 15 years and 335 days, has become the fourth youngest debutant in

men's ODI cricket. Rohit made his debut against the Netherlands

in the second ODI at the VRA Ground in Amstelveen.

Daily Current Affairs 6,August,2018

1) This ministry agreed to hand over land for works on ten

infrastructure development projects in Bengaluru.

Ans:- Ministry of Defence

Explanation:- The Ministry of Defence(MoD) Nirmala Sitharaman

agreed to hand over land for works on ten infrastructure development projects in Bengaluru. In exchange, the State

government will have to sanction land of equal value in other parts of the State. The State cabinet is yet to make its decision this

issue. Meantime, MoD has been instructed to begin the

construction works. The list of the projects is yet to be announced.

2) Which International organization will conduct an

agricultural research for rural development?

Ans:- ICRISAT

Page 28: S S C E Monthly CurrentAffair(August) · SSCE 8981426494/8296260082 Downloaded from:- Page 1 S S C E Monthly CurrentAffair(August) Contact : 08981426494/08296260082 Visit us: - www

SSCE 8981426494/8296260082 Downloaded from :- www.onlinessce.com Page 28

Explanation:- International Crops Research Institute for the

Semi-Arid Tropics (ICRISAT) inked a Memorandum of Understanding (MoU) with the Catchment Area Development

Authority of Telangana Irrigation Department at the State Secretariat. Upon this, Telangana government has asked the

ICRISAT to conduct a research on the impact of Mission Kakatiya programme. The research would be conducted for 2 two years

covering various aspects like the impact of de-siltation on water availability for irrigation, changes in cropping pattern and

recharge of groundwater. It would also incorporate economic

valuation in terms of nutrients applied.

3) The first Indian who have been selected in ICANN is;

Ans:- Ajay Data

Explanation:- Ajay Data has got a two-year term and will take up

the position during ICANN 63 Meet in Spain in October 2018. Ajay Data is the first Indian to have been selected as a member of

ccNSO and will be representing India at a global level. He will be the custodian of Asia, Australia and Pacific Islands as a member

of ccNSO. ccNSO of the ICANN is the policy-development body for

global issues.

4) Who won gold in Women's singles of BWF World

Championships 2018?

Ans:- Carolina Marin

Explanation:- Spanish badminton ace Carolina Marin won gold in

Badminton World Federation (BWF) World Championships 2018, in Nanjing. She created history by becoming the first woman to

win three badminton World titles with an emphatic victory. She beat P.V.Sindhu of India by 21-19, 21-10 in Nanjing. P.V. Sindhu

won Silver.

5) A scheme for the girl child 'Mukhyamantri Kanya Utthan

Yojana' was launched in __________.

Ans:- Bihar

Page 29: S S C E Monthly CurrentAffair(August) · SSCE 8981426494/8296260082 Downloaded from:- Page 1 S S C E Monthly CurrentAffair(August) Contact : 08981426494/08296260082 Visit us: - www

SSCE 8981426494/8296260082 Downloaded from :- www.onlinessce.com Page 29

Explanation:- Bihar Chief Minister Nitish Kumar launched

'Mukhyamantri Kanya Utthan Yojana', a scheme for the girl child. The scheme will provide Rs.54,100 for a girl child from her birth

until she graduates. This is to express the State government's resolve for the empowerment of women. The 'scheme will be run

by three departments namely education, health and social welfare. The Chief Minister also said that the amount under the 'Bicycle

Yojna' will be increased from Rs.2500 to Rs.3000.

6) This state government will setup a state of the art "B-Hub"

for biotech and biopharma sector?

Ans:- Telangana

Explanation:- The Telangana government will setup a state of the

art "B-Hub" for biotech and biopharma sector in Genome Valley, Hyderabad. It's a first-of-its-kind initiative in the country, aimed

at providing much-needed support to the biopharma research and development activities, featuring a scale-up manufacturing facility

along with a turnkey incubator, lab space module at GLP grade spreading over a built-up area of 40,000 sq.ft. This end-to-end

biopharma scale-up manufacturing facility will give domestic companies the required impetus here on par with Korea, China

and France who have taken lead in biopharma innovation.

7) Who becomes 6th Indian woman to enter top 200 Tennis

rankings in Singles?

Ans:- Karman Kaur Thandi

Explanation:- Karman Kaur Thandi, 20-year-old Delhi girl, jumped 32 places and now ranked 200th in the WTA (Women's

Tennis Association) list. In April, India's top singles player Ankita had broken into the top-200 and is now ranked 195 (-4). Earlier,

Sania Mirza, Nirupama Vaidyanathan, Shikha Uberoi and Sunitha Rao had made it to the top-200. In the men's singles, Yuki

Bhambri was unchanged at 86 while Ramkumar Ramanathan

climbed four places to a career-best 111.

Page 30: S S C E Monthly CurrentAffair(August) · SSCE 8981426494/8296260082 Downloaded from:- Page 1 S S C E Monthly CurrentAffair(August) Contact : 08981426494/08296260082 Visit us: - www

SSCE 8981426494/8296260082 Downloaded from :- www.onlinessce.com Page 30

8) Which state government constituted two committees to

deal with a possible influx of illegal immigrants?

Ans:- Manipur

Explanation:-The Manipur government has constituted two

committees - state and district level - to deal with a possible influx of illegal immigrants. The State Level Monitoring Committee will

monitor the entry or presence of illegal immigrants in the state and take steps for creating adequate public awareness to counter

the situation. It will be headed by Special Secretary (Home) while the Deputy Commissioners and Superintendents of Police as its

members. The district level committee will be headed by the concerned deputy commissioners while Superintendent of Police,

SDOs, SDPO, SDCs, OCs will be the members. The committee

with local bodies would likely to conduct headcounts if needed.

9) Who tops in the List of India's 28 Fugitive Economic

Offenders?

Ans:- Vijay Mallya

Explanation:- Vijay Mallya (promoter of defunct Kingfisher

Airlines) allegedly defrauded banks of about Rs 7,500 crore, according to the list the Finance Ministry submitted in the Lok

Sabha. Mehul Choksi, promoter of Gitanjali Jewels (?7,080 crore) and his nephew Nirav Modi (?6,498 crore) took the second and

third spots. The list also includes Sterling Biotech promoters Chetan Sandesara, Nitin Sandesara and Diptiben Sandesara, who

allegedly defrauded banks of ?5,383 crore.

10) With which country India collaborate to strengthen

sectors of agriculture, health & environment?

Ans:- Israel

Explanation:- Six Israeli startups working on healthcare,

agriculture and water management sectors have been selected under the "Israel-India Bridge to Innovation" program to interact

with various Indian businesses. The 6 startups that have been

Page 31: S S C E Monthly CurrentAffair(August) · SSCE 8981426494/8296260082 Downloaded from:- Page 1 S S C E Monthly CurrentAffair(August) Contact : 08981426494/08296260082 Visit us: - www

SSCE 8981426494/8296260082 Downloaded from :- www.onlinessce.com Page 31

selected include: Amaizz, Biofeed, Zebra Medical, MobileODT, etc.

The innovation week was started by Indian Prime Minister Narendra Modi and Israeli Prime Minister Benjamin Netanyahu

along with Israel Innovation Authority (IIA) and Invest India in 2017. India-Israel Innovation Bridge was developed to help Israeli

companies use their technologies, develop, adapt and

commercialize their innovation in India.

11) NASA selects Indian origin Sunita Williams for which

program to the International Space Station.

Ans:- Boeing

Explanation:- Indian-origin U.S. astronaut Sunita Williams is among the 9 astronauts selected by NASA (National Aeronautics

and Space Administration) to fly the first missions into space on commercial spacecraft, starting in 2019. These nine astronauts

will launch on the first crewed test flights and missions of new commercial spacecraft built and operated by the Boeing Company

and SpaceX. The astronauts will launch on Boeing CST-100 Starliner and SpaceX Dragon capsules to the International Space

Station starting in 2019. These missions will be the first crewed launches from United States soil after the end of the space shuttle

programme in 2011.

12) External Affairs Minister Sushma Swaraj and this country minister discussed the bilateral ties on health, tourism,

defence and security.

Ans;- Kyrgyzstan

Explanation:- External Affairs Minister Sushma Swaraj met

Kyrgyzstan Foreign Minister Erlan Abdyldaev discussed possible areas of collaboration and the areas discussed during the bilateral

meeting includes political and parliamentary exchanges, defense and security, science and technology, economic, health and

tourism.

13) Which of the following railway station is renamed as the

Deen Dayal Upadhyaya Junction recently?

Page 32: S S C E Monthly CurrentAffair(August) · SSCE 8981426494/8296260082 Downloaded from:- Page 1 S S C E Monthly CurrentAffair(August) Contact : 08981426494/08296260082 Visit us: - www

SSCE 8981426494/8296260082 Downloaded from :- www.onlinessce.com Page 32

Ans:- Mughalsarai Junction

Explanation:- The Mughalsarai station got a new name after Uttar Pradesh Governor Ram Naik. Bhartiya Janata Party (BJP) National

President Amit Shah inaugurated the renamed station. It is

located on the main Howrah-Delhi Grand Chord line.

14) Who topped ICC Rankings for Test Batsmen?

Ans:- Virat kohli

Explanation:- India captain Virat Kohli topped the ICC Player Rankings for Test batsmen. Kohli thus became the seventh India

batsman, and first since Sachin Tendulkar (in June 2011) to be ranked the number one Test batsman in the latest ICC rankings.

At 934 points,Kohli has become India's highest-ranked batsman,

14th overall, on the all-time tally of points.

Daily Current Affairs7,August,2018

1) Who resigned her position as CEO of PepsiCo?

Ans:- Indra Krishnamurthy Nooyi

Explanation:- Indra Krishnamurthy Nooyi, 62, resigned her position as CEO of PepsiCo. She has been leading the company

since 2006. She studied at the Madras Christian College. She went to the Indian Institute of Management, Kolkata, before

leaving for the U.S. in 1980 for a second master's degree at the Yale School of Management. Ramon Laguarta, 54, will succeed

Ms.Nooyi as CEO. Her office will resume from October 3. Ms.Nooyi will remain Chairman until early 2019 to ensure a smooth and

seamless transition.

2) Which Bike company signed an MoU with Telangana government and Adishwar Auto Ride International (AARI) for

setting a manufacturing facility?

Ans:- Benelli

Explanation:- Italian motorcycle-maker Benelli signed an MoU

with Telangana government and Adishwar Auto Ride International (AARI) for setting a manufacturing facility. The group company

Page 33: S S C E Monthly CurrentAffair(August) · SSCE 8981426494/8296260082 Downloaded from:- Page 1 S S C E Monthly CurrentAffair(August) Contact : 08981426494/08296260082 Visit us: - www

SSCE 8981426494/8296260082 Downloaded from :- www.onlinessce.com Page 33

would initially start assembling CKD (completely knocked down)

kits of the bike for Benelli to assemble, manufacture bikes and import exclusive range of the bikes from Italy and SouthEast

Asian regions.

3) Which state government has launched "Horn Not Ok"

awareness campaign?

Ans:- Himachal Pradesh

Explanation:- The Himachal Pradesh government has launched "Horn Not Ok" awareness campaign in Shimla and Mandi along

with a statewide mobile application 'shaur nahin' (no noise). The main aim of the campaign is to spread awareness of unnecessary

blowing horn as it creates unnecessary noise pollution. The other towns of the state will also be brought under the purview of no

honking zones after the successful execution of this campaign in the two tourist places for 2 months. Though, no-honking rule will

not be applicable on emergency vehicles, including ambulances and fire tenders. The "Shor Nahin" application has been developed

to report any noise pollution complaint to the authorities. It's an initiative of the Department of Environment, Science and

Technology in association with the NIC of Himachal Pradesh. It allows a user to lodge a complaint with the officials concerned,

including deputy commissioner, superintendent of police and sub divisional magistrate, who can upload the action taken report on

the app.

4) The 10th annual conference on oral implantology was held

in __________.

Ans:- New Delhi

Explanation:- The 10th International conference on oral implantology was held in New Delhi to enhance the awareness and

knowledge on the latest innovations and technologies in oral implantology world over. The 3-day conference hosted by Academy

of Oral Implantology (AOI) has representations by doctors from the UK, the US, Israel, Austria, Germany, Italy, South Korea and

Spain. Oral implantology is a highly dynamic specialty and has over the years, introduced innumerable modifications and

improvements for dental treatment of patients. The vision is to

Page 34: S S C E Monthly CurrentAffair(August) · SSCE 8981426494/8296260082 Downloaded from:- Page 1 S S C E Monthly CurrentAffair(August) Contact : 08981426494/08296260082 Visit us: - www

SSCE 8981426494/8296260082 Downloaded from :- www.onlinessce.com Page 34

promote latest technologies and encourage younger doctors, in the

field of innovation. Experienced professionals will share their work, expertise with the younger crop and this translates into

sharing of knowledge from different parts of the globe. Various companies from the medical field have also showcased their

advanced products and technologies. Teeth decay, loss of teeth due to old age, diseases or accidents, bad and crooked teeth, teeth

decay happen due to bad oral hygiene.

5) ISRO annouced to launch its heaviest satellite GSAT 11 on

which of the following month?

Ans:- November

Explanation:- Indian Space Research Organisation's (ISRO) heaviest satellite so far - the GSAT-11 (weighing over 5.7 tonnes

i.e., 5,700-kg)- which the space agency had cleared for launch in June, will take off a spaceport in French Guiana on 30 November.

GSAT-11 carries 40 transponders in the Ku-band and the Ka-band frequencies, and is capable of providing high bandwidth

connectivity with up to 14 GB per second data transfer speed.

6) Who won the 2018 Fiji International Golf title?

Ans:- Gaganjeet Bhullar

Explanation:- Indian golfer Gaganjeet Bhullar has won the 2018

Fiji International title. It is his first win on the European Tour, ninth Asian Tour and 10th international title overall. Thus, he

became the most successful Indian player on Asian Tour and the first Indian to win on Australasia Tour. Bhullar joined Arjun

Atwal, Jeev Milkha Singh, SSP Chawrasia and Anirban Lahiri as the only Indians to win on the European Tour as this was

sanctioned by Asian, Australasia and European Tours. The victory also granted him an exemption on all three Tours till end of 2019.

Other Indian in fray, Ajeetesh Sandhu scored 71 in the final round

to end his campaign at the tied 43rd spot.

7) The Election Commission of India (ECI) has conducted the first-ever Orientation Workshop on Election Management for

IAS Asstt Secretaries in which city?

Ans:- New Delhi

Page 35: S S C E Monthly CurrentAffair(August) · SSCE 8981426494/8296260082 Downloaded from:- Page 1 S S C E Monthly CurrentAffair(August) Contact : 08981426494/08296260082 Visit us: - www

SSCE 8981426494/8296260082 Downloaded from :- www.onlinessce.com Page 35

Explanation:- The Election Commission of India (ECI) has

conducted the first-ever Orientation Workshop on Election Management for IAS Asstt Secretaries in New Delhi on August 3,

2018. The aim of the workshop is to familiarize the Assistant Secretaries with all facets of Electoral Management before they are

required to handle crucial assignments in conducting Elections in their field postings soon. In it, Chief Election Commissioner Om

Prakash Rawat advised the officers to be prepared for facing the newer challenges like Data harvesting; targeted messaging &

profiling; money and media influencers.

8) This country became 3rd Asian nation to get STA 1 status

from US.

Ans:- India

Explanation:- India has become the third Asian nation after Japan and South Korea, 37th overall, and first south Asian nation

to get the Strategic Trade Authorization-1 (STA - 1) status from the US. This status paves way for high-technology product sales to

New Delhi, particularly in civil space and defence sectors. The US made an exception for India, which is yet to become a member of

Nuclear Suppliers Group (NSG). India so far was listed in STA-2 category along with Albania, Hong Kong, Israel, Malta, Singapore,

South Africa and Taiwan.

9) The world's first-ever thermal battery plant was unveiled in

__________.

Ans:- Andhra Pradesh

Explanation:- The world's first-ever thermal battery plant was unveiled in Andhra Pradesh by the state's Chief Minister N

Chandrababu Naidu. Manufactured by the Bharat Energy Storage Technology Private Limited (BEST), the first-of-its-kind batteries

will aim to boost renewable sources of energy production and reduce the dependence on non-renewable fossil fuel-based energy

generation. The technology, which will help in mitigating carbon emissions, is best suited for grid balancing and stabilisations. It

will also be used to store energy for telecommunication, commercial establishments, electric vehicles and highway

charging stations.

Page 36: S S C E Monthly CurrentAffair(August) · SSCE 8981426494/8296260082 Downloaded from:- Page 1 S S C E Monthly CurrentAffair(August) Contact : 08981426494/08296260082 Visit us: - www

SSCE 8981426494/8296260082 Downloaded from :- www.onlinessce.com Page 36

10) The International Conference on Sustainable Growth

through Material Recycling was inaugurated in which Indian

city?

Ans:- New Delhi

Explanation:- The conference was inaugurated by the Union Minister of Road Transport, Highways & Ganga Rejuvenation,

Nitin Gadkari at Pravasi Bharatiya Kendra in New Delhi. The day-long conference has 5 technical sessions on Circular Economy in

Material Recycling", "Infrastructural Issues in Recycling Industry",

"Standardization and certification in Recycling Industry", "Environmental Benefits of Material Recycling", "Financial and

Taxation issues in Recycling Industry".

11) Google launched the latest version of the Android

operating system, It is names as;

Ans:- Pie

Explanation:- Google announced 'Pie' as the name for the latest

version of the Android operating system, succeeding Android Oreo. The Android P or 'Pie' version has begun rolling out to Pixel

phones and will be available to other devices later this year.

12) Finance Minister Piyush Goyal announced a cash back of

__________ for digital transactions.

Ans:- 20%

Explanation:- Finance Minister Piyush Goyal announces 20% Cashback upto Rs 100 for the Digital Transactions done via BHIM,

Rupay or USSD mode. The main aim of the meeting on the concerns and suggestions of the Micro, Small and Medium

Enterprises. About GST Goods and Services Tax (GST) is an indirect tax (or consumption tax) levied in India on the supply of

goods and service. The GST was launched at midnight on 1 July 2017 by the President of India, Pranab Mukherjee, and the

Government of India.

13) With which country the Indian army begins the military

exercise "Maitree 2018" recently?

Ans:- Thailand

Page 37: S S C E Monthly CurrentAffair(August) · SSCE 8981426494/8296260082 Downloaded from:- Page 1 S S C E Monthly CurrentAffair(August) Contact : 08981426494/08296260082 Visit us: - www

SSCE 8981426494/8296260082 Downloaded from :- www.onlinessce.com Page 37

Explanation:- 'Maitree' is a joint military exercise between Indian

Army and Royal Thai Army which is running on in Thailand. It is a platoon level exercise which comprises of infantry component. The

exercise will emphasize to hone the tactical and technical skills in joint counter insurgency and counter-terrorist operations in the

rural and urban scenario under UN mandate.

14) Who launches Academia Alliance Programme

Ans:- Startup India

Explanation:- To fulfil the Government of India's mission to

promote the spirit of entrepreneurship in the country, Startup India launched the "Startup Academia Alliance programme". The

programme is a unique mentorship opportunity between academic scholars and startups working in similar domains. The

programme aims to reduce the gap between scientific research

and its industrial applications.

15) Name of the Indian personality who tops in the IQ world

list with the score of 148.

Ans:- Amit Sahai

Explanation:- Amit Sahai, a 43-year-old senior professional from

Kolkata has topped the prestigious Intelligence Quotient (IQ) World list along with two other Americans with a score of 148. He

has a background in international sales of software, IT, ITes services and high end audio and engineering products. He is also

a member of the International High IQ Society.

Daily Current Affairs8,August,2018

1) World's largest bird sculpture Jatayu sculpture will be

inaugurated in __________.

Ans:- Kerala

Explanation:- The Jatayu sculpture at the Jatayu Earth Center (JEC) at Chadayamangalam in Kollam district, Kerala, said to be

world's largest bird sculpture, will be inaugurated on August 17 by Chief Minister Pinarayi Vijayan. This Rs 100 crore-project is a

creation by film director and sculptor Rajiv Anchal.

Page 38: S S C E Monthly CurrentAffair(August) · SSCE 8981426494/8296260082 Downloaded from:- Page 1 S S C E Monthly CurrentAffair(August) Contact : 08981426494/08296260082 Visit us: - www

SSCE 8981426494/8296260082 Downloaded from :- www.onlinessce.com Page 38

2) Who maintains Top Spot in men's Association of Tennis

Professionals (ATP) world singles rankings?

Ans:- Rafael Nadal

Explanation:- Spain's Rafael Nadal maintained his lead in the

men's Association of Tennis Professionals (ATP) world singles rankings, with 9,310 average points, ahead of Switzerland's Roger

Federer.

3) GST Council meeting, a sub-committee was formed to address issues of Micro, Small and Medium Enterprises

(MSME) sector. Who is the head of the committee?

Ans:- Shiv Pratap Shukla

Explanation:- GST Council meeting, a sub-committee was formed

to address issues of Micro, Small and Medium Enterprises (MSME) sector. Shiv Pratap Shukla is the head of the commitee.

The committee will consist of Manish Sisodia, Sushil Modi, Himanta Biswa Sarma, Thomas Isaac and Manpreet Badal. The

issues related to law would be taken up by the law committee and issues related to rates would be taken up by the fitment

committee. The reports would be presented before the sub-

committee.

4) Which state hosted the 5th Act East Business Show -2018

to strengthen business ties between ASEAN countries?

Ans:- Meghalaya

Explanation:- 5th Act East Business Show -2018 was held in

Shillong, Meghalaya. It aims to strengthen business ties between

ASEAN countries and northeastern states. Also, to explore the potential of Meghalaya and its neighboring states. This is Indian

Chamber of Commerce along with Ministry of Development of North Eastern Region (MDoNER). Main focus was to boost the

bamboo sector of the region. North eastern states have great

potential for development of bamboo-based industries.

Page 39: S S C E Monthly CurrentAffair(August) · SSCE 8981426494/8296260082 Downloaded from:- Page 1 S S C E Monthly CurrentAffair(August) Contact : 08981426494/08296260082 Visit us: - www

SSCE 8981426494/8296260082 Downloaded from :- www.onlinessce.com Page 39

5) How many districts are identified by the NITI Aayog as

Aspirational Districts for RUSA Scheme?

Ans:- 117

Explanation:- During the second phase of the Centrally

Sponsored Scheme of Rashtriya Uchchatar Shiksha Abhiyan (RUSA), central assistance is provided for opening of new Model

Degree Colleges (MDCs) in 'Aspirational Districts' identified by NITI Aayog and in unserved & underserved districts in North Eastern

and Himalayan States. These districts have been selected on the basis of the composite index which includes published data of

deprivation enumerated under Socio-Economic Caste Census,

Health & Nutrition, Education and Basic Infrastructure.

6) Name of the Indian tennis player who won a bronze medal in the 2018 Hang Seng Hong Kong Junior and Cadet Open, in

Hong Kong.

Ans:- Swastika Ghosh

Explanation:- India's Swastika Ghosh won a bronze medal in the junior girls' doubles along with Singapore's Jingyi Zhou in the

2018 Hang Seng Hong Kong Junior and Cadet Open, in Hong Kong. Swastika Ghosh and Jingyi Zhou pair was defeated by Li

Lin Jassy Tan of Singapore and Ruei Ling Wen of Taipei 2-3 in the semi finals. In the junior boys' doubles category, Indian pair of

Raegan Alberquerque and Payas Jain pair reached pre-quarters. But they were knocked out as they lost to Yanapong Panagitgun

and Supakron Pankhaoyoy of Thailand.

7) S Gopakumar appointed Director and General Manager of;

Ans:- UIIC

Explanation:- Public sector non-life insurance firm United India Insurance Company announced the appointment of S Gopakumar

as its Director and General Manager with immediate effect. Prior to taking up the new role, Gopakumar was serving GIC Housing

Finance Ltd as its Managing Director and Chief Executive Officer.

Page 40: S S C E Monthly CurrentAffair(August) · SSCE 8981426494/8296260082 Downloaded from:- Page 1 S S C E Monthly CurrentAffair(August) Contact : 08981426494/08296260082 Visit us: - www

SSCE 8981426494/8296260082 Downloaded from :- www.onlinessce.com Page 40

8) Who of the following entering into a strategic partnership

with Reliance Money?

Ans:- PayU

Explanation:- PayU India announced entering into a strategic

partnership with Reliance Money, leading NBFC brands in India, to offer instant app-based personal loans, via LazyPay. It is an

attempt to reach out to the underserved consumers in India. The loan amount will start from as low as Rs. 100 and will extend up

to Rs. 1,00,000 with a duration varying from 15 days to 24 months. The loan amount will start from as low as Rs. 100 and

will extend up to Rs. 1,00,000 with a duration varying from 15

days to 24 months.

9) Which country successfully tests first hypersonic aircraft:

Xingkong-2, recently?

Ans:- China

Explanation:- China successfully tested "Xingkong-2" or "Starry

Sky-2", its first waverider hypersonic aircraft that could carry nuclear warheads and penetrate any current generation anti-

missile defence systems. Xingkong-2 or Starry Sky-2 was launched in a target range in Northwest China. It was launched in

a rocket and was released in the air after 10 minutes. It flew independently and landed in the targeted area after making large-

angle turning maneuvers. It reached an altitude of30 km with a speed of Mach 5.5-6. It was designed by the China Academy of

Aerospace Aerodynamics (CAAA) along with the China Aerospace

Science and Technology Corporation.

10) What is the rank of Indian men's hockey team in the

latest FIH rankings?

Ans:- 5

Explanation:- Indian men's hockey team has improved one rank and has reached the 5th spot in the latest FIH rankings chart.

Germany, England, Spain, New Zealand and Ireland are the other

Page 41: S S C E Monthly CurrentAffair(August) · SSCE 8981426494/8296260082 Downloaded from:- Page 1 S S C E Monthly CurrentAffair(August) Contact : 08981426494/08296260082 Visit us: - www

SSCE 8981426494/8296260082 Downloaded from :- www.onlinessce.com Page 41

teams in the top 10. In the women's rankings, the Indian team

improved one spot to and reached 9th position.

11) Andhra govt signed an MoU with this company for setting

up electronics manufacturing plant.

Ans:- Holytech

Explanation:- Andhra Pradesh Government signed an MoU with electronics company Holytech. Under this MoU, Holytech will set

up its plant in 75 acres land in Tirupati and Chittoor districts. It will have cost of Rs. 1400 crores. This would be the first-of-its-

kind facility in India. The production of the facility is estimated to

begin in the first quarter of 2019.

12) The Asin Nations Cup Chess Tournament 2018 was held in

__________.

Ans:- Iran

Explanation:- The biennial Asian Nations Cup Chess Tournament was held in Hamadan, Iran. It was organized by Iran Chess

Federation on behalf of Asian Chess Federation. India won 5 medals: 1 gold, 2 silver, 2 bronze. India ranked 2nd in the overall

ranking. he winning team of Gold consisted of : Grand Master Harika Dronavalli , International Master Eesha Karavade,

International Master Padmini Rout and Women's International

Masters (WIMs) Vaishali R, Aakanksha Hagawane.

13) Reserve Bank of India (RBI) has allowed non-fund-based

__________ at State levels to revive India's co-operative

banking sector?

Ans:- Umbrella organisations (UO)

Explanation:- The Reserve Bank of India (RBI) has allowed non-fund-based Umbrella organisations(UO) at State levels to revive

India's co-operative banking sector. It would conduct non-fund activities in the areas of training, capacity building and IT

infrastructure and research. Gujarat is the first State in the

Page 42: S S C E Monthly CurrentAffair(August) · SSCE 8981426494/8296260082 Downloaded from:- Page 1 S S C E Monthly CurrentAffair(August) Contact : 08981426494/08296260082 Visit us: - www

SSCE 8981426494/8296260082 Downloaded from :- www.onlinessce.com Page 42

country to have an UO for the UCB sector. Along with this, it is

conducting a two-day conference and exhibition of co-operative banking sector under the name Sahakar Setu 2018, in

Ahmedabad, from August 4-5.

14) Which team won the gold medal in the Women's Hockey

World Cup 2018?

Ans:- Netherlands

Explanation:- Women's Hockey World Cup 2018 was held in Queen Elizabeth Olympic Park, London. Defending champions the

Netherlands defeated Ireland 6-0 in the finals and won the

Women's Hockey World Cup 2018 title. The Netherlands have emerged as the world champions in Women's hockey for a record

8th time. They have extended their unbeaten performance in all competitions to 32 matches. Kitty van Male, one among the 6

different goal scorers emerged as the top scorer of the tournament

with 8 goals.

15) Who won his first European Tour title at the Fiji

International, at Natadola Bay Championship Golf Course, in

Fiji?

Ans:- Gaganjeet Bhullar

Explanation:- Gaganjeet Bhullar won the title at the Fiji International with a six-under 66 for a four-day total of 14-under

274. Australia's Anthony Quayle finished in the second position. South Africa's Ernie Els finished at tied third with Australia's Ben

Campbell. Another Indian Ajeetesh Sandhu finished at tied 43rd position. This is Gaganjeet Bhullar's 10th overall title. Now he has

become the most successful Indian golf player on Asian Tour.

Daily Current Affairs9,August,2018

1) Which team has won the 2018 Cotif Cup football

tournament?

Ans:- India

Page 43: S S C E Monthly CurrentAffair(August) · SSCE 8981426494/8296260082 Downloaded from:- Page 1 S S C E Monthly CurrentAffair(August) Contact : 08981426494/08296260082 Visit us: - www

SSCE 8981426494/8296260082 Downloaded from :- www.onlinessce.com Page 43

Explanation:- The Indian Under-20 football team has created

history by winning the 2018 Cotif Cup football tournament by defeating the 6-time U20 World Cup champions Argentina by 2-1

in their final match in Spain on 6 August. Deepak Tangri scored for India in the 4th minute and Anwar Ali's 68th minute put India

2-0 up against the most successful team in U-20 World Cup history. Argentina pulled one back in the 72nd minute, and hit

the cross bar in added time, but failed to break the Indian resolve as India concluded the tournament with a memorable

performance.

2)Who launches reusable rocket to send a communication

satellite for Indonesia into the orbit?

Ans:- SpaceX

Explanation:- SpaceX relaunched its newest version of its Falcon 9 rocket booster to send a communication satellite for Indonesia

into the orbit. The so-called Block 5 booster, which SpaceX says will be able to fly as many as 10 times with routine maintenance.

The booster first flew on May 11 2018, when it delivered the first

Bangladeshi satellite into space.

3) Which Constitutional Amendment Bill has passed by the

Parliment to grant constitutional status to the National

Commission for Backward Classes (NCBC)?

Ans:- 123rd Constitutional Amendment Bill

Explanation:- The Parliament of India has passed the 123rd Constitutional Amendment Bill, which grants constitutional status

to the National Commission for Backward Classes (NCBC). The bill was passed after the House repealed the NCBC Act, 1993. It will

contribute to the empowerment of the OBC communities across India. The bill states that the President may specify the socially

and educationally backward classes in the various states and union territories. He may do this in consultation with the

Governor of the concerned state. It will give the institution all powers to safeguard the rights of the socially and educationally

backward classes and fulfils a long pending commitment of the central govt to the OBCs. The Clause-3 (5) of the Bill gives the

proposed Commission the power to inquire into complaints of

Page 44: S S C E Monthly CurrentAffair(August) · SSCE 8981426494/8296260082 Downloaded from:- Page 1 S S C E Monthly CurrentAffair(August) Contact : 08981426494/08296260082 Visit us: - www

SSCE 8981426494/8296260082 Downloaded from :- www.onlinessce.com Page 44

deprivation of rights and safeguards. Clause 3 (8) gives it the

powers of a civil court trying a suit and allows it to summon anyone, require documents to be produced and receive evidence

on affidavit.

4) Which country's women team has clinched gold in the Blitz

event at the Asian Nations Chess Cup 2018?

Ans:- India

Explanation:- The Indian women chess team has clinched the gold in the Blitz event at the Asian Nations Chess Cup 2018 at

Hamadan, Iran on August 3. It's India's first gold-medal finish at the event since 2014. The Indian team comprising Harika

Dronavalli, Eesha Karavade, Padmini Rout, Vaishali R and Aakanksha Haga left the teams from Vietnam and China behind to

clinch gold. Apart from this, they also bagged a silver and a bronze in the Rapid and Classical categories, respectively. India (1st)

finished with 21.5 points, Vietnam (2nd) settled for silver with

18.5 and China (3rd) bagged bronze with 17.5.

5) Which firm unveiled a new logo symbolizing integrity,

excellence, trust and commitment?

Ans:- NSE

Explanation:- To mark 25 years of its existence, the National

Stock Exchange of India (NSE) unveiled a new logo symbolizing integrity, excellence, trust and commitment. The National Stock

Exchange of India Limited is the leading stock exchange of India, located in Mumbai. The NSE was established in 1992 as the first

demutualized electronic exchange in the country.

6) Rashida Tlaib set to become the 1st Muslim woman to be

elected to the parliament of;

Ans:- United States

Explanation:- Rashida Tlaib set to become 1st Muslim woman in

US Congress. She (42-year-old) is the daughter of Palestinian immigrants and won a Democratic primary election to represent

Michigan's 13th district, securing 33.6 percent of the vote. Tlaib is

set to replace John Conyers. She served in the Michigan state

Page 45: S S C E Monthly CurrentAffair(August) · SSCE 8981426494/8296260082 Downloaded from:- Page 1 S S C E Monthly CurrentAffair(August) Contact : 08981426494/08296260082 Visit us: - www

SSCE 8981426494/8296260082 Downloaded from :- www.onlinessce.com Page 45

house from 2009 to 2014, and was also the first Muslim woman

ever elected to the state's legislature.

7) IKEA, Swedish home furnishings opened its first store in

which Indian city?

Ans:- Hyderabad

Explanation:- IKEA, Swedish home furnishings major, opened its first store in Hyderabad, India. IKEA has already been sourcing for

the past three decades in India. It will be opening its other branches in Mumbai, Bengaluru and National Capital Region. The

store is of 400,000 sq. ft located at the heart of city's IT hub. IKEA in Hyderabad employs 950 people directly, half of them women,

and 1,500 indirectly in services and expects to host close to 7

million visitors each year.

8) The 22nd edition of 'Parampara series - National Festival of

Music and Dance' festival 2018 was held in __________.

Ans:- New Delhi

Explanation:- The 22nd edition of 'Parampara series - National

Festival of Music and Dance' festival will be held at Kamani Auditorim in New Delhi from August 10, 2018. The purpose of the

festival is to offer a platform to young talents trying to make their foothold in the world of art and culture. It is organised by Natya

Tarangini led by Padma Bhushan awardee Kuchipudi couple Raja and Radha Reddy. The 3-day event will also see performances by

veteran classical dance couple 'The Dhananjayans' (Shanta and Vannadil Pudiyaveettil Dhananjayan) and Kathak exponent Uma

Dogra. The festival has featured artistes like Grammy winning instrumentalist Pandit Vishwa Mohan Bhatt, Carnatic vocalist

Abhishek Raghuram, Khyal singers Pandit Rajan and Sajan

Mishra are among the many eminent artistes.

9) This country hosted the communication exercise "Pacific Endeavor-2018 (PE-18)" under the Multinational

Communications Interoperability Program (MCIP), recently.

Ans:- Nepal

Explanation:- The communication exercise "Pacific Endeavor-2018 (PE-18)" under the Multinational Communications

Page 46: S S C E Monthly CurrentAffair(August) · SSCE 8981426494/8296260082 Downloaded from:- Page 1 S S C E Monthly CurrentAffair(August) Contact : 08981426494/08296260082 Visit us: - www

SSCE 8981426494/8296260082 Downloaded from :- www.onlinessce.com Page 46

Interoperability Program (MCIP) has started in Kathmandu, Nepal

on August 6, 2018. The basic objective of 12 day exercise is to develop common communications operating procedures to enable

military forces in the Asia Pacific region to collectively work in the wake of disaster. Around 270 personnel from 20 Asia-Pacific

Countries are participating in the exercise, which is jointly organised by the Nepali Army and United States Pacific Command

(USPC). The Pacific Endeavor began in 2005 and it focuses on establishing rapid and effective inter-operable communication

systems to jointly act in disaster relief operations.

10) Which bank launched a payments machine called MOPAD & allows to facilitate transactions from cards to QR code

based payments?

Ans:- SBI

Explanation:- State Bank of India launched a payments machine

called MOPAD. The full form of MOPAD is Multi Option Payment Acceptance Device. It is a Point of Sale (PoS) terminal that will

allow to facilitate transactions from cards to QR code based payments. It would accept payments through UPI, Bharat QR, and

SBI Buddy wallet which till now required different tools to receive payments. Customers won't have to carry cards and can just scan

the QR code to make the payment.

11) S. Gurumurthy and Satish Kashinath Marathe as part time

non-official directors on the central board of

Ans:- RBI

Explanation:- The Centre appointed S. Gurumurthy and Satish Kashinath Marathe as part time non-official directors on the

central board of Reserve Bank of India for four years. Gurumurthy is a noted chartered accountant, economic and political columnist,

and editor of Tamil magazine Thuglak. The proposal for the appointment came from the Finance Ministry's Department of

Financial Services.

12) Which of the following sculpture will be returned to India

by the Metropolitan Museum of Art, the largest museum in

the US?

Page 47: S S C E Monthly CurrentAffair(August) · SSCE 8981426494/8296260082 Downloaded from:- Page 1 S S C E Monthly CurrentAffair(August) Contact : 08981426494/08296260082 Visit us: - www

SSCE 8981426494/8296260082 Downloaded from :- www.onlinessce.com Page 47

Ans:- Limestone sculpture of 'Head of a Male Deity'

Explanation:- An eighth-century stone sculpture of Goddess

Durga and a third century limestone sculpture 'Head of a Male Deity' will be returned to India by the Metropolitan Museum of Art,

the largest museum in the US. The stone sculpture of Durga Mahishasuramardini was donated to the museum in 2015. The

limestone sculpture, donated in 1986, was part of

Nagarjunakonda Site Museum's excavated inventory.

13) The former President of this country is named as new

chief of UN Human Rights Council.

Ans:- Chile

Explanation:- UN (United Nations) Secretary-General Antonio

Guterres has chosen former two time Chilean President Michelle Bachelet to be new UN high commissioner for human rights. She

would replace Jordan's Zeid Ra'ad al-Hussein. Bachelet who ranks among the world's most powerful women in politics, also served in

2010 as the first director of UN Women, the UN agency promoting

gender equality worldwide.

14) Which of the following category will be introduced this

coming year, at the 91st Oscars?

Ans:- Popular Film

Explanation:- The new Oscar will be awarded for "Outstanding Achievement in Popular Film" and will be introduced this coming

year, at the 91st Oscars. The Academy of Motion Picture Arts and Sciences, which awards the Oscars, has added a new category

after 17 years to recognise popular films. The last category to be

added was 'Best Animated Feature Film'.

15) A facial recognition system using artificial intelligence

will be used at which of the following event?

Ans:- 2020 Tokyo Olympic and Paralympic Games

Explanation:- A facial recognition system using artificial

intelligence will be used at the 2020 Tokyo Olympic and Paralympic Games. The Tokyo Olympics will be the first games to

have this kind of security system using facial recognition technology. It is aimed at strengthening security and preventing

Page 48: S S C E Monthly CurrentAffair(August) · SSCE 8981426494/8296260082 Downloaded from:- Page 1 S S C E Monthly CurrentAffair(August) Contact : 08981426494/08296260082 Visit us: - www

SSCE 8981426494/8296260082 Downloaded from :- www.onlinessce.com Page 48

terrorism, as well as shortening waiting times to lower athlete

stress.

Daily Current Affairs10,August,2018

1) Who signed an MoU with the Indian Institute of Technology

to boost research in traditional medicine and streamline

hospital care facilities in the country?

Ans:- AIIA

Explanation:- All India Institute of Ayurveda (AIIA) signed an MoU with the Indian Institute of Technology to boost research in

traditional medicine and streamline hospital care facilities in the country. The MoU is aimed at upgrading the national institutes of

traditional medicine at par with the IITs and IIMs. AIIA is a public

Ayurveda medicine & research institution located in New Delhi.

2) Google has launched a new video-based Q&A app which

allows people to answer questions about themselves, then share those answers directly on Google. The name of the app

is;

Ans:- Cameos

Explanation:- Google has launched a new video-based Q&A app

called 'Cameos' on the App Store, which allows people to answer questions about themselves, then share those answers directly on

Google. The app is aimed at celebrities and other public figures, who are often the subject of people's Google searches. With the

Cameos app, they can address fans' questions in their own voice,

instead of leaving the answers up to other websites.

3) Project SWAYAM that aims to provide one integrated platform and portal for online courses. Which ministry

launched the project?

Ans:- Ministry of Human Resource Development

Explanation:- HRD ministry launched a project named Study Webs of Active Learning for Young Aspiring Minds' (SWAYAM),

Page 49: S S C E Monthly CurrentAffair(August) · SSCE 8981426494/8296260082 Downloaded from:- Page 1 S S C E Monthly CurrentAffair(August) Contact : 08981426494/08296260082 Visit us: - www

SSCE 8981426494/8296260082 Downloaded from :- www.onlinessce.com Page 49

aims to provide one integrated platform and portal for online

courses. This course covers all higher education subjects and skill sector courses. The main objective is to ensure that every student

in the country has access to the best quality higher education at

the affordable cost.

4) Who will replace Indra Nooyi as Chief Executive Officer

(CEO) of PepsiCo Inc, on 3rd October 2018?

Ans:- Ramon Laguarta

Explanation:- PepsiCo Inc announced that Indra Nooyi would

step down as its long time chief executive officer. She will replaced

by Ramon Laguarta. He will take over on October 3 and will also join the board. Laguarta oversaw global operations, corporate

strategy, public policy and government affairs in his role as president. He has also served as CEO of the company's Europe

Sub-Saharan Africa division.

5) Who will addreses the 2018 World Biofuel Day?

Ans:- Shri Narendra Modi

Explanation:- World Biofuel Day is observed every year on August

10 to create awareness about the importance of non-fossil fuels as an alternative to conventional fossil fuels. Prime Minister

Narendra Modi will address a diverse gathering consisting of farmers, scientists, entrepreneurs, students, government officials

and legislators in New Delhi today to mark the World Biofuel Day

2018.

6) With which banke PayPal joint hands to offer safer, faster

and convenient payment experiences for the bank's

cardholders?

Ans:- HDFC Bank

Explanation:- PayPal, a digital payments platform has announced its strategic partnership with and HDFC Bank to offer safer, faster

and convenient payment experiences for the bank's cardholders.

Page 50: S S C E Monthly CurrentAffair(August) · SSCE 8981426494/8296260082 Downloaded from:- Page 1 S S C E Monthly CurrentAffair(August) Contact : 08981426494/08296260082 Visit us: - www

SSCE 8981426494/8296260082 Downloaded from :- www.onlinessce.com Page 50

To initiate incremental digital spend, the HDFC cards will be

presented as a payment option during enrollment and subsequent payments, with the ability for consumers to easily open PayPal

accounts and set it as their preferred payment method. The partnership will create a number of joint growth opportunities that

will advance PayPal and HDFC Bank's shared vision of offering

consumers greater choice and flexibility in managing their money.

7) Who of the following has been elected as the Deputy

Chairman of Rajya Sabha?

Ans:- Ram Shakal

Explanation:- The BJP-led NDA candidate Harivansh Narayan Singh was elected the Rajya Sabha's Deputy Chairman post,

which fell vacant after the retirement of Congress' PJ Kurien on July 1. Harivansh got 125 votes, while Congress-led UPA's

candidate BK Hariprasad got 105 votes. Harivansh Singh is a

member of the Janata Dal (United), the BJP's ally in Bihar.

8) Who inaugurated the 'One District One Product' Summit in

Lucknow, Uttar Pradesh?

Ans:- Shri Ram Nath Kovind

Explanation:- The President of India, Ram Nath Kovind,

inaugurated the 'One District One Product' Summit in Lucknow, Uttar Pradesh. The special emphasis was laid on micro, small and

medium enterprises (MSMEs). After the agricultural sector, most people find employment in this sector. Uttar Pradesh contributes

around 44% in the total handicraft exports of the country. Under the 'One District One Product' scheme, the state government has

set a goal of providing employment to 25 lakh people in five years

through the financial assistance of Rs. 25,000 crores.

9) Who launched a new digital platform 'MyDeal' to raise funds

via Capital Markets?

Ans:- HSBC

Page 51: S S C E Monthly CurrentAffair(August) · SSCE 8981426494/8296260082 Downloaded from:- Page 1 S S C E Monthly CurrentAffair(August) Contact : 08981426494/08296260082 Visit us: - www

SSCE 8981426494/8296260082 Downloaded from :- www.onlinessce.com Page 51

Explanation:- Hongkong and Shanghai Banking Corporation

(HSBC) has launched a new digital platform 'MyDeal' to simplify capital raising process through capital markets by providing real-

time access to information such as investors' feedback, profiles, client orders and deal pricing. The platform contains all the

information relating to a client's capital markets transaction and is updated on real-time basis. In its pilot phase, MyDeal has

raised over USD 25 billion in the first seven months through over

30 transactions globally.

10) Who of the following has been reappointed as the

chairman of TRAI?

Ans:- Ram Sewak Sharma

Explanation:- The government reappointed Ram Sewak Sharma as Chairman of Telecom Regulatory Authority of India (TRAI) till

September 2020 when he turns 65. He was named the TRAI chief in July 2015 for a three-year period. Sharma, who recently shared

his Aadhaar number on Twitter, also served as UIDAI's Director General between 2009-2013 and even wrote the first software for

UIDAI for enrolment.

11) Who launched the mobile application Niryat Mitra'?

Ans:- Shri Suresh Prabhu

Explanation:- Commerce and Industry Minister Suresh Prabhu

launched Mobile App Niryat Mitra. The app developed by the Federation of Indian Export Organisations (FIEO) is available both

on Android and on IOS platforms. The app provides a wide range of information required to undertake international trade right from

the policy provisions for export and import, applicable GST rate, available export incentives, tariff, preferential tariff, market access

requirements - SPS and TBT measures.

12) Who hosted the Investor's Conference at Pravasi Bharatiya

Kendra for the Holistic Development of Islands?

Ans:- NITI Aayog

Page 52: S S C E Monthly CurrentAffair(August) · SSCE 8981426494/8296260082 Downloaded from:- Page 1 S S C E Monthly CurrentAffair(August) Contact : 08981426494/08296260082 Visit us: - www

SSCE 8981426494/8296260082 Downloaded from :- www.onlinessce.com Page 52

Explanation:- NITI Aayog hosted an Investors' Conference at

Pravasi Bharatiya Kendra for the Holistic Development of Islands along with the respective UT Administration. The Conference was

inaugurated by Amitabh Kant, CEO, NITI Aayog. NITI Aayog hosted an Investors' Conference at Pravasi Bharatiya Kendra for

the Holistic Development of Islands along with the respective UT Administration. The Conference was inaugurated by Amitabh

Kant, CEO, NITI Aayog. It attracted investment for the sustainable development of eco-tourism projects in Andaman & Nicobar and

Lakshadweep islands. The 11 anchor tourism projects are

proposed to be implemented with private sector participation under the suitable risk-sharing model and through open-

competitive bidding.

13) Which state government launched Mukhyamantri Kanya

Utthan Yojana for empowerment of women?

Ans:- Bihar

Explanation:- Bihar Chief Minister Nitish Kumar launched Mukhyamantri Kanya Utthan Yojana for empowerment of women.

A sum of Rs 7221 crore will be spend in a year on the programme. It will be run by three departments - education, health and social

welfare. It will provide Rs 54,100 from the birth of a girl child till

she graduates.

14) Maharashtra Government has announced a special

assistance of __________ for OBC community to help them

develop employment opportunities for the youth?

Ans:- Rs 500 cr

Explanation:- Maharashtra Government announced a special assistance of Rs 500 crore for OBC community to help them

develop employment opportunities for the youth. This announcement was made by Maharashtra Chief Minister

Devendra Fadnavis at the 3rd National Convention of National OBC Association in Mumbai. Maharashtra government has also

recommended to the Centre to award Bharat Ratna posthumously to 19th-century social reformer Jyotiba Phule and his wife

Page 53: S S C E Monthly CurrentAffair(August) · SSCE 8981426494/8296260082 Downloaded from:- Page 1 S S C E Monthly CurrentAffair(August) Contact : 08981426494/08296260082 Visit us: - www

SSCE 8981426494/8296260082 Downloaded from :- www.onlinessce.com Page 53

Savitribai Phule, who is a pioneer of women's education. The

government will assess the extent of representation offered to OBCs in jobs and take measures to address the backlog, in a

timely manner. OBCs constitute 52 % of Maharashtra's

population. They have been given 17 % reservation.

15) Who will be the flag bearer for the Indian contingent at

the opening ceremony of the Asian Games 2018?

Ans:- Neeraj Chopra

Explanation:- Javelin thrower Neeraj Chopra will be the flag

bearer for the Indian contingent at the opening ceremony of the

Asian Games 2018. The 20-year-old has won a gold medal in World Junior Championships, Asian Championships, South Asian

Games and Commonwealth Games in his career. The Haryana athlete equalled the national record when he recorded a throw of

82.23m at the 2016 South Asian Games. Chopra set a new national record when he registered a throw of 87.43m at the

Diamond League in May 2018.

Daily Current Affairs11,August,2018

1) Which Union Minister launched Bid round -II under

Discovered Small Field Policy?

Ans:- Dharmendra Pradhan

Explanation:- Bid round -II under Discovered Small Field Policy was launched by Shri Dharmendra Pradhan, Minister of

Petroleum and Natural Gas & Skill Development and Entrepreneurship. The government aims to increase India's

growth to become the 4th largest economy and is bringing policy reforms to the energy sector. They are focused on: Energy Access,

Energy Efficiency, Energy Sustainability and Energy Security.

Discovered Small Fields (DSF) policy is a part of that reform.

2) Recently, Defence Minister Nirmala Sitharaman launched a

defence industrial corridor in which state?

Page 54: S S C E Monthly CurrentAffair(August) · SSCE 8981426494/8296260082 Downloaded from:- Page 1 S S C E Monthly CurrentAffair(August) Contact : 08981426494/08296260082 Visit us: - www

SSCE 8981426494/8296260082 Downloaded from :- www.onlinessce.com Page 54

Ans:- Uttar Pradesh

Explanation:- Defence Minister Nirmala Sitharaman and Uttar Pradesh Chief Minister Yogi Adityanath launched a defence

industrial corridor in Aligarh, Uttar Pradesh. The move is part of the government's efforts to develop specialised zones dedicated to

defence production in the state. Six nodal points, Agra, Aligarh, Lucknow, Kanpur, Chitrakoot and Jhansi have been identified in

state for the corridor. There will be an exhibition of products by the armed forces, defence public sector undertakings and DRDO

which are planned to be indigenised over the next five years.

Another defence corridor was also launched in Tamil Nadu.

3) Which become the first bank in the country to introduce

Iris Scan Authentication feature for Aadhaar -based

transactions through its micro ATM tablets?

Ans:- Axis Bank

Explanation:- Axis bank has become the first bank in the country to introduce Iris Scan Authentication feature for Aadhaar -based

transactions through its micro ATM tablets. This is being run as a pilot project in 8 branches of rural Punjab, Haryana, Gujarat and

Andhra Pradesh.

4) This company acquired a multi-million dollar order from AP

Cargo Logistic Network Corporation.

Ans:- Ramco Systems

Explanation:- Ramco Systems, a Software firm, acquired a multi-million dollar order from Manila-based air freight leader AP Cargo

Logistic Network Corporation with an aim to implement its logistics software. It is expected that AP Cargo would implement

the software to streamline its freight operations countrywide. AP Cargo partnered with Ramco after the partnership with the

Philippines as it was adopting IT solutions to ramp up its logistics capabilities and infrastructure. The freight and logistics market is

expected to reach $60.22 billion by 2023.

Page 55: S S C E Monthly CurrentAffair(August) · SSCE 8981426494/8296260082 Downloaded from:- Page 1 S S C E Monthly CurrentAffair(August) Contact : 08981426494/08296260082 Visit us: - www

SSCE 8981426494/8296260082 Downloaded from :- www.onlinessce.com Page 55

5) Who has been appointed as the chairperson of the National

Commission for Women, recently?

Ans:- Rekha Sharma

Explanation:- NCW member Rekha Sharma has been appointed

as the chairperson of the National Commission for Women. Rekha Sharma was earlier a member of the Commission and was holding

the additional charge of chairperson. The National Commission for Women is the apex national level organisation of India with the

mandate of protecting and promoting the interests of women.

6) Which renowned earth scientist has become the new vice

chancellor of the University of Kashmir?

Ans:- Prof. Talat Ahmad

Explanation:- Prof. Talat Ahmad, the renowned earth scientist, has become the new vice chancellor of the University of Kashmir.

He succeeded Khurshid Iqbal Andrabi. Prior to this post, Ahmad was the vice chancellor of Jamia Millia Islamia. He has been a

recipient of several prestigious awards and citations, including the National Mineral Award, 1994 from the Government of India. He

has also been a fellow of the Indian National Science Academy in New Delhi, Indian Academy of Sciences in Bangalore, fellow of the

National Academy of Sciences in Allahabad, J C Bose National

Fellowship, DST, New Delhi.

7) Which country has emerged as the top source of inward remittances according to the Reserve Bank of India's survey

for 2016-17?

Ans:- United Arab Emirates

Explanation:- According to the Reserve Bank of India's survey of

inward remittances for 2016-17, the United Arab Emirates (UAE) has emerged as the top source of inward remittances, while Kerala

has received the maximum funds sent from abroad. UAE's share in total remittances was 26.9%, followed by the United States

(22.9%), Saudi Arabia (11.6%), Qatar (6.5%) and Kuwait (5.5%).

Page 56: S S C E Monthly CurrentAffair(August) · SSCE 8981426494/8296260082 Downloaded from:- Page 1 S S C E Monthly CurrentAffair(August) Contact : 08981426494/08296260082 Visit us: - www

SSCE 8981426494/8296260082 Downloaded from :- www.onlinessce.com Page 56

According to the survey, 82% of the total remittances received by

India originated from eight countries - UAE, the U.S, Saudi

Arabia, Qatar, Kuwait, Oman, the United Kingdom and Malaysia.

8) The government approved a pact between the __________

and Chartered Professional Accountants, Canada.

Ans:- ICAI

Explanation:- The government approved a pact between the

Institute of Chartered Accountants of India (ICAI) and Chartered Professional Accountants, Canada. The MoU includes

collaboration on the definition, learning, and evaluation of the

professional qualities and skills and competencies of Chartered Accountants. It will provide an opportunity for the ICAI members

to expand their professional horizon.

9) How many branches of India Post Payments Bank (IPPB) will be opened by the Prime Minister Narendra Modi on the

21st August, 2018?

Ans:- 650

Explanation:- Prime Minister Narendra Modi will launch 650

branches of India Post Payments Bank (IPPB) on the 21st August, 2018. The IPPB has been setup under the Department of Post,

Ministry of Communication with 100% equity owned by the Indian government. The IPPB will leverage the vast network of the

Department of Posts with 1.55 lakh access points, three lakh

postmen and Grameen Dak Sewaks.

10) The theme for International Day of the World's Indigenous

Peoples 2018 is;

Ans:- Indigenous people's migration and movement.

Explanation:- International Day of the World's Indigenous Peoples

was observed all over the world on Aug 9th of every year. In 1994, the United Nations General Assembly declared August 9 as

International Day of the World's Indigenous Peoples. This day is

Page 57: S S C E Monthly CurrentAffair(August) · SSCE 8981426494/8296260082 Downloaded from:- Page 1 S S C E Monthly CurrentAffair(August) Contact : 08981426494/08296260082 Visit us: - www

SSCE 8981426494/8296260082 Downloaded from :- www.onlinessce.com Page 57

observed to promote and protect the rights of the world's

indigenous population. The theme for International Day of the World's Indigenous Peoples 2018 is: Indigenous peoples migration

and movement.

11) How many branches of India Post Payments Bank (IPPB)

will be opened by the Prime Minister Narendra Modi on the

21st August, 2018?

Ans:- 650

Explanation:- Prime Minister Narendra Modi will launch 650

branches of India Post Payments Bank (IPPB) on the 21st August,

2018. The IPPB has been setup under the Department of Post, Ministry of Communication with 100% equity owned by the Indian

government. The IPPB will leverage the vast network of the Department of Posts with 1.55 lakh access points, three lakh

postmen and Grameen Dak Sewaks.

12) Name of the Justice who taken charge as Chief Justice of

Delhi High Court, recently?

Ans:- Rajendra Menon

Explanation:- The Delhi high court with a sanctioned strength of 60 judges after the appointment of Justice Rajendra Menon. He

served as a judge in the Gwalior Bench of the Madhya Pradesh.

13) Which of the following is the 11th Biosphere Reserve from India to be included in the World Network of Biosphere

Reserves (WNBR) of UNESCO?

Ans:- Khangchendzonga Biosphere Reserve

Explanation:- UNESCO has included the Khangchendzonga

Biosphere Reserve (KBR) in the list of World Network of Biosphere Reserve (WNBR) at the 30th Session of the Man and Biosphere

International Coordinating Council (ICC) in Palembang, Indonesia. With this, KBR has now become the India's 11th internationally

designated WNBR and joined the list of other biospheres such as

Page 58: S S C E Monthly CurrentAffair(August) · SSCE 8981426494/8296260082 Downloaded from:- Page 1 S S C E Monthly CurrentAffair(August) Contact : 08981426494/08296260082 Visit us: - www

SSCE 8981426494/8296260082 Downloaded from :- www.onlinessce.com Page 58

Nandadevi, Simlipal, Sunderbans, Nokrek , Pachmarhi and Nilgiri.

The reserve, which is located in Sikkim, includes a range of ecoclines varying from subtropical to arctic, as well as vast natural

forests in different biomes that support an immensely rich diversity of forest types and habitats resulting in high species

diversity and endemism. The core zone of the biosphere alone has over 150 glaciers and 73 glacial lakes, the prominent one being

the famous 26-km-long Zemu glacier. It is home to a number of globally threatened fauna including musk deer, snow leopard, red

panda and Himalayan Tahr and many ethnic communities

including Lepcha, Nepalese, and Bhutia.

14) Who sworn as the 60th and youngest president of

Colombia?

Ans:- Ivan Duque

Explanation:- Colombia's President-elect 42-year-old Ivan Duque has been sworn in as the country's 60th president. He replaced

Juan Manuel Santos and became the youngest president in Colombia's recent history. He defeated left-leaning candidate,

Gustavo Petro, in a presidential runoff in June with 54 percent of

the vote.

15) Exercise SCO Peace Mission 2018 to be held in __________.

Ans:- Russia

Explanation:- As part of Shanghai Cooperation Organization (SCO) initiatives, SCO Peace Mission Exercise is conducted

biennially for SCO member states. The joint exercise for the year 2018 will be conducted by Central Military Commission of Russia

from 22 August to 29 August 2018 at Chebarkul, Chelyabinsk, Russia. This will be a historic occasion due to the maiden

participation of India post becoming a full member of the SCO in June 2017. This exercise marks a major milestone in the

multilateral relations of SCO member nations.

Page 59: S S C E Monthly CurrentAffair(August) · SSCE 8981426494/8296260082 Downloaded from:- Page 1 S S C E Monthly CurrentAffair(August) Contact : 08981426494/08296260082 Visit us: - www

SSCE 8981426494/8296260082 Downloaded from :- www.onlinessce.com Page 59

16) The head of the 4-member Group of Ministers (GoM) who

has been constituted to discuss concerns of trade pact with

RCEP countries is;

Ans:- Shri Suresh Prabhu

Explanation:- Four-member Group of Ministers (GoM) headed by Union Minister of Commerce and Industry Suresh Prabhu has

been constituted to to discuss concerns of trade pact with RCEP countries. This meeting will determine whether India is willing to

give 92% of its market access to these countries or not for free trade or not. The trade pact will affect the sectors of steel,

pharmaceuticals, agriculture, electronics, heavy industry, textiles and chemicals as there will be an influx of Chinese products

effecting the domestic markets. These talks are to be held before the upcoming trade negotiations in RCEP Summit with member

nations - including Japan, China, Australia, New Zealand and

South Korea in November 2018.

17) Which Indian-American personality has won the International Geography Bee World Championship 2018 in

junior varsity division?

Ans:- Avi Goel

Explanation:- Avi Goel (14), the Indian-American high school student, has won the International Geography Bee World

Championship 2018 in junior varsity division in Berlin, Germany. Avi is a tenth grader at Silver Creek High School in Evergreen, San

Jose, California. He won a gold medal in 7 out of the 10 medal

events, and a silver medal in two of the events, topping the overall medals table. He also bagged the top spot in all the three events

counting towards the world championship title: the International Geography Exam, the International Geography Showdown, and

the International Geography Bee. The competition took place from 11th to 18th of last month, bringing together over 100 of the

world's top geography students to vie for the world championship title in the subject. The competition was held by International

Academic Competitions, a worldwide organization that runs

Page 60: S S C E Monthly CurrentAffair(August) · SSCE 8981426494/8296260082 Downloaded from:- Page 1 S S C E Monthly CurrentAffair(August) Contact : 08981426494/08296260082 Visit us: - www

SSCE 8981426494/8296260082 Downloaded from :- www.onlinessce.com Page 60

academic competitions across the world, including the newly

formed International Geography Bee.

Daily Current Affairs12,August,2018

1) Who launched the World's first mission to touch the Sun

"Parker Solar Probe"?

Ans:- NASA

Explanation:- NASA's Parker Solar Probe, the world's first mission

to touch the Sun, was launched from Space Launch Complex 37 at Cape Canaveral Air Force Station, in United States. The Parker

Solar Probe was carried by a United Launch Alliance Delta IV Heavy rocket. The Parker Solar Probe has been sent on a 7 year

long mission to study the Sun's outer atmosphere and its effects on space weather. The Parker Solar Probe has been sent on a 7

year long mission to study the Sun's outer atmosphere and its

effects on space weather.

2) Indian Government extended e-visa facility for citizens of;

Ans:- 165 countries

Explanation:- In order to promote tourism, the Indian government has extended e-visa facility for citizens of 165 countries at 25

airports and five seaports. On March 2018, The Ministry of Tourism has launched the 'Incredible India 2.0' campaign, to

promote various destinations and tourism products of the country including spiritual, medical and wellness tourism in important

and potential source markets overseas.

3) Kidambi Srikanth ranked __________ in the 2018 BWF

ranking.

Ans:- 8th

Explanation:- India's Saina Nehwal has come out of the top 10 and Kidambi Srikanth has come down to 8th place in BWF

ranking, as per the Badminton World Federation (BWF) rankings released on 9th August 2018. Following poor performance at the

Page 61: S S C E Monthly CurrentAffair(August) · SSCE 8981426494/8296260082 Downloaded from:- Page 1 S S C E Monthly CurrentAffair(August) Contact : 08981426494/08296260082 Visit us: - www

SSCE 8981426494/8296260082 Downloaded from :- www.onlinessce.com Page 61

badminton World Championship, the ranks of Saina Nehwal and

Kidambi Srikanth have dropped. Saina Nehwal has been ranked world number 11 currently, in women's singles rankings. Kidambi

Srikanth has gone down two places and has reached 8th position in men's singles rankings. PV Sindhu remains at world number 3

in women's singles rankings.

4) Which digital payments major has acquired Balance

Technology, a Bengaluru-based savings management start-up?

Ans:- Paytm

Explanation:- The digital payments major Paytm acquired

Bengaluru-based savings management start-up, Balance Technology, for an undisclosed amount. The transaction is

expected to be about $2 million. This would enhance its user and merchant interfaces with customised and intuitive user

experiences. A six-member team of Balance Technology has joined

the product and design team.

5) Which team won the COTIF Tournament in Spain?

Ans:- Argentina

Explanation:- Argentina's U-20 football team won Torneo Internacional de Futbol Sub-20 de L'Alcudia (L'Alcudia

International Under-20 Football Tournament), also known as COTIF Tournament, held at Valencia, Spain by defeating Russian

team in the Extra Time by 2-1 goals. India was playing for the first time in the tournament. Floyd Pinto was the coach of the Under

20 team appointed by AIFF(All India Football Federation).

6) For, which state Southern Naval Command lunched an

operation "Madad" recently?

Ans:- Kerala

Explanation:- Operation "Madad" was launched by the Southern Naval Command (SNC) at Kochi, in Kerala, to assist the Kerala

state administration and to carry out disaster relief operations due

Page 62: S S C E Monthly CurrentAffair(August) · SSCE 8981426494/8296260082 Downloaded from:- Page 1 S S C E Monthly CurrentAffair(August) Contact : 08981426494/08296260082 Visit us: - www

SSCE 8981426494/8296260082 Downloaded from :- www.onlinessce.com Page 62

to the floods. Several parts of Kerala have been flooded due to

continuous rainfall and release of excess water from Idukki and other dams. Naval helicopters are used for ferrying divers, power

tools, axes and relief material to the flooded areas to speed up the

relief operations.

7) What will be the theme of Vibrant Gujarat Global Summit-

2019?

Ans:- Shaping of a New India

Explanation:- The theme of Vibrant Gujarat Global Summit-2019

will be 'Shaping of a New India'. It was decided at the first meeting

of the Advisory Committee of the event. The summit is proposed to

be held between January 18 and 20 next year.

8) Who of the following has been appointed as member of

National Commission for Protection of Child Rights?

Ans:- R. G Anand

Explanation:- Dr. R. G Anand has been appointed as member of

National Commission for Protection of Child Rights. Government has appointed Dr. R. G Anand as Member of National Commission

for Protection of Child Rights with effect from 13th July 2018 for a period of 3 years, or until he attains 60 years of age, or until

further orders, whichever is earlier

9) Which city was declared as India's most polluted city by Central Pollution Control Board (CPCB) in its Air quality index

(AQI), on 5th August 2018?

Ans:- Gurugram

Explanation:- The AQI of Gurugram has crossed 300 mark

marking it in the 'Very Poor' category of AQI. It scored 321 marks. 6 cities have been marked as 'Poor', 1 City (Gurugram) has been

marked as 'Very Poor', 2 cities has been marked as 'Good', 26

cities marked as 'Satisfactory' and 27 cities as 'Moderate'.

10) Who signed MOU with National Medicinal Plant Board?

Page 63: S S C E Monthly CurrentAffair(August) · SSCE 8981426494/8296260082 Downloaded from:- Page 1 S S C E Monthly CurrentAffair(August) Contact : 08981426494/08296260082 Visit us: - www

SSCE 8981426494/8296260082 Downloaded from :- www.onlinessce.com Page 63

Ans:- Tribal Cooperative Marketing Development Federation

of India Limited (TRIFED)

Explanation:- The main aim to promote Medicinal and Aromatic

Plants (MAPs) forest produced for livelihood development of tribal people by promoting primary level value addition to MAPs produce

from forest area at the grassroots level. Tribal Cooperative Marketing Development Federation of India Limited (TRIFED) was

established in August 1987 by the Ministry of Welfare,

Government of India.

11) The Theme of 18th International Youth Day is;

Ans:- Safe space for Youth

Explanation:- August 12 is annually celebrated as International Youth Day. It is an awareness day designated by the United

Nations to draw attention to cultural and legal issues surrounding youth. The Theme for IYD 2018 is: "Safe space for Youth". It was

designated by the United Nations in 1999 with the adoption of

Resolution 54/120.

12) The World Elephant Day is observed on which date?

Ans:- August 12

Explanation:- August 12 is annually celebrated as World Elephant Day. It is an international annual event dedicated to the

preservation and protection of the world's elephants. It is an international annual event dedicated to the preservation and

protection of the world's elephants. It was first conceived by Canadian filmmakers Patricia Sims and Michael Clark of

Canazwest Pictures, and Sivaporn Dardarananda, Secretary-General of the Elephant Reintroduction Foundation in Thailand. It

was started in August 12, 2012.

13) With which bank Bajaj Allianz General Insurance entered

into a bancassurance deal?

Ans:- Vijaya Bank

Page 64: S S C E Monthly CurrentAffair(August) · SSCE 8981426494/8296260082 Downloaded from:- Page 1 S S C E Monthly CurrentAffair(August) Contact : 08981426494/08296260082 Visit us: - www

SSCE 8981426494/8296260082 Downloaded from :- www.onlinessce.com Page 64

Explanation:- Bajaj Allianz General Insurance entered into a

bancassurance deal with Vijaya Bank. The General Insurance products would be available through the bank's network of 2,129

branches. Insurance products such as health, personal accident, home, motor and travel along with a commercial line of insurance

products will be offered. The partnership will strengthen the

presence of Bajaj Allianz General Insurance in southern India.

14) Global Innovation Index 2018 (GII- 2018) was launched in

__________.

Ans:- New Delhi

Explanation:- Global Innovation Index 2018 (GII- 2018) was launched in India in New Delhi. The event was organized by the

Confederation of Indian Industry (CII) along with the World Intellectual Property Organization (WIPO), in collaboration with

Department of Industrial Policy and Promotion (DIPP). CII is one of the founding partners of GII. Global Innovation Index 2018 is the

11th edition of GII. The theme for GII 2018 is: "Energizing the World with Innovation". The GII 2018 analyses the energy

innovation landscape of the next decade and identify possible new technological breakthroughs and innovative approaches and their

sources at the country, region, or firm level. India's rank on the Global Innovation Index (GII) is 57 in 2018. It has improved from

60 in 2017. India has been consistently improving in GII ranking

in the last 2 years.

15) Which country opens Visa Application Centre in Kolkata?

Ans:- Israel

Explanation:- Israel opened a visa application centre in the city of Kolkata for tourists from West Bengal and north eastern states.

Visa applications in the jurisdiction under the New Delhi centre would also be accepted here. The visa application categories

include : employment, business, tourism, meeting and conferences, students. Israel announced a reduction in visa fees to

INR 1,100 from its previous INR 1700 charge from Indians.

Page 65: S S C E Monthly CurrentAffair(August) · SSCE 8981426494/8296260082 Downloaded from:- Page 1 S S C E Monthly CurrentAffair(August) Contact : 08981426494/08296260082 Visit us: - www

SSCE 8981426494/8296260082 Downloaded from :- www.onlinessce.com Page 65

Daily Current Affairs13,August,2018

1) Which Union minister released 'Digital North East Vision

2022' in Guwahati, recently?

Ans:- Shri Ravi Shankar Prasad

Explanation:- Union Minister for Electronics & Information

Technology Ravi Shankar Prasad released 'Digital North East Vision 2022' in Guwahati. Digital technologies to transform the

lives of people of the northeast and enhance the ease of living. A cloud hub for North East will be created in Guwahati and capacity

building for 50,000 government staff will be taken up. The number of seats planned for BPOs in the Northeastern states will be

doubled to 10,000. The network of common service centres will be

expanded to cover all villages.

2) India's 1st Tribal Language to get own Wikipedia Edition is;

Ans:- Santhali

Explanation:- Santhali (Santali), a tribal language primarily spoken in India, Bangladesh and Nepal, received global

recognition when it got a Wikipedia edition in its own script. The language is been spoken by over 6.4 million people in the states of

Jharkhand, West Bengal, Odisha and Assam. The initiative to get a Santhali edition for Wikipedia, began six years ago, in 2012.

Santhali, written in the Ol Chiki script currently has content of

about 70,000 words.

3) This state government plans to launch Ayushman Bharat scheme to provide cashless healthcare facilities to

beneficiaries at empanelled hospitals.

Ans:- Haryana

Explanation:- Haryana Government to launch Ayushman Bharat

scheme on Aug 15. The main aim of the scheme to provide cashless healthcare facilities to beneficiaries at empanelled

hospitals.The scheme would be initially implemented on a pilot basis in one government hospital in each of the 22 districts, one

ESI hospital and one medical college.Under the scheme, families

identified by the Socio-Economic and Caste Census (SECC) would

be provided with coverage of Rs 5 lakh.

Page 66: S S C E Monthly CurrentAffair(August) · SSCE 8981426494/8296260082 Downloaded from:- Page 1 S S C E Monthly CurrentAffair(August) Contact : 08981426494/08296260082 Visit us: - www

SSCE 8981426494/8296260082 Downloaded from :- www.onlinessce.com Page 66

4) Which Indian authority has launched the RUCO initiative

recently?

Ans:- Food Safety and Standards Authority of India

Explanation:- The Food Safety and Standards Authority of India

(FSSAI) launched an initiative "RUCO (Repurpose Used Cooking Oil)" that will enable collection and conversion of used cooking oil

to biodiesel. Under this initiative, 64 companies at 101 locations have been identified to enable collection of used cooking oil.

According to FSSAI regulations, the maximum permissible limits

for Total Polar Compounds (TPC) have been set at 25%, beyond which the cooking oil is unsafe for consumption. Beside this,

FSSAI is also working in partnership with Biodiesel Association of India and the food industry to ensure effective compliance of used

cooking oil regulations.

5) Who has started a wellness clinic for its medical students

and doctors?

Ans:- AIIMS

Explanation:- All India Institute of Medical Sciences (AIIMS) has

started a wellness clinic for its medical students and doctors. Medical college in India experiences the highest level of stress

than students in any other country. The wellness clinic is being

run under the supervision of the AIIMS psychiatric department.

6) Who has topped the 'Women who Changed the World' list as

per the survey of BBC History Magazine?

Ans:- Marie Curie

Explanation:- Nobel Prize-winning scientist Marie Curie was

adjudged the most influential woman in history, through a poll conducted by BBC History Magazine. Marie Curie has topped the

'Women who Changed the World' list followed byRosa Parks (US civil rights activist), Emmeline Pankhurst (leader of the British

suffragette movement), Emmeline Pankhurst (leader of the British

suffragette movement).

7) India's first all-woman Special Weapons and Tactics (SWAT)

team for anti-terrorist operations has been inducted for

whom?

Page 67: S S C E Monthly CurrentAffair(August) · SSCE 8981426494/8296260082 Downloaded from:- Page 1 S S C E Monthly CurrentAffair(August) Contact : 08981426494/08296260082 Visit us: - www

SSCE 8981426494/8296260082 Downloaded from :- www.onlinessce.com Page 67

Ans:- Delhi Police

Explanation:- India's first all-woman Special Weapons and

Tactics (SWAT) team for anti-terrorist operations has been inducted in Delhi Police. The 36 elite women commandos, hailing

from north-eastern states, have undergone rigorous training of around 15 months from specialists all across India and abroad to

handle urban situations as well as jungle operations. Their expertise includes unarmed combat, ambush and counter-

ambush and urban operations like building interventions, vehicle intervention and VVIP security. They will be deployed at Red Fort

and India Gate during the 2018 Independence Day celebrations. The SWAT team is the brainchild of Amulya Patnaik, the Delhi

Police Commissioner.

8) Wipro has launched Santoor women's scholarship for higher education to girls from economically weaker families for

which three states?

Ans:-Karnataka, Andhra Pradesh and Telangana

Explanation:- The Wipro Consumer Care & Lighting will offer 3-

year Santoor women's scholarship for higher education to girls from economically weaker families in the three southern states of

Karnataka, Andhra Pradesh and Telangana. Approx 900 girls from the 3 states will be given Santoor scholarship, with Rs 24,000 for

each of them per annum to do under-graduation.. The students can apply for the scholarship on "www.santoorscholarships.com"

till September 15 or in state-run colleges across the three states. The scholarship is an attempt to empower women to change their

life and the world. Enrollment ratio of women pursuing higher education in the country is low and many give up higher study

due to lack of financial support.

9) ISRO will celebrate centenary celebration to commemorate

its first ISRO chief and renowned cosmic ray scientist. Who is

the renowned scientist?

Ans:- Vikram Sarabhai

Explanation:- The Indian Space Research Organisation will have

a year-long Vikram Sarabhai centenary celebration starting in August 2019 to honour visionary scientist and its legendary

Page 68: S S C E Monthly CurrentAffair(August) · SSCE 8981426494/8296260082 Downloaded from:- Page 1 S S C E Monthly CurrentAffair(August) Contact : 08981426494/08296260082 Visit us: - www

SSCE 8981426494/8296260082 Downloaded from :- www.onlinessce.com Page 68

founding father. Sarabhai, the architect of the Indian space

programme, the first ISRO chief and renowned cosmic ray

scientist, was born on August 12, 1919.

10) Which bank has signed a multilateral cooperation

agreement with member development banks of BRICS?

Ans:- Exim Bank

Explanation:- Exim Bank of India has signed a multilateral

cooperation agreement with member development banks of BRICS to undertake "collaborative research" in distributed

ledger/blockchain technology. The MoU signed by Exim Bank Managing Director David Rasquinha was an outcome of the

discussions the member development banks had during their annual meeting, with the objective of strengthening the

cooperation under the BRICS Interbank Cooperation Mechanism.

11) Who of the following was sworn in as the Chief Justice of

the Madras High Court?

Ans:- Vijaya Kamlesh Tahilramani

Explanation:- Justice Vijaya Kamlesh Tahilramani was sworn in as the Chief Justice of the Madras High Court. The swearing was

administered by Tamil Nadu Governor Banwarilal Purohit at a ceremony at Raj Bhavan, Chennai. Justice Tahilramani served in

the Bombay High Court. The appointment is after the elevation of Justice Indira Banerjee, as a Supreme Court judge.

Ms.Tahilramani, who was born in 1958, enrolled the Bar Council of Maharashtra and Goa in 1982 and was serving as a government

leader and has been practising in Bombay HC. In 2001, she was

elevated as a judge of the Bombay High Court.

12) Name the person who has been taken charge as the new

chairperson of Appellate Tribunal of Electricity.

Ans:- Manjula Chellur

Explanation:- Smt. Justice Manjula Chellur took Oath as Chairperson, Appellate Tribunal for Electricity, Ministry of Power.

Prior to this, Smt. Justice Manjula Chellur was Chief Justice of

Bombay High Court. She was the first woman Chief Justice of

Calcutta High Court.

Page 69: S S C E Monthly CurrentAffair(August) · SSCE 8981426494/8296260082 Downloaded from:- Page 1 S S C E Monthly CurrentAffair(August) Contact : 08981426494/08296260082 Visit us: - www

SSCE 8981426494/8296260082 Downloaded from :- www.onlinessce.com Page 69

13) Somnath Chatterjee who passed away recently was served

as a __________.

Ans:- Lok Sabha Speaker

Explanation:- Former Lok Sabha speaker Somnath Chatterjee

passed away. He was 89 years old. Chatterjee was the Speaker of the Lower House from 2004-09 when Manmohan Singh-led UPA-

1 (United Progressive Alliance) was in power.

14) Name of the Indian Author who won the 2018 Hindu

Playwright Award is;

Ans:- Annie Zaidi

Explanation:- Annie Zaidi, an Indian author, has won the 2018 Hindu Playwright Award 2018 for her play 'Untitled' at the 14th

edition of The Hindu Theatre Fest at the Taj Coromandel. The award was presented by Ratna Pathak Shah, actor and director.

The award, instituted in 2008, carries a prize of Rs 2 lakh for the best original, unpublished and unperformed play script in

English.

15) According to the International Monetary Fund (IMF), which of the following place would become World's richest

place by 2020?

Ans:- Macau

Explanation:- According to the International Monetary Fund

(IMF), Macau in China would overtake oil-rich Qatar with the highest per-capita gross domestic product of any country or

jurisdiction on earth, by 2020. Macau is the only place in China where casinos are legal and its gross domestic product has more

than tripled from about $34,500 per capita in 2001, the IMF data shows. Three European countries -- Luxembourg, Ireland, and

Norway -- made the top 10 places expected to be the world's

wealthiest by 2020, while the US came in at No. 12.

Daily Current Affairs14,August,2018

1) Balramji Das Tandon a Governor of this state passed away

at the age of 90.

Page 70: S S C E Monthly CurrentAffair(August) · SSCE 8981426494/8296260082 Downloaded from:- Page 1 S S C E Monthly CurrentAffair(August) Contact : 08981426494/08296260082 Visit us: - www

SSCE 8981426494/8296260082 Downloaded from :- www.onlinessce.com Page 70

Ans:- Chhattisgarh

Explanation:- Chhattisgarh Governor Balramji Das Tandon passed away at the age of 90. He was admitted to a hospital in

Raipur after he suffered cardiac arrest.

2) The lander of the Chandrayaan-2 to be named as;

Ans:- Vikram

Explanation:- The lander on the Chandrayaan-2 mission, which

is scheduled to be launched in January 2019 will be named 'Vikram' after Vikram Sarabhai, the father of Indian space

programme. The Indian Space Research Organisation (ISRO) has got clearance from the space commission. This mission, unlike

Chandrayaan-1 which only orbited the Moon, involves a Lander soft-landing on the lunar surface and unloading a Rover to study

and take measurements from the Moon, while the orbiter will go

around the Earth's satellite.

3) For which city, Justice Mukesh R Shah was sworn in as the

Chief Justice, recently?

Ans:- Patna

Explanation:- Justice Mukesh R Shah was sworn in as the Chief

Justice of Patna High Court. Justice Shah was a judge of the Gujarat High Court before his appointment as the Chief Justice of

Patna High Court. Bihar Governor Satya Pal Malik administered the oath of office to Justice Shah. He became a judge of the

Gujarat High Court on March 7, 2004 and was confirmed as

permanent judge on June 22, 2005.

4) Which city tops in the "Ease of Living Index 2018" survey?

Ans:- Pune

Explanation:- Of total 111 cities ranked under the 'Ease of Living Index 2018' which was released by Union Minister Hardeep Singh

Puri, Pune has been ranked first while Navi Mumbai has surfaced as the second preferred spot in terms of 'liveability'. Top 10 cities

Page 71: S S C E Monthly CurrentAffair(August) · SSCE 8981426494/8296260082 Downloaded from:- Page 1 S S C E Monthly CurrentAffair(August) Contact : 08981426494/08296260082 Visit us: - www

SSCE 8981426494/8296260082 Downloaded from :- www.onlinessce.com Page 71

in 'Ease of Living Index' 2018 list are: 1- Pune; 2- Navi Mumbai; 3-

Greater Mumbai; 4- Tirupati; 5- Chandigarh; 6- Thane; 7- Raipur;

8- Indore; 9- Vijaywada; 10- Bhopal.

5) Which of the following party formed the 15th National

Assembly in Pakistan?

Ans:- Pakistan Tehreek-e-Insaf

Explanation:- Pakistan's 329 newly-elected members, including

Imran Khan, of the National Assembly took the oath on 13th August. Imran Khan is all set to form the next government, only

the second democratic transition of power in the country. Speaker

Ayaz Sadiq administered the oath to the leaders in the 342-member house. Imran Khan and other prominent leaders

including Shahbaz Sharif, Bilawal Bhutto-Zardari and former President Asif Ali Zardari also took the oath. The maiden session

of the 15th National Assembly, the lower house of Parliament, happened after the swearing of the members. The PTI's final tally

reached 158 after it was allotted 28 out of 60 seats reserved for

women and 10 seats reserved for minorities.

6) Who of the following has been appointed as the UN High

Commissioner for Human Rights, reccently?

Ans:- Michelle Bachelet

Explanation:- Michelle Bachelet, the former two-time President of

Chile, has been officially appointed new UN High Commissioner for Human Rights for a period of 4 years. She will succeed

Jordanian diplomat Zeid Ra'ad al-Hussein in September 2018. The High Commissioner is the principle official who speaks out for

human rights across the whole UN system. It strengthens human rights mechanisms, enhance equality & fight discrimination in all

its forms. It also strengthens accountability & the rule of law and widens the democratic space & protecting the most vulnerable

from all forms of human rights abuse.

7) UK-based online market research and data analytics firm

YouGov reported this as India's most Patriotic Brand.

Page 72: S S C E Monthly CurrentAffair(August) · SSCE 8981426494/8296260082 Downloaded from:- Page 1 S S C E Monthly CurrentAffair(August) Contact : 08981426494/08296260082 Visit us: - www

SSCE 8981426494/8296260082 Downloaded from :- www.onlinessce.com Page 72

Ans:- SBI

Explanation:- Country's largest lender State Bank of India (SBI) is considered to be the most patriotic brand, according to a survey

conducted by UK-based online market research and data analytics firm YouGov, followed by Tata Motors, Patanjali, Reliance Jio and

BSNL. Around 16% respondents ranked SBI to be the most patriotic brand overall, followed by Tata Motors and Patanjali with

8% each, and Reliance Jio and BSNL with 6% each. The survey, which covered 152 brands across 11 categories, is based on data

collected online by YouGov Omnibus among 1,193 respondents in

the country using its panel of an online representation of India.

8) India's first genetic bank for wildlife conservation has been

inaugurated in __________.

Ans:- Hyderabad

Explanation:- In Hyderabad, India's first genetic bank for wildlife

conservation has been inaugurated by Union Minister for Science and Technology Dr. Harsh Vardhan at the Laboratory for

Conservation of Endangered Species (LaCONES) in CSIR-Centre for Cellular and Molecular Biology (CCMB) to protect India's

biodiversity and environment. It is dubbed as National Wildlife Genetic Resource Bank (NWGRB), which can store 17,000 vials

worth of samples. Currently genetic material, including DNA, cells and tissues besides semen and eggs of 23 wildlife species are

stored. This facility would also facilitate exchange of genetic material between Indian zoos for maintaining genetic diversity and

conservation management made accessible to scientists and wildlife managers for implementing conservation programmes. The

Genetic Resource Bank (GRB) is the systematic collection and preservation of tissues, sperm, eggs and embryos, genetic material

(DNA/RNA).

9) The first state to provide Right to Skill's Development to

the youth is;

Ans:- Chhattisgarh

Page 73: S S C E Monthly CurrentAffair(August) · SSCE 8981426494/8296260082 Downloaded from:- Page 1 S S C E Monthly CurrentAffair(August) Contact : 08981426494/08296260082 Visit us: - www

SSCE 8981426494/8296260082 Downloaded from :- www.onlinessce.com Page 73

Explanation:- Chhattisgarh Chief Minister Raman Singh

announced that Chhattisgarh is the first among the 28 other states in the country to provide Right to Skills' Development to the

youth. The Chief Minister along with Union Minister of State for Skill Development Anant Kumar Hegde flagged off fifteen Skill

Chariots to launch the government's project of 'Skills on Wheels' in the state. The chariots aimed at creating awareness regarding

the project will tour 13 districts in the first phase and the rest 14

in the second phase.

10) Who has become the youngest Indian golfer to claim Asian

Tour title at the 2018 Take Solutions Masters?

Ans:- Viraj Madappa

Explanation:- Viraj Madappa (20) from Kolkata has become the youngest Indian golfer to claim the Asian Tour title after lifting the

2018 TAKE Solutions Masters trophy in Bengaluru on August 12. Beside this, Madappa has also become the first rookie and

youngest player to win on Tour this season. He takes home a winner's prize purse of US$63,000 and earned a one-year winner

exemption on the region's premier Tour. The earlier record was held by Gaganjeet Bhullar, who won the Indonesia Invitational,

when he was 21 years and three months, while Shubhankar Sharma won Joburg Open when he was 21 years and five months

old.

11) Ministry of Railways released a report of cleanliness.

Which among the following tops in the A1 category?

Ans:- Jodhpur railway station

Explanation:- In a cleanliness report released by the Ministry of Railways, Jodhpur and Jaipur of North Western Railway in

Rajasthan captured the first two spots in the A1 Category stations. Tirupati, an important site for religious tourism and in

South Central Railway, ranked third. Rajasthan's Marwar topped among the A Category railway stations, followed by Phulera and

Warangal. There are over 8,000 stations on the Indian Railway

Page 74: S S C E Monthly CurrentAffair(August) · SSCE 8981426494/8296260082 Downloaded from:- Page 1 S S C E Monthly CurrentAffair(August) Contact : 08981426494/08296260082 Visit us: - www

SSCE 8981426494/8296260082 Downloaded from :- www.onlinessce.com Page 74

network and they are classified into seven categories: A-1, A, B, C,

D, E and F based on their annual passenger revenue.

12) Name of the Football Player who has been named as the as

captain of Barcelona?

Ans:- Lionel Messi

Explanation:- Lionel Messi has succeeded Andres Iniesta as captain of Barcelona, after the Spaniard ended his 22-year stay at

the club to join Japanese side Vissel Kobe in May. Sergio Busquets will be the vice-captain, with Gerard Pique and Sergi

Roberto the third and fourth captains respectively. Messi, 31,

joined the Barcelona club in 2001 and is their record goalscorer.

13) ______________ is planning to launch a TV channel to promote scientific temper among people and conduct training

camps for school students.

Ans:- ISRO

Explanation:- The Indian Space Research Organisation (ISRO) is

planning to launch a TV channel to promote scientific temper among people and conduct training camps for school students.

ISRO will select students and organise training camps for 25 to 30 days and the students will be allowed to visit laboratory and make

their small satellite. ISRO was planning to set up an incubation

centre for startups.

14) Which city hosted the Nepal-India Literature Festival

2018?

Ans:- Birgunj

Explanation:- The Nepal-India Literature Festival 2018 was held

in Birgunj, Nepal on August 12 to strengthen Indo-Nepal ties. On this occasion, the renowned writers from both the countries were

also felicitated. Approx 250 participants from India and Nepal attended the two day festival. The event was organized by Nepal

India Co-operation Forum.

Page 75: S S C E Monthly CurrentAffair(August) · SSCE 8981426494/8296260082 Downloaded from:- Page 1 S S C E Monthly CurrentAffair(August) Contact : 08981426494/08296260082 Visit us: - www

SSCE 8981426494/8296260082 Downloaded from :- www.onlinessce.com Page 75

15) Name the bill that was passed in the Parliament that seeks

to clarify that allottees under a real estate project should be

treated as financial creditors.

Ans:- Insolvency and Bankruptcy Code (Second Amendment)

Bill, 2018

Explanation:- Parliament has passed the Insolvency and

Bankruptcy Code (Second Amendment) Bill,2018 with the Rajya Sabha's approval. The Bill seeks to clarify that allottees under a

real estate project should be treated as financial creditors. Home buyers will get due representation in the Committee of Creditors

and make them an important part of decision making process. Setting up of a special dispensation for small sector enterprises

has also been made.

Daily Current Affairs15,August,2018

1) Karnataka government launched a drive under Hasiru Karnataka programme on 15th August to increase the tree

cover of the city and the district. It is named as;

Ans:- Green Mysuru drive

Explanation:- The Forest Department of Karnataka is to launch a Green Mysuru drive on 15th August 2018 under Hasiru

Karnataka programme. The aim is to increase the tree cover of the city and the district. The program involves planting saplings in

vacant spaces of government offices and schools, available space

on the roadside, embankment abutting farmland and agricultural fields, courtyards etc. The drive considers the participation public

as its key success of the programme. A total of 100,000 saplings have been prepared in various nurseries of the Forest Department.

In the year 2018-19, the Forest Department has distributed around 22.21 lakh saplings and 17.18 lakh tree guards to the

public. As per the National Forest Policy, at least 33% of the land

Page 76: S S C E Monthly CurrentAffair(August) · SSCE 8981426494/8296260082 Downloaded from:- Page 1 S S C E Monthly CurrentAffair(August) Contact : 08981426494/08296260082 Visit us: - www

SSCE 8981426494/8296260082 Downloaded from :- www.onlinessce.com Page 76

area should be under green cover wherein Karnataka has the

green cover of 22.56%.

2) Who was elected as vice president of South West Asian

Football Federation?

Ans:- Subrata Dutta

Explanation:- All India Football Federation (AIFF) senior vice president Subrata Dutta was elected as a vice president of the

newly formed South West Asian Football Federation (SWAFF). He has been elected for a period of 4 years. The President of SWAFF is

Dr. Adel Ezzat. He is also, the President and Chairman of Saudi

Arabian Football Federation. SWAFF consists of 10 member countries: India, Maldives, Saudi Arabia, United Arab Emirates

(UAE), Bahrain, Bangladesh, Kuwait, Pakistan, Sri Lanka and

Yemen.

3) Whom will be conferred the Shaurya Chakra award?

Ans:- Rifleman Aurangzeb and Major Aditya Kumar

Explanation:- Rifleman Aurangzeb and Major Aditya Kumar of Indian Army will be conferred the Shaurya Chakra along with 18

others Armed Forces Personnel and members of Paramilitary Forces. Aurangzeb, who has been selected posthumously, was

abducted and later killed by terrorists in Jammu and Kashmir. Besides, Sepoy Vrahma Pal Singh will be awarded with Kirti

Chakra posthumously. President Ram Nath Kovind has approved these names on the eve of Independence Day along with three Bar

to Sena Medals (Gallantry), 93 Sena Medals (Gallantry) and, 11 Nao Sena Medals (Gallantry). The President also conferred Vayu

Sena Medal (Gallantry) to Group Captain Abhishek Sharma, Squadron Leader Vernon Desmond Keane and Sergeant

Shashidhar P Prasad.

4) Which city tops in the Economist Intelligence Unit's Global

Liveability Index?

Ans:- Vienna

Page 77: S S C E Monthly CurrentAffair(August) · SSCE 8981426494/8296260082 Downloaded from:- Page 1 S S C E Monthly CurrentAffair(August) Contact : 08981426494/08296260082 Visit us: - www

SSCE 8981426494/8296260082 Downloaded from :- www.onlinessce.com Page 77

Explanation:- Vienna has replaced Melbourne for the first time at

the top of the Economist Intelligence Unit's Global Liveability Index. Damascus retained the last place, followed by the Dhaka,

and Lagos in Nigeria. The survey does not include several of the

world's most dangerous capitals, such as Baghdad and Kabul.

5) Who among the following was ranked first in the 'Kotak

Wealth Hurun-Leading Wealthy Women 2018 list?

Ans:- Smitha V. Crishna

Explanation:- Smitha V. Crishna was ranked first in the 'Kotak

Wealth Hurun-Leading Wealthy Women 2018 list'. Roshni Nadar,

the CEO and executive director at HCL, ranks second in the list. The chairperson of the Times Group, Indu Jain, ranks third in the

list. Kiran Mazumdar-Shaw, founder and MD of Biocon, ranks

fourth in the list.

6) Which Indian Film Actor becomes Brand Ambassador for

Road Safety campaigns?

Ans:- Akshay Kumar

Explanation:- Union Minister for Road Transport & Highways,

Shipping, Water Resources, River Development and Ganga Rejuvenation Nitin Gadkari launched three short films for

generating public awareness towards road safety. Film actor Shri Akshay Kumar has featured in these films, which are directed by

Shri R.Balki. Mr Gadkari announced the appointment of Shri

Akshay Kumar as Road Safety Brand Ambassador.

7) International left handers day is observed on which date?

Ans:- Aug 13

Explanation:- August 13 is celebrated as International left handers day annually. It aims to celebrate the uniqueness and

differences of the left handers. The day was first observed in the year 1976 by Dean R. Campbell, founder of the Lefthanders

International, Inc.

Page 78: S S C E Monthly CurrentAffair(August) · SSCE 8981426494/8296260082 Downloaded from:- Page 1 S S C E Monthly CurrentAffair(August) Contact : 08981426494/08296260082 Visit us: - www

SSCE 8981426494/8296260082 Downloaded from :- www.onlinessce.com Page 78

8) PV Sindhu ranked __________ in the BWF 2018 ranking.

Ans:- 3rd

Explanation:- In the latest Badminton World Federation (BWF) singles rankings, P V Sindhu remained static at the third spot,

Saina Nehwal lost a place to be ranked 11th now. Kidambi Srikanth slipped two places to eighth. Viktor Axelsen from

Denmark and Tai Tzu Ying from Chinese Taipei ranked first in

BWF Men's Singles and Women's Singles, respectively.

9) The Indian Banks' Association has appointed __________ as a

'Knowledge Partner' to the Banks Board Bureau.

Ans:- Egon Zehnder International

Explanation:- The Indian Bank's Association has appointed Egon

Zehnder International as a 'Knowledge Partner' to the Banks Board Bureau. It will design, implement and institutionalise a

flagship leadership development strategy for public sector banks in India. In addition, Hay Consultants Private Limited will assist

the bureau in assessing leadership competencies of candidates.

10) Name of the country which is first to participate in SCO

Peace Mission 2018 exercise at Russia is;

Ans:- India

Explanation:- India will participate for the first time in the joint SCO Peace Mission Exercise, a part of Shanghai Cooperation

Organization (SCO) initiatives, being conducted by Central Military Commission of Russia from August 22-29 at Chebarkul. India

became a full member of the SCO in June 2017. The Shanghai

Cooperation Organization (SCO) is an eight-member multilateral organization, established on 15 June 2001 in Shanghai, China. 8

members of SCO are:- China, Kazakhstan, Kyrgyzstan, Russia,

Tajikistan, Uzbekistan, India and Pakistan.

11) Who was appointed the Chief Executive Officer (CEO) of

Pradhan Mantri Fasal Bima Yojana (PMFBY)?

Page 79: S S C E Monthly CurrentAffair(August) · SSCE 8981426494/8296260082 Downloaded from:- Page 1 S S C E Monthly CurrentAffair(August) Contact : 08981426494/08296260082 Visit us: - www

SSCE 8981426494/8296260082 Downloaded from :- www.onlinessce.com Page 79

Ans:- Ashish Kumar Bhutani

Explanation:- Ashish Kumar Bhutani was appointed the Chief Executive Officer (CEO) of Pradhan Mantri Fasal Bima Yojana

(PMFBY). He is an IAS officer of the 1992 batch of Assam-Meghalaya cadre. He has been appointed the CEO of PMFBY till

9th May 2020. PMFBY provides a comprehensive insurance cover against failure of the crop and helps to stabilise the income of the

farmers.

12) This country test fired and unveiled the latest generation

of its Fateh ballistic missile named Fateh Mobin.

Ans:- Iran

Explanation:- Iran has successfully test fired and unveiled the latest generation of its Fateh ballistic missile named Fateh Mobin

or Bright Conqueror. It is a short-range ballistic missile. It has a range between 300-500 kilometers. It is capable of striking targets

on land and sea. It has been indigenously designed. It is an agile,

radar-evading and tactical missile with pinpoint accuracy.

13) Which country has decided to recommence the Adoption

Programme with India, as per Hague Convention on Inter-

Country Adoption?

Ans:- Australia

Explanation:- The Australia Government has decided to recommence the Adoption Programme with India, as per Hague

Convention on Inter-Country Adoption. The Australian Government had put on hold adoptions from India, 8 years ago,

due to allegations of child trafficking for Inter-country adoption by some recognized Indian placement agencies. Inter-country

adoption regulations have been made stricter by the Indian Government with the passing of Juvenile Justice Act, 2015 and

notification of Adoption Regulations, 2017.

14) Pitch to MOVE" a mobility pitch competition to provide

budding entrepreneurs of India is launched by;

Page 80: S S C E Monthly CurrentAffair(August) · SSCE 8981426494/8296260082 Downloaded from:- Page 1 S S C E Monthly CurrentAffair(August) Contact : 08981426494/08296260082 Visit us: - www

SSCE 8981426494/8296260082 Downloaded from :- www.onlinessce.com Page 80

Ans:- NITI Aayog

Explanation:- NITI Aayog has launched "Pitch to MOVE" - a mobility pitch competition that aims to provide budding

entrepreneurs of India a unique opportunity to pitch their business ideas to a distinguished jury. Startups working in the

various fields of mobility can pitch their ideas to industry leaders and Venture Capitalists for raising investments. Winners of the

event will be felicitated by PM during the Global Mobility Summit. Pitch to MOVE aims to identify and incentivise the startups, which

will help the Government realize its vision of Shared, Connected,

Intermodal and Environment-Friendly Mobility for India.

15) This state government started a 'Tatkal sewa' for property

registration across the state.

Ans:- Punjab

Explanation:- The Punjab government started a 'Tatkal sewa' for

property registration across the state. Under this, people can seek appointment for 'tatkal' (instant) registration for properties for an

hour-long slot from 9 am to 10 am. The facilitation charges for the service would be Rs. 5000. 20 additional dedicated sub-registrars

had been deputed at Ludhiana (East), Jalandhar, Amritsar, Patiala, Kharar and some more districts. So far 1,62,410 online

documents have been registered under the new system.

16) Which of the following is the largest natural gas field in

the world?

Ans:- South Pars

Explanation:- The China National Petroleum Corporation (CNPC), the world's third-largest oil and gas firm, is likely to expand its

stake in an Iranian gas field. CNPC has replaced Total, the French energy giant, in the South Pars, world's single largest natural gas

field, Phase 11 project. In the project 2017, Total acquired a 50.1%, CNPC held a 30% stake and the rest was allocated to

Iran's Petropars Ltd. Meantime, Washington, U.S. demanded both Total and CNPC to scrap their businesses from Iran. Following the

Page 81: S S C E Monthly CurrentAffair(August) · SSCE 8981426494/8296260082 Downloaded from:- Page 1 S S C E Monthly CurrentAffair(August) Contact : 08981426494/08296260082 Visit us: - www

SSCE 8981426494/8296260082 Downloaded from :- www.onlinessce.com Page 81

U.S. framework, Total apparently withdrew from the project. The

option lies with Chinese firm the to take over Total's stake if it left Iran. Chinese Foreign Ministry opposed to "unilateral sanctions"

imposed by the U.S. against Iran and it is likely to expand its

stake in an Iranian gas field.

Daily Current Affairs16,August,2018

1) Who of the following become the 53rd Grandmaster of

India?

Ans:- Nihal Sarin

Explanation:- International Master Nihal Sarin became the 53rd Grandmaster of India despite losing his final round game to

Richard Rapport of Hungary in the ninth and final round of Abu Dhabi Masters. The 14-year-old Nihal tallied 5.5 points out of a

possible nine and the final GM norm came the Kerala boy's way

with one round to spare.

2) Which bank's server was under a malware attack for 2 days

and an amount of Rs 94 crore has been transferred off?

Ans:- Cosmos Bank

Explanation:- Cosmos bank announced that the server of Pune-

based bank was under a malware attack for 2 days between August 11 and August 13 and an amount of Rs 94 crore has been

transferred off. The fraudulent transactions were carried out via cloning of thousands of the bank's debit cards and 25 ATMs

located in Canada, Hong Kong and a few in India. The cards of the cooperative bank customers that were cloned were of Visa and

Rupay. The malware attack was on the switch, which is operative

for payment gateways of Visa and Rupay debit cards.

3) __________ and Airtel Payments Bank entered into an alliance to offer the 'Pradhan Mantri Jeevan Jyoti Bima

Yojana' (PMJJBY).

Ans:- Bharti Axa Life Insurance

Page 82: S S C E Monthly CurrentAffair(August) · SSCE 8981426494/8296260082 Downloaded from:- Page 1 S S C E Monthly CurrentAffair(August) Contact : 08981426494/08296260082 Visit us: - www

SSCE 8981426494/8296260082 Downloaded from :- www.onlinessce.com Page 82

Explanation:- Bharti Axa Life Insurance and Airtel Payments

Bank entered into an alliance to offer the 'Pradhan Mantri Jeevan Jyoti Bima Yojana' (PMJJBY), a government-backed life insurance

scheme. The alliance will leverage Airtel Payments Bank's vast network to reach deep rural pockets and serve the under-insured.

With this, Airtel Payments Bank also becomes the first payments bank in India to offer the Government of India-backed life

insurance scheme. The product (PMJJBY) provided by Bharti Axa Life Insurance offers Rs 2 lakh life insurance cover for a nominal

premium of Rs 330 per annum and can be purchased by all

existing or new Airtel Payments Bank savings bank account

holders between the ages of 18-50.

4) President Ram Nath Kovind approved Nao Sena (Gallantary)

medal for how many womens of Indian Naval Sailing Vessel

(INSV) Tarini?

Ans:- 6

Explanation:- President Ram Nath Kovind approved Nao Sena (Gallantary) medal for the 6 women crew members of Indian Naval

Sailing Vessel (INSV) Tarini, who circumnavigated the globe. Lt Cdr Vartika Joshi captained the 194-day mission. The mission

started on 10th September 2017. Nao Sena Gallantary medals were also announced for marine Lt Cdr Anil Raina and Kunal

Saini for anti-insurgency operations.

5) Madhya Pardesh Governor Anandiben Patel was sworn in as

Governor of which state?

Ans:-Chhattisgarh

Explanation:- Madhya Pardesh Governor Anandiben Patel was sworn in as Governor of Chhattisgarh. President Ram Nath Kovind

gave Anandiben Patel the additional charge as Governor of Chhattisgarh following the death of Chhattisgarh governor

Balramji Dass Tandon. Chhattisgarh High Court Chief Justice Ajay Kumar Tripathi administered the oath of office to Anandiben

Patel. Anandiben Patel will perform additional duties as

Page 83: S S C E Monthly CurrentAffair(August) · SSCE 8981426494/8296260082 Downloaded from:- Page 1 S S C E Monthly CurrentAffair(August) Contact : 08981426494/08296260082 Visit us: - www

SSCE 8981426494/8296260082 Downloaded from :- www.onlinessce.com Page 83

Chhattisgarh governor until regular arrangements for the office of

the governor of Chhattisgarh are made.

6) Supreme Court has approved proposal to use hologram-

based colour-coded stickers for vehicles of this city.

Ans:- Delhi

Explanation:- For better tackling of air pollution related issues in the Delhi, Supreme Court has approved the central government's

proposal to use hologram-based colour-coded stickers for vehicles in Delhi based on the fuel used. The vehicles that run on petrol

and CNG will have light blue stickers, while the diesel vehicles will

have orange stickers. The scheme is based on a model used in

Paris.

7) Who won Tamil Nadu Premier League 2018 title?

Ans:- Madurai Panthers

Explanation:- Madurai Panthers won their first Tamil Nadu Premier League 2018 title, at the MA Chidambaram Stadium in

Chennai. Madurai Panthers beat Dindigul Dragons by seven wickets in the final and won the 2018 Tamil Nadu Premier League

title. Madurai Panthers' Arun Karthik scored his sixth fifty of this

season and remained the highest run scorer in the tournament.

8) As per Ficci's survey 'Ficci's Economic Outlook Survey', the

Indian economy is expected to grow at __________ in the

current fiscal.

Ans:- 7.4

Explanation:- As per Ficci's survey 'Ficci's Economic Outlook Survey', the Indian economy is expected to grow at 7.4 per cent in

the current fiscal. It projects that GDP will grow at 7.4 per cent for 2018-19, with a minimum and maximum range of 7.1 per cent

and 7.5 per cent, respectively. The economic activity on the first quarter of the current fiscal would be 7.1 per cent. The study

Page 84: S S C E Monthly CurrentAffair(August) · SSCE 8981426494/8296260082 Downloaded from:- Page 1 S S C E Monthly CurrentAffair(August) Contact : 08981426494/08296260082 Visit us: - www

SSCE 8981426494/8296260082 Downloaded from :- www.onlinessce.com Page 84

further said the median growth forecast for agriculture and allied

activities has been put at 3 per cent for 2018-19.

9) The 2018 World Organ Donation Day (WODD) has been

observed on which date?

Ans:- August 13

Explanation:- The World Organ Donation Day (WODD) is observed every year on August 13 to encourage people to donate

their precious organs. Every year, about 5 lakh people die awaiting organ transplants due to non-availability of organs. There

is a wide gap between the number of transplants awaited and the

organs available. Organ donation is a process where an organ is transferred from the body of the one who is giving the organ

(donor) to the one who needs the organ (recipient). The donor can be living or deceased. Organs that can be donated are kidney,

lungs, heart, eye, liver, pancreas, cornea, small intestine, skin

tissues, bone tissues, heart valves and veins.

10) With which techonlogical giant Prasar Bharati join hands

to telecast 2018 Independence Day programme live online?

Ans:- Google

Explanation:- Prasar Bharati has tied up with Google and its

video sharing platform YouTube to telecast the 72nd Independence Day programme live online on 15th August 2018 to

increase digital presence. Prasar Bharati CEO Shashi Shekhar Vempati has stated that the live stream window will be seen on

the homepage of Google itself so that people do not have to look for a separate link to go to the Youtube channel. In a first-of-its-

kind digital drive, Prime Minister Narendra Modi's speech from Red Fort will be live streamed on both Google and YouTube. This

year, Doordarshan anchors will introduce the programme from Red Fort instead of the studios. Doordarshan will also translate

the PM's speech in over 20 regional languages. The Independence Day coverage will open with a song composed by Shankar

Mahadevan.

Page 85: S S C E Monthly CurrentAffair(August) · SSCE 8981426494/8296260082 Downloaded from:- Page 1 S S C E Monthly CurrentAffair(August) Contact : 08981426494/08296260082 Visit us: - www

SSCE 8981426494/8296260082 Downloaded from :- www.onlinessce.com Page 85

11) The Indian Naval Ship that is one a goodwill visit to Fiji is;

Ans:- INS Sahyadri

Explanation:- Indian Naval Ship (INS) Sahyadri which was on a good will visit to the island nation entered the Port of Suva on

13th August 2018. The naval ship is visiting the Republic of Fiji en route to India after participating in the Rim of the Pacific

Exercise (RIMPAC)-2018 organised at Hawaii. During the visit, the Indian naval ship personnel will have professional interaction with

their Republic of Fiji counterparts and the ship will have an 'Open

Day' for general public.

12) Who has been appointed as head coach of the women's

national cricket team till the ICC World T20?

Ans:- Ramesh Powar

Explanation:- Former India spinner Ramesh Powar has been

appointed as head coach of the women's national cricket team till the ICC World T20, scheduled to be held in November, by BCCI.

He was appointed interim coach of the team in July after the resignation of Tushar Arothe. He is the third coach of women's

team in past one and a half year after Purnima Rau and Tushar Arothe. Powar has played 31 ODIs and two Tests from 2004 to

2007, picking 34 wickets with a best of 54 in the 50-over format.

13) D Bala Venkatesh Varma has been appointed as the

Ambassador to __________.

Ans:- Russia

Explanation:- D Bala Venkatesh Varma has been appointed as

the Ambassador to Russia. He is currently serving as Indian envoy in Spain. He will succeed Pankaj Saran who has been appointed

as the deputy National Security Advisor.

14) Which bank increased the marginal cost of funds based lending rate (MCLR) by up to 0.10 per cent for various tenors,

recetly?

Page 86: S S C E Monthly CurrentAffair(August) · SSCE 8981426494/8296260082 Downloaded from:- Page 1 S S C E Monthly CurrentAffair(August) Contact : 08981426494/08296260082 Visit us: - www

SSCE 8981426494/8296260082 Downloaded from :- www.onlinessce.com Page 86

Ans:- IDBI Bank

Explanation:- IDBI Bank increased the marginal cost of funds based lending rate (MCLR) by up to 0.10 per cent for various

tenors. For overnight and one-month tenor loans, the MCLR are increased by 0.05 per cent each to 8 per cent and 8.25 per cent

respectively. The one year loan will be priced at 8.85 per cent, two

year and three year tenors will be priced at 8.90 per cent each.

15) Name the girl who became India's latest Woman

Grandmaster (WGM) recently.

Ans:- R Vaishali

Explanation:- R Vaishali became India's latest Woman

Grandmaster (WGM). She is the elder sister of R. Praggnanandhaa, who became the second youngest Grand Master

in the world in June 2018. She is 17 years old. She completed her third and final WGM norm in the Riga Technical University Open

chess tournament in Riga, Latvia.

Daily Current Affairs17,August,2018

1) Which city will get free WiFi and high-speed broadband under 'Smart City' project launched by the Union Home

Minister Rajnath Singh?

Ans- Delhi

Explanation:- Union Home Minister Rajnath Singh launched a

host of new Smart City projects in Delhi. The event took place at the Charkha Park in Connaught Place. Besides launching the two

projects to provide high speed connectivity in the city, the Union Home Minister also launched other projects under the Centre's

Smart City initiative. These projects include-- a solar tree, an ideation centre, smart poles, Amdedkar Vatika, two hi-tech

nurseries, 50 LED interactive screens, four mechanical road

sweepers, two litter picking machines and 10 auto tippers.

2) Which app partners with IRCTC app Rail connect to

facilitate convenient payments?

Ans:- PhonePe

Page 87: S S C E Monthly CurrentAffair(August) · SSCE 8981426494/8296260082 Downloaded from:- Page 1 S S C E Monthly CurrentAffair(August) Contact : 08981426494/08296260082 Visit us: - www

SSCE 8981426494/8296260082 Downloaded from :- www.onlinessce.com Page 87

Explanation:- India's fastest growing payments platform, PhonePe

entering into a partnership with Indian Railway Catering and Tourism Corporation (IRCTC) to facilitate convenient, fast and

secure payments through PhonePe on the IRCTC Rail connect Android app. The partnership will enable PhonePe's 100 million

plus users to now pay directly from their bank accounts using UPI, credit and debit cards and also the PhonePe wallet for their

travel bookings.

3) Which state government has decided to observe September as the "month of nutrition" to create awareness among people

living in the state?

Ans:- Rajasthan

Explanation:- The Rajasthan government has decided to observe

September as the "month of nutrition" to create awareness among people living in the state. From September, the students of Class 1

to 8 of government schools will be given milk daily instead of thrice a week for which additional funds of Rs 203 crore will be

allocated. Besides, students in the age group of 3-6 years, adolescent girls and pregnant women registered with Anganwadi

centres will also be provided milk thrice a week and the government will allocate Rs 100 crore for this. In the month of

nutrition, the awareness in rural as well as urban areas will be

created for nutrition.

4) For which South American country, Mario Abdo Benitez

was elected as the new President?

Ans:- Paraguay

Explanation:- Colorado Party's Mario Abdo Benitez was sworn in

as the new President of Paraguay for a five-year term. Benitez clinched the top spot after defeating his closest rival, Efrain Alegre

from the liberal opposition alliance, by four percentage points in

the nation's elections, which were held in April 2018.

5) The First project under the Swadesh Darshan Scheme "North East Circuit: Imphal & Khongjom" was inagurated in

__________.

Ans:- Manipur

Page 88: S S C E Monthly CurrentAffair(August) · SSCE 8981426494/8296260082 Downloaded from:- Page 1 S S C E Monthly CurrentAffair(August) Contact : 08981426494/08296260082 Visit us: - www

SSCE 8981426494/8296260082 Downloaded from :- www.onlinessce.com Page 88

Explanation:- The governor of Manipur Dr. Najma A. Heptulla,

inaugurated project "Development of North East Circuit: Imphal & Khongjom" implemented under the Swadesh Darshan Scheme of

Ministry of Tourism in Imphal, Manipur. It is the first project under the Swadesh Darshan Scheme being inaugurated in the

country. The project under inauguration was sanctioned by the Ministry of Tourism in September 2015 for Rs.72.30 Crores.

Under this project Ministry has carried out developmental works at a cost of Rs. 63.90 Crore. The project covers two sites i.e.

Kangla Fort and Khongjom.

6) This country plans to invest 15 Billion in Turkey's

Financial Markets and Banks?

Ans:- Qatar

Explanation:- Qatar King Sheikh Tamim bin Hamad Al Thani has pledged to make direct investment in Turkey and announced a 15-

billion US dollar investment into the country's financial markets and banks. The announcement came after holding talks with

Turkish President Recep Tayyip Erdogan in Ankara. Turkey is currently grappling with a currency crisis and heightened tensions

with the United States, its NATO ally. Sheikh Tamim is the first foreign head of state to visit Ankara since the stand-off with

Washington.

7) Railway Ministry launched Digital Screens for how many

stations?

Ans:- 22

Explanation:- In the wake of Prime Minister's vision for creating digital museums at stations using QR code, Ministry of Railway

has made "digital screens" operational at 22 stations on this Independence Day as an innovative low-cost solution to spread

awareness among public about the opulent heritage of Indian Railways. The project aims at showcasing the legacy of Indian

Railways through one to two-minute-long movie clips on digital LED screens at the entrance gate of railway stations and also at

different comfort areas. In addition, QR code based posters on Railway Heritage are also being displayed at these stations. A

passenger can scan the QR code on their mobile to view a

Page 89: S S C E Monthly CurrentAffair(August) · SSCE 8981426494/8296260082 Downloaded from:- Page 1 S S C E Monthly CurrentAffair(August) Contact : 08981426494/08296260082 Visit us: - www

SSCE 8981426494/8296260082 Downloaded from :- www.onlinessce.com Page 89

streaming video on their mobile on various facades of Railway

heritage.

8) How many gallantry awards were conferred to Armed Forces

Personnel and members of Paramilitary Forces, this year?

Ans:-131

Explanation:- Rifleman Aurangzeb has been honoured posthumously. He was abducted and killed by terrorists in

Pulwama district of Jammu and Kashmir in June 2018. Also, Sepoy Vrahma Pal Singh has been conferred with Kirti Chakra

posthumously. Also, Vayu Sena Medal (Gallantry) has been conferred on Group Captain Abhishek Sharma, Squadron Leader

Vernon Desmond Keane and Sergeant Shashidhar P Prasad. The President has approved 131 Gallantry Awards to Armed Forces

Personnel and members of Paramilitary Forces. This consists of: 1 Kirti Chakra, 20 Shaurya Chakras, three Bar to Sena Medals

(Gallantry), 93 Sena Medals (Gallantry), 11 Nao Sena Medals

(Gallantry) and three Vayu Sena Medals (Gallantry).

9) Who has won the Community Award in the Junior NBA

World Basketball Championship 2018?

Ans:- Sunishka Kartik

Explanation:- Sunishka Kartik from Bengaluru won the

Community Award in the 2018 Junior NBA World Basketball Championship, which concluded at the ESPN Wide World of

Sports Complex at Walt Disney World Resort near Orlando, USA, on August 12. She had top-scored with 17 points in the team's

impressive win over South America Combined in a pool game. The award recognizes players who are committed to give back to those

in need. The tournament featured the best 13 and 14-year-old

boys and girls from around the world in 32 teams.

10) Which country forms the 'Iran Action Group' for Post-

Nuclear Deal Policy?

Ans:- US

Explanation:- US secretary of state, Mike Pompeo announced the

formation of the Iran Action Group (IAG) to coordinate and run the country's policy towards Iran. The development follows President

Page 90: S S C E Monthly CurrentAffair(August) · SSCE 8981426494/8296260082 Downloaded from:- Page 1 S S C E Monthly CurrentAffair(August) Contact : 08981426494/08296260082 Visit us: - www

SSCE 8981426494/8296260082 Downloaded from :- www.onlinessce.com Page 90

Donald Trump's announcement of the unilateral withdrawal of the

United States from the multinational Iran nuclear deal. The aim behind the formation of IAG is to change the Iranian regime's

behaviour. Pompeo has named Brian Hook, the State

Department's director of policy planning, as the group's head.

11) The multi-purpose 'Barak 8' missile defence system was

jointly developed by India and __________.

Ans:- Israel

Explanation:- The multi-purpose Barak 8 missile defence system,

jointly developed by India and Israel, will be procured by the Israeli Navy to protect its economic zones and strategic facilities

from diversified threats and the sales for it have exceeded USD 5 billion. Barak-8 is an operational air and missile defense system

used by the Israeli Navy as well as by the Indian Navy and air forces. Barak 8 has been jointly developed by Israel Aerospace

Industries (IAI), India's DRDO, Israel's Administration for the Development of Weapons and Technological Infrastructure, Elta

Systems, Rafael and some other Indian defence companies.

12) The 24th World Congress of Philosophy (WCP) was held in

__________.

Ans:- Beijing

Explanation:- The 24th World Congress of Philosophy (WCP) was held in Beijing, China. It was organized by the International

Federation of Philosophical Societies, and Peking University. Its theme was "Learning To Be Human". This was the first time that

the quinquennial (once in five year) event was held in China.

13) Which state CM launched a scheme Rythu Bima for

farmers on 72nd Independence Day?

Ans:- Telangana

Explanation:- Telangana launches two major schemes on

Independence Day. 'Rythu Bima', aimed at providing life insurance cover of Rs 5 lakh to every farmer. 'Kanti Velugu', the entire

population of 3.7 crore will be provided free eye care.

14) Who inagurated the 3 mobile apps launched for e-filing of

cases?

Page 91: S S C E Monthly CurrentAffair(August) · SSCE 8981426494/8296260082 Downloaded from:- Page 1 S S C E Monthly CurrentAffair(August) Contact : 08981426494/08296260082 Visit us: - www

SSCE 8981426494/8296260082 Downloaded from :- www.onlinessce.com Page 91

Ans:- Dipak Misra

Explanation:- The E-committee of the Supreme Court headed by

Justice M B Lokur, launched 3 mobile applications for the benefit of litigants and lawyers. The apps were inaugurated by Chief

Justice of India Dipak Misra. These software have been developed on "open source technology" and thus has saved Rs 1,670 crore.

The litigants and the lawyers would be able to file petitions in almost 17,000 district and 'taluka' courts in the country. e-pay

would enable the litigants to pay court fees, costs imposed by

courts, maintenance in matrimonial cases through digital.

15) Good Governance Day is observed in India on the 25th

December every year. It is a honor of this leader's birth

anniversary.

Ans:- Atal Bihari Vajpayee

Explanation:- Atal Bihari Vajpayee (93), the veteran leader, prolific writer and former Prime Minister of India, has passed away

in New Delhi on 16th August 2018. Born on 25th December 1924 in Gwalior, he was first elected to the Lok Sabha in 1957 and had

a long parliamentary experience spanning over four decades. He was elected to the Lok Sabha nine times and two times for the

Rajya Sabha. Vajpayee was the Minister of External Affairs (MEA) in the cabinet of Prime Minister Morarji Desai. He served as the

PM of India between 1996 and 2004 in three non-consecutive terms viz. May 1996 (for 13 days), 1998-1999 (for 11 months) and

1999-2004. He was awarded the India's highest civilian honour 'Bharat Ratna' in 2015. His birthday is celebrated as the Good

Governance Day (GGD) by the Central Government since 2014 to foster awareness among the people of accountability in

government.

Daily Current Affairs18,August,2018

1) How many US scientists was confrerred with the 2018

Albany Medical Center Prize?

Ans:- 3

Explanation:- Three US scientists James Allison, Carl June and

Steven Rosenberg were declared winners 2018 Albany Medical

Page 92: S S C E Monthly CurrentAffair(August) · SSCE 8981426494/8296260082 Downloaded from:- Page 1 S S C E Monthly CurrentAffair(August) Contact : 08981426494/08296260082 Visit us: - www

SSCE 8981426494/8296260082 Downloaded from :- www.onlinessce.com Page 92

Center Prize in Medicine and Biomedical Research. They were

awarded in recognition of their research in immunology and translation of their ideas into effective therapies that have led to

innovative treatments for cancer, HIV and other diseases. They will share this $500,000 medical prize. They will receive the award

at the ceremony to be held in September 2018 in Albany, New

York.

2) Who of the following has been elected the Chair of the

International Nitrogen Initiative (INI)?

Ans:- N Raghuram

Explanation:- N Raghuram has been elected the Chair of the

International Nitrogen Initiative (INI), a global policy making initiative. N Raghuram is an Indian scientist and academician. He

is a professor at the GGS Indraprastha University, New Delhi. He has been the President of the Indian Nitrogen Group and a

Steering Committee member of the UNEP Global Partnership on Nutrient Management. He is the first Indian and Asian to be

elected as Chair of INI. He will succeed Mark Sutton as chair of

INI.

3) Who develops missiles to counter the military expansion of

China?

Ans:- Taiwan

Explanation:- Taiwan is building up and developing missiles and

interceptors in order to counter the military expansion of China. Since 2016, under the Presidentship of Tsai Ing-wen, Taiwan has

deployed one set of missiles, perfected another and sped production of a third. This development in its defence is to reduce

Beijing's military advantage over the self-ruled island. As a training mission, Chinese President Xi Jinping has sent warships,

bombers and fighter planes over the self-governed island in a

show of strength.

4) The Jakarta Palembang 2018 was begin in __________.

Ans:- Indonesia

Explanation:- This year's Asian Games, also known as Jakarta Palembang 2018, has started from 18th August to 02nd

Page 93: S S C E Monthly CurrentAffair(August) · SSCE 8981426494/8296260082 Downloaded from:- Page 1 S S C E Monthly CurrentAffair(August) Contact : 08981426494/08296260082 Visit us: - www

SSCE 8981426494/8296260082 Downloaded from :- www.onlinessce.com Page 93

September in the Indonesian cities of Jakarta and Palembang. It is

the second time when Jakarta is hosting the sports games after 1962. A total of 572 Indian athletes will be participating in the

event in 36 different sports categories. The contingent will be led

by Chef de Mission Brij Bhushan Sharan Singh.

5) Who is appointed as Chairman Of Marine Products Export

Development Authority ?

Ans:- Shri K S Srinivas

Explanation:- IAS officer K S Srinivas has taken over as

Chairman of Marine Products Export Development Authority (MPEDA). Prior to this appointment, he had served as Joint

Secretary in Department of Agriculture Cooperation and Farmer's

Welfare in Ministry of Agriculture.

6) According to BSE data, which company becomes the most

valued Indian company?

Ans:- TCS

Explanation:- According to BSE data, TCS overtakes RIL to

become most valued Indian company. The market capitalisation (m-cap) of TCS stood at Rs 7,69,065.04. RIL stood at Rs

7,60,644.73 crore. There is a difference of total Rs 8,420.31 crore

in m cap.

7) What is the position of India in the FIFA world rankings?

Ans:- 96th

Explanation:- Indian football team improved a rank and secured

96th position in the latest FIFA world rankings. India's improvement in the ranking is seen after the introduction of a new

ranking system called ELO by FIFA. As per ELO, points will be added or subtracted to a national team's already existing total

before a match.

8) Who launched a new campaign named 'Our Safety, Our Rights', among children against sexual abuse on International

Day of the Girl Child?

Ans:- Amnesty International India

Page 94: S S C E Monthly CurrentAffair(August) · SSCE 8981426494/8296260082 Downloaded from:- Page 1 S S C E Monthly CurrentAffair(August) Contact : 08981426494/08296260082 Visit us: - www

SSCE 8981426494/8296260082 Downloaded from :- www.onlinessce.com Page 94

Explanation:- Amnesty International India launched a new

campaign named 'Our Safety, Our Rights', among children against sexual abuse. The campaign will culminate on October 11 -

International Day of the Girl Child. The campaign in collaboration with Bharat Gyan Vigyan Samiti, People's Action for Rural

Awakening (PARA) and Institute of Social Education. 50,000 students, parents, teachers and children from 400 schools, across

four Indian states will be participating in this campaign. This include schools in Amnesty International India's Human Rights

Education network in Madhya Pradesh, Telangana, Tamil Nadu

and Karnataka.

9) This country releases additional funds for Nepal's Postal

Highway, recently.

Ans:- India

Explanation:- Indian Government released additional NPR 470

million for the Postal Highway being constructed in Southern Plains of Nepal. Indian Ambassador to Nepal Manjeev Singh Puri

handed over NPR 470 million to the Secretary of Nepal's Minister for Physical Infrastructure and Transport Madhusudan Adhikari.

This amount has been released for the ongoing construction of 14 road packages under Postal Highway Project. With this payment, a

total of NPR 2.35 billion has been released to Nepal out of the total grant assistance of NPR 8 billion committed by Indian Government

for implementing 14 road packages. With this payment, a total of NPR 2.35 billion has been released to Nepal out of the total grant

assistance of NPR 8 billion committed by Indian Government for implementing 14 road packages.With this payment, a total of NPR

2.35 billion has been released to Nepal out of the total grant assistance of NPR 8 billion committed by Indian Government for

implementing 14 road packages. Indian Mission in Nepal also gifted books to 68 libraries and educational institutions in Nepal

worth NPR 17 Lakhs.

10) With which hospital did Microsoft join to introduce an AI-

powered Cardiovascular Disease risk?

Ans:- Apollo Hospitals

Page 95: S S C E Monthly CurrentAffair(August) · SSCE 8981426494/8296260082 Downloaded from:- Page 1 S S C E Monthly CurrentAffair(August) Contact : 08981426494/08296260082 Visit us: - www

SSCE 8981426494/8296260082 Downloaded from :- www.onlinessce.com Page 95

Explanation:- Microsoft India and Apollo Hospitals together have

built an Artificial Intelligence powered CardioVascular Disease (CVD) risk score Application Program Interface (API). The API is

built on Microsoft's cloud computing platform Azure. It is the first of its kind in India. It has been designed to predict the risk of CVD

in the Indian population. The aim is to prevent heart disease, save lives. It is expected that AI can augment human ingenuity. It

would also enable doctors across the Apollo network of hospitals to drive preventive cardiac care across the country. The

collaboration is because Microsoft needed to partner (Apollo

hospitals) with a firm that can provide key ingredients such as relevant data and the right level of domain expertise in cardiology

to build the technology.

11) Who to be Honoured by Directors Guild of America?

Ans:- Ang Lee

Explanation:- Oscar-winning director Ang Lee will be one of the recipients of the annual Directors Guild of America (DGA) honours

at its ceremony. The event will be held at the DGA Theatre in New York City on October 18. Apart from the 63-year-old director, Fox

Searchlight chairman Nancy Utley, US Senator Amy Klobuchar, SAG-AFTRA senior adviser John McGuire and costume designer

Ann Roth will also be honoured at the ceremony.

12) Name of the Indian Cyclist who won Silver at 2018 UCI Junior Track Cycling World Championships in Aigle,

Switzerland.

Ans:- Esow Alben

Explanation:- Indian cyclist Esow Alben has won India's first

medal (a silver medal) at the 2018 UCI Junior Track Cycling World Championships in Aigle, Switzerland. In the Men's Keirin event,

Czech Republic's Jakub Stastny won gold medal. Esow Alben finished 0.017 seconds behind him and won silver medal.

Kazakhstan's Andrey Chugay won the bronze medal. Esow Alben also participated in the men's team sprint event with Mayur Pawar

and James Keithellakpam Singh. But the team did not qualify for the finals. Esow Alben is 17 years old. He hails from Andaman

Page 96: S S C E Monthly CurrentAffair(August) · SSCE 8981426494/8296260082 Downloaded from:- Page 1 S S C E Monthly CurrentAffair(August) Contact : 08981426494/08296260082 Visit us: - www

SSCE 8981426494/8296260082 Downloaded from :- www.onlinessce.com Page 96

and Nicobar islands. He trains at the Sports Authority of India's

National Cycling Academy in Delhi.

13) Name the person who was re-elected as president of Mali?

Ans:- Ibrahim Boubacar Keita

Explanation:- Ibrahim Boubacar Keita was re-elected as president

of Mali. Ibrahim Boubacar Keita won 67 percent of the vote and defeated his opponent Soumaila Cisse in the presidential election.

Ibrahim Boubacar Keita will serve his second five-year term as President of Mali. He is 73 years old. He will assume office on 4th

September 2018.

14) Indian Institute of Food Processing Technology's 'iCRAFPT

'18' was held in which city in Tamilnadu?

Ans:- Thanjavur

Explanation:- By commemorating its golden jubilee 'iCRAFPT '18' , the Indian Institute of Food Processing Technology (IIFPT),

started an international conference on recent advances in food processing technology till August 19, 2018 at its campus in

Thanjavur. It would be inaugurated by union minister of food processing industries, Harsimrat Kaur Badal. The theme of the

conference is: 'Doubling Farmers Income through Food

Processing'.

Daily Current Affairs19,August,2018

1) DRDO's light weight glide bomb Smart Anti Airfield Weapon

(SAAW) was tested near __________.

Ans:- Pokhran

Explanation:- The Defence Research and Development

Organisation (DRDO) has successfully tested indigenously developed light weight glide bomb Smart Anti Airfield Weapon

(SAAW) dropped from an Indian Air Force (IAF) aircraft. Total of three tests with different release conditions and ranges were

conducted at Chandan range near Pokhran in Rajasthan. Total of three tests with eighth round of developmental trails with different

release conditions were conducted during 16 to 18 August 2018

Page 97: S S C E Monthly CurrentAffair(August) · SSCE 8981426494/8296260082 Downloaded from:- Page 1 S S C E Monthly CurrentAffair(August) Contact : 08981426494/08296260082 Visit us: - www

SSCE 8981426494/8296260082 Downloaded from :- www.onlinessce.com Page 97

and all the mission objectives have been achieved. During the test,

weapon system was integrated with live warhead. It successfully

destroyed targets with high precision.

2) India's first Humboldt penguin was born in __________.

Ans:- Jijamata Udyan

Explanation:- India's first Humboldt penguin was born on 15th August at Mumbai's Veer Jijamata Udyan. It is the first penguin to

be born in the country. Eight Humboldt penguins were brought to Mumbai from South Korea in July 2016. The zoo authorities will

later conduct a DNA test to find out its gender and then name it.

3) The atmosphere of which exoplanet is made up of vaporized

iron and titanium?

Ans:- Kelt-9b

Explanation:- The astronomers carried out new observations of exoplanet Kelt-9b. Kelt-9b is the hottest exoplanet discovered and

was discovered in 2017. Astronomers discovered that the atmosphere is made up of vaporized iron and titanium. The team

used the Galileo National Telescope in La Palma, Canary Islands,

to observe the planet.

4) PM Narendra Modi announced immediate relief of Rs _______

crore to flood hit Kerala.

Ans:- Rs 500 crore

Explanation:- Prime Minister Narendra Modi announced on Saturday immediate financial assistance of Rs 500 crore for rain-

battered Kerala, after reviewing the flood situation in the state. Modi also announced an ex-gratia of Rs 2 lakh per person to the

next of kin of the deceased and Rs 50,000 to those seriously

injured from the Prime Minister's National Relief Fund.

5) The 11th World Hindi Conference has started in which of

the Mauritius city?

Ans:- Port Louis

Explanation:- The 11th World Hindi Conference has started in

Port Louis the capital of Mauritius. At the beginning of the conference, a two-minute silence was observed to pay tribute to

Page 98: S S C E Monthly CurrentAffair(August) · SSCE 8981426494/8296260082 Downloaded from:- Page 1 S S C E Monthly CurrentAffair(August) Contact : 08981426494/08296260082 Visit us: - www

SSCE 8981426494/8296260082 Downloaded from :- www.onlinessce.com Page 98

former Prime Minister Atal Bihari Vajpayee. After the inauguration

session, a condolence meeting will be held in the memory of former Prime Minister Atal Bihari Vajpayee. In the afternoon

sessions, delegates from 20 countries including India will deliberate on eight sub-topics related to "Hindi World and Indian

Culture".

6) Which team won the South Asian Football Federation

(SAFF) Under-15 Women's Championship in Thimpu, Bhutan?

Ans:- India

Explanation:- India defeated Bangladesh in the final of the South Asian Football Federation (SAFF) Under-15 Women's

Championship in Thimpu, Bhutan. A solitary second-half strike by Sunita Munda helped the Indian U-15 team clinch the title.

This is the second edition of the Championship. Bangladesh had won the inaugural edition last year, beating India, 1-0 in the final

played at Dhaka.

7) The Biennia multinational air exercise PB-18 was hosted

by;

Ans:- Royal Australian Air Force

Explanation:- Pitch Black - 18 (PB-18), biennia multinational air exercise hosted by Royal Australian Air Force (RAAF) concluded

recently Darwin, Australia. Exercise Pitch Black 18 involves the participation of more than 4,000 personnel and 140 military

aircraft from different countries across the globe. The exercise is part of the USAF's biennial, capstone International Engagement

activity with participation from nearly 15 airforces from different regional and coalition nations. The participating nations include

Canada, France, Germany, India, Japan, Republic of Korea, New

Zealand, Philippines, and the US amongst others.

8) Who is the world's highest paid female actress according to

Forbes?

Ans:- Scarlett Johansson

Explanation:- Actress Scarlett Johansson has emerged as the

world's highest paid female actress according to Forbes. Scarlett Johansson is 33 years old. She earned $ 40.5 million in pre-tax

Page 99: S S C E Monthly CurrentAffair(August) · SSCE 8981426494/8296260082 Downloaded from:- Page 1 S S C E Monthly CurrentAffair(August) Contact : 08981426494/08296260082 Visit us: - www

SSCE 8981426494/8296260082 Downloaded from :- www.onlinessce.com Page 99

earnings from June 1, 2017, to June 1, 2018. Angelina Jolie has

ranked second with $28 million and actress Jennifer Aniston, is in

third position with $19.5 million.

9) Which country successfully test fires an anti-tank guided

missile "HeliNa"?

Ans:- India

Explanation:- India successfully test-fired its indigenously

developed HeliNa anti-tank missile at the Pokhran Range in Rajasthan. The HeliNa stands for Helicopter-launched Nag, and is

a variant of the land-launched Nag anti-tank guided missile. The HeliNa is designed to be launched from the helicopters operated

by the Indian Army Aviation Corps. It has been designed to be launched from the HAL Light Combat Helicopter, the HAL Dhruv

and its armed variant, HAL Rudra.

10) Who is the head of the panel formed to review red-labelling draft of Food Safety and Standards Authority of India

(FSSAI)?

Ans:- B Sesikeran

Explanation:- Abill for red-labelling of packaged products

containing fat, sugar and salt has been halted by the government as per FSSAI's report. Additionally, a group of experts from health

and nutrition background has been set up to look into the issue of labeling. The group is headed by B Sesikeran, former director of

National Institute of Nutrition (NIN) as well as the current director

Hemalatha and doctor Nikhil Tandon.

11) Nobel Peace Prize awardee Kofi Annan passed away

recently, he was the former secretery of this organisation.

Ans:- UN

Explanation:- Former United Nations Secretary-General Kofi Annan has passed away at the age of 80. Annan, who was the

seventh Secretary-General of the United Nations, served at the UN from 1997 to 2006. He was the first to emerge from the ranks of

United Nations staff. Annan, of Ghanaian nationality, died in

hospital in Bern, Switzerland. Annan was awarded the Nobel

Page 100: S S C E Monthly CurrentAffair(August) · SSCE 8981426494/8296260082 Downloaded from:- Page 1 S S C E Monthly CurrentAffair(August) Contact : 08981426494/08296260082 Visit us: - www

SSCE 8981426494/8296260082 Downloaded from :- www.onlinessce.com Page 100

Peace Prize along with the UN in 2001 "for their work for a better

organized and more peaceful world".

12) Who has been named as the 'Best Sportsperson of India' by

Calcutta Sports Journalists' Club?

Ans:- Sunil Chhetri

Explanation:- India football team captain Sunil Chhetri, 34-year-old, is currently taking part in Bengaluru FC's pre-season training

in Spain. Special awards would also be given to Hima Das for winning the 400m gold in the under-20 world championship and

to Dipa Karmakar for finishing on top on her comeback event in Turkey. The other Award Winners are: Best Junior Sportsperson

of the Country: Ishan Porel Lifetime Achievement awards: Aroon Ghosh, Arun Lal, Akhtar Ali Best athlete (men): Safiqul Mondol;

Best athlete (women): Bhairabi Roy.

13) India's first mission to study the solar corona, that is planned to be launched in 2019-2020, by Polar Satellite

Launch Vehicle (PSLV-XL) from Sriharikota is;

Ans:- Aditya-L1

Explanation:- ISRO's Aditya-L1 mission, India's first mission to

study the solar corona, is planned to be launched in 2019-2020, by Polar Satellite Launch Vehicle (PSLV-XL) from Sriharikota,

Andhra Pradesh. Aditya - L1 will be inserted in a halo orbit around the Lagrangian point 1 (L1) that is 1.5 million km from the

Earth. Solar corona is the outer layers of the Sun. Lagrangian point is a point where the attraction by the Sun and the Earth

becomes equal.

14) Who developed the new version of UPI (Unified Payments

Interface) version 2.0?

Ans:- NPCI

Explanation:- RBI Governor Urjit Patel unveiled UPI version 2.0.

The new version has been developed by the National Payments Corporation of India (NPCI). It will allow the customers to use

overdraft limit for making payments for merchants. This has been

made in need for ensuring adequate importance on cyber security. UPI 2.0 member banks as on date are: State Bank of India, HDFC

Page 101: S S C E Monthly CurrentAffair(August) · SSCE 8981426494/8296260082 Downloaded from:- Page 1 S S C E Monthly CurrentAffair(August) Contact : 08981426494/08296260082 Visit us: - www

SSCE 8981426494/8296260082 Downloaded from :- www.onlinessce.com Page 101

Bank, Axis Bank, ICICI Bank, IDBI Bank, RBL Bank, YES Bank,

Kotak Mahindra Bank, IndusInd Bank, Federal Bank and HSBC.

15) Name the mountain which is climbed by the Indian Army recently to achieve greater heights in the field of

mountaineering.

Ans:- Mt Makalu

Explanation:- To achieve greater heights in the field of

mountaineering, Indian Army is planning to attempt fifth highest mountain peak in the world - Mt Makalu (8,485 metres) in 2019.

Under the protection of Army Adventure Wing, an expedition to Mount Kamet (7756 meters), Joshimath (district Chamoli,

Uttarakhand), is also being conducted from August to September. Mount Kamet is the third highest peak of India after

Kanchanjunga and Nandadevi and the only highest peak available

to climb within the country.

Daily Current Affairs20,August,2018

1) Who inaugurates 'Panini Language Laboratory' in

Mauritius?

Ans:- Smt Sushma Swaraj

Explanation:- External Affairs Minister Sushma Swaraj inaugurates Panini Language Laboratory In Mauritius. The lab is

gifted by the Indian government and will help MGI in teaching Indian languages in Mauritius. Education Minister of Mauritius

Leela Devi Dookun and other dignitaries were also present on the

occasion.

2) Which country will host the prestigious Asia Cup 2018?

Ans:- UAE

Explanation:- The Board of Control for Cricket in India (BCCI) and the Emirates Cricket Board (ECB) signed an agreement for the

United Arab Emirates (UAE) to host the prestigious Asia Cup. The Asia Cup 2018 will have cricket teams from India, Pakistan, Sri

Lanka, Bangladesh and Afghanistan. The tournament will be held

from September 15 to September 28 in Abu Dhabi and Dubai.

Page 102: S S C E Monthly CurrentAffair(August) · SSCE 8981426494/8296260082 Downloaded from:- Page 1 S S C E Monthly CurrentAffair(August) Contact : 08981426494/08296260082 Visit us: - www

SSCE 8981426494/8296260082 Downloaded from :- www.onlinessce.com Page 102

3) Walmart completes acquisition of 77% stake in Flipkart for

__________.

Ans:- $16-billion

Explanation:- Walmart and Flipkart announced the completion of

the $16-billion deal. Walmart now holds 77 per cent stake in the Indian firm. Walmart will appoint five members to the eight-

member board of Flipkartout of which 2 will be independent directors, Initially, Steuart Walton, Judith McKenna and Dirk Van

den Berghe, regional CEO of Walmart Canada and Asia, will join

the Flipkart board. $16 billion investment includes a $2 billion of new equity funding to help accelerate the growth of the Flipkart

business.

4) Subhash Sheoratan Mundra has been appointed as

Independent Director for which of the following?

Ans:- Indiabulls Housing Finance

Explanation:- Subhash Sheoratan Mundra has been appointed as

Independent Director on the Board of Indiabulls Housing Finance Ltd (IBHFL) for a period of three years. The former Deputy

Governor of the Reserve Bank of India (RBI) and senior banker is not replacing an existing member on the Board. Mundra, a Post-

Graduate from the University of Poona, used to be the Chairman of Bank of Baroda and an Executive Director at Union Bank of

India. He left the RBI in July 2017.

5) On which date the World photography day is observed?

Ans:- August 19th

Explanation:- August 19 is celebrated as World photography day.

It is celebrated in order to inspire photographers to share their work and to celebrate the art of photography. The birth of this day

dates back to 1839 in France, but it came in to force in 2009.

6) Name of the company which becomes first PSU to transact

on TReDS.

Ans:- HAL

Explanation:- Hindustan Aeronautics (HAL) has become the first

public sector enterprise to make a transaction on the "TReDS

Page 103: S S C E Monthly CurrentAffair(August) · SSCE 8981426494/8296260082 Downloaded from:- Page 1 S S C E Monthly CurrentAffair(August) Contact : 08981426494/08296260082 Visit us: - www

SSCE 8981426494/8296260082 Downloaded from :- www.onlinessce.com Page 103

platform". TReDS is an online electronic institutional mechanism

for facilitating the financing of trade receivables of micro, small and medium enterprises (MSME) through multiple financiers. In

2017, the government mandated all major PSUs to join TReDS

platform.

7) Who has been awarded the prestigious WHO World No

Tobacco Day 2017 Award?

Ans:- S K Arora

Explanation:- Delhi government's additional director of health S K

Arora has been awarded the prestigious WHO World No Tobacco Day 2017 Award for his extraordinary contribution towards

tobacco control. Tobacco prevalence in Delhi has come down by 6.5% in the past six years. The National Health Policy 2017 has

set targets of relative reduction in tobacco prevalence by 15% by

2020 and 30% by 2025.

8) The Chief Financial Officer, M D Ranganath has recently

resigned. He was a CFO of which company?

Ans:- Infosys

Explanation:- IT major Infosys said its Chief Financial Officer M D

Ranganath has resigned from the company. Ranganath will continue in his current position till November 16, 2018.

Ranganath had taken over the said role after the then CFO Rajiv Bansal had quit in 2015. Ranganath has worked at Infosys for 18

years.

9) The Central Government launched the 'Digital Northeast

Vision Document 2022' in __________.

Ans:- Guwahati

Explanation:- The Central Government launched the 'Digital

Northeast Vision Document 2022' in Guwahati. North East Indian region has become the newest member of the Centre's flagship

programme 'Digital India' initiative. 'Digital Northeast India Vision 2022' envisages in implementing the initiative throughout the

North East through optimum utilization of information,

communication, and technology.

Page 104: S S C E Monthly CurrentAffair(August) · SSCE 8981426494/8296260082 Downloaded from:- Page 1 S S C E Monthly CurrentAffair(August) Contact : 08981426494/08296260082 Visit us: - www

SSCE 8981426494/8296260082 Downloaded from :- www.onlinessce.com Page 104

10) Apurvi Chandela and Ravi Kumar won _________ in 10m Air

Rifle mixed Team Shooting event in 2018 Asian Games.

Ans:- Bronze

Explanation:- Apurvi Chandela and Ravi Kumar won bronze in

10m Air Rifle mixed Team Shooting event in Asian Games 2018, Indonesia. The team scored 429.9. It was a maiden Asian Games

medal for Apurvi. Chinese Taipei's Yingshin Lin and Shaochuan Lu won the gold by scoring 494.1. The silver was won by Chinese

pair Ruozhu Zhao and Haoran Yang who scored 492.5. Korea and

Mongolia finished fourth and fifth respectively in the event held for

the first time at the Games.

11) Which Indian Army person conferred with the US 'Legion

Of Merit' 2018?

Ans:- Dalbir Singh Suhag

Explanation:- The United States government has awarded the Legion of Merit (Degree of Commander) to General Dalbir Singh

Suhag (Retired), of the Indian Army, for exceptionally meritorious service as chief of the Army staff, Indian Army. The award was

presented to General Suhag at Pentagon, Washington DC, in the presence of his family members and the Indian Military Attache to

the United States of America.

12) Who has been appointed as the head of the Iran Action

Group?

Ans:- Brian Hook

Explanation:- Mike Pompeo, the US secretary of state, has announced the creation of the Iran Action Group (IAG). IAG is a

dedicated group to coordinate and run the US's policy towards Iran following President Donald Trump's unilateral withdrawal

from a multinational nuclear deal with Tehran. Brian Hook, the Department of State's director of policy planning, has been

appointed as its head.

13) Who of the following will receive BAFTA's John

Schlesinger Britannia Award?

Ans:- Steve McQueen

Page 105: S S C E Monthly CurrentAffair(August) · SSCE 8981426494/8296260082 Downloaded from:- Page 1 S S C E Monthly CurrentAffair(August) Contact : 08981426494/08296260082 Visit us: - www

SSCE 8981426494/8296260082 Downloaded from :- www.onlinessce.com Page 105

Explanation:- British director Steve McQueen will receive BAFTA's

John Schlesinger Britannia Award. McQueen, who won an Oscar for 12 Years a Slave, would be this year's recipient of the John

Schlesinger Britannia Award for excellence in directing. The award's previous recipients include Danny Boyle, Christopher

Nolan, and Quentin Tarantino.

14) This IIT has decided to set up a Re-Water Research Center

to replenish and rejuvenate water resources?

Ans:- IIT Kharagpur

Explanation:- The IIT Kharagpur has decided to set up a Re-Water Research Center to replenish and rejuvenate water

resources. It will be seed-funded by two alumni members - Aneesh Reddy & Aditya Choubey. The purpose of the 'Aditya Choubey

Center for Re-Water Research' is to tackle the challenge of resolving two burning issues faced by urban India - sewage

disposal and access to clean potable water. The institute will set up an on-campus plant which will convert 1.35 million litre of

sewage water from the hostels to 1.2 million litre of potable water, on a daily basis. The project is being developed to attract potential

entrepreneurs and government agencies to take up sewage treatment at a large scale and with a business model for banks to

gain confidence in funding such businesses.

15) The theme for the 2018 World Humanitarian Day is;

Ans:- NotATarget

Explanation:- August 19th is annually observed as World

Humanitarian Day 2018. It is observed to pay tribute to aid workers who risk their lives for humanitarian services. The theme

for the 2018 World Humanitarian Day is : #NotATarget. The day was designated by the UN General Assembly to commemorate the

19 August 2003 bombing of the United Nations headquarters in

Baghdad, Iraq.

16) Which state decided to honour former PM Atal Bihari

Vajpayee with 3 awards?

Ans:- Madhya Pradesh

Page 106: S S C E Monthly CurrentAffair(August) · SSCE 8981426494/8296260082 Downloaded from:- Page 1 S S C E Monthly CurrentAffair(August) Contact : 08981426494/08296260082 Visit us: - www

SSCE 8981426494/8296260082 Downloaded from :- www.onlinessce.com Page 106

Explanation:- Madhya Pradesh Chief Minister Shivraj Chouhan to

have 3 awards after former PM Atal Bihari Vajpayee. The first award will be given for poets (as former PM was also a poet). The

second award will be for the outstanding contribution in the field of journalism. And the third award would be for good work in

administration.

17) Who won gold in the 2018 Asian Games 65kg freestyle

wrestling competition?

Ans:- Bajrang Punia

Explanation:- Bajrang Punia won gold in the final of the 65kg freestyle wrestling competition at the Asian Games 2018. He

defeated Japan's Daichi Takatani with a scoreline of 11-8. He became the winner of India's 140th Asiad gold. Punia is the sole

torch-bearer of Indian wrestling. Punia said that he has fixed his eyes on bigger targets, the first of which would be the World

Championships in October, with an Olympic quota at stake. He

also said he dedicates his win to former PM Atal Bihari Vajpayee.

Daily Current Affairs21,August,2018

1) Maxxis, Taiwan based company planned to launch how

many tyre units in India?

Ans:- 5

Explanation:- Maxxis India, the Indian subsidiary of Taiwan-based tyre-maker Maxxis, plans to launch 5 new manufacturing

plants in India. The company had been supplying Original Equipment Manufacturer (OEM) tyre fitments to HMSCI through

the Vietnam facility since 2016. But recently, it rolled out its first two-wheeler tyre consignment from its Sanand facility, Gujarat to

Honda Motorcycle and Scooter India (HMSCI), Gurgaon. The Sanand facility is currently manufacturing two-wheeler tyres and

tubes. It has a capacity to produce about 20,000 tyres and 40,000 tubes a day. The company now targets to supply one million tyres

to its OEM partners by the end of this year. It also aims to capture

at least 15% of the country's two-wheeler tyre market by 2023.

Page 107: S S C E Monthly CurrentAffair(August) · SSCE 8981426494/8296260082 Downloaded from:- Page 1 S S C E Monthly CurrentAffair(August) Contact : 08981426494/08296260082 Visit us: - www

SSCE 8981426494/8296260082 Downloaded from :- www.onlinessce.com Page 107

2) Which car company launched a new version of Ciaz 2018

recently?

Ans:- Maruti Suzuki

Explanation:- Maruti Suzuki launched a new version of Ciaz 2018

with prices starting from Rs.8.19 lakh. The mid-sized sedan is designed with a new 1.5-litre K15 petrol engine along with smart

hybrid technology featuring a lithium-ion battery. Manual, as well as automatic transmissions, are available for the petrol version.

The price of the manual variants of the petrol version is between

Rs.8.19 lakh and Rs.9.97 lakh and the automatic transmission version is between Rs.9.8 lakh and Rs.10.97 lakh. The fuel

efficiency of the newly launched Ciaz will be 21.56 km/litre. The diesel versions of the same would continue to be powered by the

original 1.3-litre diesel engine with hybrid technology. It would be

priced between Rs.9.19 lakh and Rs.10.97 lakh.

3) The Union Government has proposed to set up which committee to suggest changes to the JEE (Advanced)

examination?

Ans:- Bhaskar Ramamurthy committee

Explanation:-The Union Ministry of Human Resource and

Development (HRD) has proposed to set up a 5-member committee to suggest changes to the JEE (Advanced) examination in the

wake of an inadequate number of candidates qualifying the entrance test this year. The aim of the committee, which will be

headed by IIT-Madras director "Bhaskar Ramamurthy", is to develop a robust and scientifically designed entrance exam system

to test the potential of the candidates as well as to reduce their dependence on coaching institutes. The proposal is on the agenda

of the IIT Council, which is considered to be the top decision-making body of all the IITs. Additionally, the IIT-Council is

expected to discuss a suggestion made by one of the 23 premier engineering schools that the students who qualify JEE (Advanced)

should be allocated an institute and not the engineering branch.

According to the suggestion, the student should be allocated an engineering branch only in the second year, based on the aptitude

exhibited in the first year.

Page 108: S S C E Monthly CurrentAffair(August) · SSCE 8981426494/8296260082 Downloaded from:- Page 1 S S C E Monthly CurrentAffair(August) Contact : 08981426494/08296260082 Visit us: - www

SSCE 8981426494/8296260082 Downloaded from :- www.onlinessce.com Page 108

4) India-Thailand's 'Exercise Maitree 2018' concludes in

__________.

Ans:- Srinagar

Explanation:- The Indian and Thailand's armed forces concluded

a two-week long platoon level military exercise called 'Exercise Maitree 2018' in Jammu & Kashmir's Srinagar. The exercise was

an annual event designed to strengthen the partnership between armies of the two countries. The exercise began on August 6 and

comprised of cross-training period, which included familiarization

between the two armies to evolve drills and procedures that help in counter-insurgency and counter-terrorism operations in varied

terrains under the United Nations (UN) mandate.

5) Who wins Gold medal in 10m Air Pistol of Asian Games

2018?

Ans:- Saurabh Chaudhary

Explanation:- 16-Year-Old Saurabh Chaudhary became only the

fifth Indian shooter to claim a gold in the Asian Games history, beating a field of multiple world and Olympic champions in the

10m air pistol finals in Indonesia. Another Indian making his international debut, 29-year-old Abhishek Verma, shot 219.3 to

secure bronze.

6) What technology did Paytm launch recently?

Ans:- Paytm AI Cloud for India

Explanation:- One97 Communications, the parent entity of Paytm

launched "Paytm AI Cloud for India", an AI Cloud computing platform designed for developers, startups and enterprises. Paytm

AI Cloud will process and store all the data of the consumer locally in servers located only in India while conforming to the highest

security and privacy standards. For this cloud computing infrastructure, Paytm had partnered with Chinese technology

giant Alibaba. The company has committed around Rs.250 crore in this business. The AI Cloud computing platform will offer a

suite of business-centric apps for organisations that need solutions for cloud computing, ready-to-use services to automate

Page 109: S S C E Monthly CurrentAffair(August) · SSCE 8981426494/8296260082 Downloaded from:- Page 1 S S C E Monthly CurrentAffair(August) Contact : 08981426494/08296260082 Visit us: - www

SSCE 8981426494/8296260082 Downloaded from :- www.onlinessce.com Page 109

their workflow, easy to integrate payments, messaging and

customer engagement.

7) With which country Indian government agree to expand maritime co-operation to ensure peace & stability in Indo-

Pacific region?

Ans:- Japan

Explanation:- India and Japan have resolved to expand their

maritime cooperation and work together to ensure peace and stability in the Indo-Pacific region. Both sides also exchanged

views on the current security situation in the Indo-Pacific region,

including developments in the Korean Peninsula.

8) Who of the following was appointed the next Ambassador of

India to the Democratic Republic of Sao Tome and Principe?

Ans:- Shri Srikumar Menon

Explanation:- Srikumar Menon was appointed the next

Ambassador of India to the Democratic Republic of Sao Tome and Principe. Currently, he is the Ambassador of India to Angola. He

has been concurrently appointed the next Ambassador of India to the Democratic Republic of Sao Tome and Principe, with residence

in Luanda.

9) The first Indian woman wrestler to win gold in the Asian

Games is;

Ans:- Vinesh Phogat

Explanation:- India got its second Gold medal at the 18th Asian

Games in Indonesia when Vinesh Phogat defeated Japan's Yuki Irie in the final of freestyle 50 kg category in Women's Wrestling.

With this victory, Vinesh created history as she is the first Indian woman wrestler to win gold in the Asian Games. Vinesh Phogat

had entered the final after beating Uzbekistan's Yakhshimuratova Dauletbike on technical superiority. Bajrang Punia had given

India its first Gold of the Games in men's freestyle 65 Kg category.

10) This state government has launched Disaster Response

Force (DRF) vehicles.

Ans:- Telangana

Page 110: S S C E Monthly CurrentAffair(August) · SSCE 8981426494/8296260082 Downloaded from:- Page 1 S S C E Monthly CurrentAffair(August) Contact : 08981426494/08296260082 Visit us: - www

SSCE 8981426494/8296260082 Downloaded from :- www.onlinessce.com Page 110

Explanation:- Telangana Government has launched Disaster

Response Force (DRF) vehicles in Hyderabad to handle situations like flash floods, heavy rains, building collapse and fire mishaps.

The DRF vehicles will be parked at 24 locations with Greater Hyderabad Municipal Corporation (GHMC) DRF staff. It will

reached affected spots for relief work during necessary situations. DRF personnel will be available on the field for 24/7. Hyderabad is

the second city to have an exclusive Enforcement Vigilance

Disaster Management wing.

11) On which date, World Mosquito Day is observed?

Ans:- Aug 20th

Explanation:- World Mosquito Day observed on 20th August every year. This day is a honor of British doctor Sir Ronald Ross's

discovery in 1897 that female mosquitoes transmit malaria between humans. Mosquitoes have been around for over a

hundred million years. Mosquitoes spread diseases like malaria, West Nile virus, dengue, chikungunya, Zika among human beings.

The London School of Hygiene & Tropical Medicine holds Mosquito Day celebrations every year, including events such as parties and

exhibitions, a tradition dating back to as early as the 1930s.

12) Union Ministry of Women and Child Development declared this month as National Nutrition Month to mark the country's

fight against malnutrition.

Ans:- September 2018

Explanation:- Union Ministry of Women and Child Development

announced to observe September 2018 as the National Nutrition Month to mark the country's fight against malnutrition. The

month-long intensive campaign will be undertaken with an aim of reaching every household with message of nutrition - 'har ghar

posha'.

13) Historic Esala Maha Perehera festival begins in __________.

Ans:- Sri Lanka

Explanation:- In Sri Lanka, the ten-day festival of the historic

Esala Maha Perehera began in Kandy with a grand procession. The procession set off from the biggest Buddhist temple in Sri

Page 111: S S C E Monthly CurrentAffair(August) · SSCE 8981426494/8296260082 Downloaded from:- Page 1 S S C E Monthly CurrentAffair(August) Contact : 08981426494/08296260082 Visit us: - www

SSCE 8981426494/8296260082 Downloaded from :- www.onlinessce.com Page 111

Lanka, Sri Dalada Maligawa at an auspicious time. The Esala

Perahera is one of the oldest and grandest of all Buddhist festivals in Sri Lanka, featuring dancers, jugglers, musicians, fire-

breathers and lavishly decorated elephants.

14) As per All India Financial Inclusion Survey (NAFIS) by NABARD more than what per cent of rural households have

bank accounts?

Ans:- 88%

Explanation:- All India Financial Inclusion Survey (NAFIS) by

NABARD was released. According to the survey; more than 88 per cent of rural householdshave bank accounts. Only about 24per

cent of the 88% of rural households use ATM services at least once in three months, 7.4per cent of these households use debit

or credit card. 7.5per cent use cheque to make a payment at least

once in three months.

15) Pakistan Foreign Minister Qureshi called India for an

Uninterrupted dialogue in 2018 because;

Ans:- To resolve all issues

Explanation:- Pakistan's newly elected Foreign Minister Shah

Mahmood Qureshi extended the hand of friendship. He offered an Uninterrupted dialogue. This talk is aimed to resolve all

outstanding issues so that peace can be maintained between the countries. He also said that Pakistan is interested in resolving the

Kashmir issue and to rebuild ties with eastern and western neighbours and to create peace in the region. Mr.Qureshi

welcomed the wishes from India's External Affairs Minister Sushma Swaraj. He said Pakistan will bilaterally try to resolve all

issues with Afghanistan as it will be working through socio-

economic development.

Daily Current Affairs22,August,2018

1) Chhattisgarh's Chief Minister Raman Singh renamed this city as Atal Nagar to tribute the late former prime minister

Atal Bihari Vajpayee.

Ans:- Naya Raipur

Page 112: S S C E Monthly CurrentAffair(August) · SSCE 8981426494/8296260082 Downloaded from:- Page 1 S S C E Monthly CurrentAffair(August) Contact : 08981426494/08296260082 Visit us: - www

SSCE 8981426494/8296260082 Downloaded from :- www.onlinessce.com Page 112

Explanation:- In a tribute to the late former prime minister Atal

Bihari Vajpayee, the Chhattisgarh Cabinet approved the proposal to rename Naya Raipur as 'Atal Nagar'. Chief Minister Raman

Singh made the announcement after a Cabinet meeting. The Bilaspur University will be known as Atal Bihari Vajpayee

University while the Narrow Gauge Line will be called Atal Path. The Central Park, which is being constructed beside collectorate,

will be named as Atal Park

2) According to the annual FIBAC productivity report; which

two states lead the way in internet banking?

Ans:- Manipur and Telangana

Explanation:- Eastern and the Southern States such as Manipur

and Telangana lead the way in internet banking according to the annual FIBAC productivity report FIBAC productivity report has

been published by the Boston Consulting Group, FICCI and Indian Banks' Association. In Telangana, 21.7% of the savings

banks accounts carried out the financial transaction through

internet banking.

3) Name of the company which tops in Fortune's 'Change the

World' list.

Ans:- Reliance Jio

Explanation:- Mukesh Ambani-led telecom company Reliance Jio

secured the top spot in Fortune's global 'Change the World' list released on August 20. American pharma company Merck ranked

second while Bank of America ranked third on the list. Chinese

group Alibaba is ranked 5th on the list.

4) Indian shooter Sanjeev Rajput clinched a __________ medal

in the men's 50m Rifle in 2018 Asian Games.

Ans:- Silver

Explanation:- Indian shooter Sanjeev Rajput clinched a silver

medal in the men's 50m Rifle 3 Positions event at the 18th Asian

Page 113: S S C E Monthly CurrentAffair(August) · SSCE 8981426494/8296260082 Downloaded from:- Page 1 S S C E Monthly CurrentAffair(August) Contact : 08981426494/08296260082 Visit us: - www

SSCE 8981426494/8296260082 Downloaded from :- www.onlinessce.com Page 113

Games on 21st August, 2018. China won the gold medal with a

score of 453.3 while Japan bagged the bronze medal with 441.4. Rajput had won the gold medal at the 50m rifle 3 positions in the

Gold Coast Commonwealth Games 2018.

5) Union Ministry of Earth Sciences has announced to set up

Cyclone Warning Centre in __________, Kerala.

Ans:- Thiruvananthapuram

Explanation:- Union Ministry of Earth Sciences has announced to set up Cyclone Warning Centre in Kerala's capital

Thiruvananthapuram. It also proposed to set up C-band Doppler

weather radar at Mangalore, Karnataka. This comes in view of recent heavy rainfall and concerns over extreme weather activities

in future. At present, India Meteorological Centre (IMD) has six cyclone warning centres, four along the east coast at Chennai,

Visakhapatnam, Bhubaneswar and Kolkata, and two along the

west coast at Ahmedabad and Mumbai.

6) Who is the only Indian to feature in Forbes list of World's

top 10 highest-paid female athletes of 2018?

Ans:- PV Sindhu

Explanation:- With earnings of $8.5 million (nearly ₹60 crore), Rio

Olympic silver medallist PV Sindhu is the only Indian to feature in Forbes list of top 10 highest-paid female athletes of 2018. Placed

seventh, Sindhu is the only badminton player in the list, which features eight tennis players and one race car driver- Danica

Patrick. Twenty-three-time Grand Slam champion Serena Williams

tops the list, with $18.1-million total earnings.

7) How many new Governors are appointed by the President

Ram Nath Kovind recently?

Ans:- 7

Explanation:- President Ram Nath Kovind appointed governors

for seven states - Bihar, Haryana, Uttarakhand, Jammu and

Page 114: S S C E Monthly CurrentAffair(August) · SSCE 8981426494/8296260082 Downloaded from:- Page 1 S S C E Monthly CurrentAffair(August) Contact : 08981426494/08296260082 Visit us: - www

SSCE 8981426494/8296260082 Downloaded from :- www.onlinessce.com Page 114

Kashmir, Sikkim, Meghalaya, and Tripura. Governor Satya Pal

Malik has been transferred from Bihar to Jammu and Kashmir to replace NN Vohra, while senior BJP leader Lalji Tandon has been

appointed the Governor of Bihar, replacing Malik. 1. Satya Pal Malik - Jammu & Kashmir. 2. Tathagata Roy - Meghalaya

(Replaced Ganga Prasad). 3. Lalji Tandon - Bihar (Replaced Satya Pal Malik). 4. Ganga Prasad - Sikkim (Replaced Shriniwas Patil). 5.

Kaptan Singh Solanki -Tripura (Replaced Tathagata Roy). 6. Satyadev Narayan Arya - Haryana (Replaced Kaptan Singh

Solanki). 7. Baby Rani Maurya - Uttarakhand (Replaced K K Paul).

8) With with Airforce; Indian Air Force (IAF) planned to

conduct its joint air exercise to deepen defence and military

ties between the two countries?

Ans:- RMAF

Explanation:- The first-ever joint air exercise involving the Indian Air Force (IAF) and the Royal Malaysian Air Force (RMAF)

commenced at Subang Air Base in Malaysia. India has been eyeing a foothold at the strategically important Subang Air Base.

The main aim of the exercise seeks to deepen defence and military

ties between the two countries.

9) Who among the following has won the title of Cincinnati

Masters on 19 August 2018?

Ans:- Novak Djokovic

Explanation:- Novak Djokovic defeated Roger Federer in the finals

to win his first title of Cincinnati Masters. Novak Djokovic won the Wimbledon Championships in July 2018, was the runner-up of

the Cincinnati Masters on five occasions. Novak Djokovic became

the first player to win all nine ATP Masters 1000 tournaments.

10) The Advanced Medium Combat Aircraft, India's next

indigenous fighter, is expected to make its first flight by

Ans:- 2032

Page 115: S S C E Monthly CurrentAffair(August) · SSCE 8981426494/8296260082 Downloaded from:- Page 1 S S C E Monthly CurrentAffair(August) Contact : 08981426494/08296260082 Visit us: - www

SSCE 8981426494/8296260082 Downloaded from :- www.onlinessce.com Page 115

Explanation:- The Advanced Medium Combat Aircraft (AMCA),

India's next indigenous fighter, is expected to make its first flight by 2032.The aircraft will have a low radar cross-section, making it

difficult for the enemy to spot it. This is India's only fifth generation aircraft programme. The AMCA will feature geometric

stealth and will initially fly with two GE-414 engines.

11) Book titled '281 and Beyond' is the autobiography of;

Ans:- VVS Laxman

Explanation:- Veteran cricketer VVS Laxman, known for the

suppleness of his wrists, has penned down his autobiography

titled, "281 and Beyond". The book will be released this year in November. The book will be released by the Westland Publications.

The title has been borrowed from the Hyderabad stylist's fabulous series-turning innings of 281 against Australia at the Eden

Gardens in 2001

12) Who was the highest paid female athlete for a third consecutive year in 2018, as per the "List Of Highest Paid

Female Athletes 2018" by Forbes?

Ans:- Serena Williams

Explanation:- Serena Williams (with earnings of $18.1 million)

was the highest paid female athlete for a third consecutive year in 2018, according to "List Of Highest Paid Female Athletes 2018"

published by Forbes. Dane Caroline Wozniacki, who captured her maiden grand slam title at Australian Open 2018, was second on

the list with combined earnings of ($13 million). Indian badminton player P.V. Sindhu was at seventh place with earnings of $8.5

million.

13) With which company, Paytm joint hands to launch AI-

based cloud computing platform 'Paytm AI Cloud'?

Ans:- Alibaba

Page 116: S S C E Monthly CurrentAffair(August) · SSCE 8981426494/8296260082 Downloaded from:- Page 1 S S C E Monthly CurrentAffair(August) Contact : 08981426494/08296260082 Visit us: - www

SSCE 8981426494/8296260082 Downloaded from :- www.onlinessce.com Page 116

Explanation:- Digital payments firm Paytm has partnered with

China's Alibaba to launch AI-based cloud computing platform 'Paytm AI Cloud'. The platform will offer business-centric apps to

developers, startups and enterprises that need cloud-computing solutions including customer engagement and easy-to-integrate

payments. It will process and store all consumer data locally in servers located only in India. The company has committed around

Rs 250 crore in this business.

14) According to European Stability Mechanism, which

country has successfully exited its final, 3-year bailout

programme?

Ans:- Greece

Explanation:- According to European Stability Mechanism (ESM), Greece has successfully exited its final, 3-year bailout programme,

agreed in August 2015 to help it cope with the continued fallout from a debt crisis.The ESM has disbursed 61.9 billion euros over

3 years in support of macroeconomic adjustment and bank recapitalisation.The ESM is a European Union agency that

provides financial assistance.

15) Which centre won an award for Outstanding Contribution to Gold Policy Research in 2017-18 at the 15th India

International Gold Convention held in Kochi?

Ans:- India Gold Policy Centre (IGPC), IIM Ahmedabad

Explanation:- The India Gold Policy Centre (IGPC), IIM

Ahmedabad won an award for Outstanding Contribution to Gold Policy Research in 2017-18 at the 15th India International Gold

Convention held in Kochi. The award was received by Professor Arvind Sahay, Chairman of IGPC, along with his team. The

convention was held at Le Meridien Convention Centre. Around

400 delegates participated.

Daily Current Affairs23,August,2018

1) Which airport has emerged as the second fastest growing

airport in the world?

Page 117: S S C E Monthly CurrentAffair(August) · SSCE 8981426494/8296260082 Downloaded from:- Page 1 S S C E Monthly CurrentAffair(August) Contact : 08981426494/08296260082 Visit us: - www

SSCE 8981426494/8296260082 Downloaded from :- www.onlinessce.com Page 117

Ans:- Kempegowda International Airport

Explanation:- The Kempegowda International Airport (KIA) has

emerged as the second fastest growing airport in the world in the first half of 2018 in terms of actual growth in the number of

passengers. It has recorded 1,58,50,352 flyers during the six-month period. Only Tokyo's Haneda International has bettered

KIA's growth. The report was published by RoutesOnline, a company focussing on the quality and standards of aviation

globally. New Delhi's Indira Gandhi International Airport is placed sixth (with a growth of 32,76,183) while Hyderabad is placed 17th

in terms of actual growth of passengers (20,97,087 passengers).

2) The Indian team won __________ medal in Sepak takraw in

the 18th edition of Asian Games in Indonesia.

Ans:- Bronze

Explanation:- The Indian men's Sepak takraw team secured their maiden bronze medal after losing to Thailand at the ongoing 18th

edition of Asian Games in Indonesia. Sepak takraw is a traditional sport of the nations in Southeast Asia. Sepak takraw has been

part of the Asian Games since the 11th edition in Beijing 1990. A match is played by two Regus (teams) each made up of three

players.

3) When was World Senior Citizen Day celebrated?

Ans:- August 21

Explanation:- World Senior Citizen Day, 2018 will be observed worldwide on August 21. The history of International Senior

Citizens Day comes from the period of 1988. It was officially established by the former president of United States of America

i.e. Ronald Reagan. He had signed the 5847 promulgation on August 19, 1988, which manifested August 21 as the National

Senior Citizens Day.

4) Who inagurted the International Buddhist Conclave in New

Delhi?

Ans:- Ram Nath Kovind

Explanation:- President Ram Nath Kovind inaugurated the International Buddhist Conclave in New Delhi. The theme of the

Page 118: S S C E Monthly CurrentAffair(August) · SSCE 8981426494/8296260082 Downloaded from:- Page 1 S S C E Monthly CurrentAffair(August) Contact : 08981426494/08296260082 Visit us: - www

SSCE 8981426494/8296260082 Downloaded from :- www.onlinessce.com Page 118

conclave is 'Buddha Path - The Living Heritage' which aims to

showcase Buddhist Heritage in India and boost tourism to the Buddhist sites in the country. The Ministerial level delegation from

ASEAN countries will participate in the conclave along with delegates from 29 countries including US, UK, Germany, France

and Russia. This year Japan is the partnering country for the

Buddhist conclave.

5) Which space agency has confirmed presence of ice on Moon

using data from the Chandrayaan-I spacecraft?

Ans:- NASA

Explanation:- Using data from the Chandrayaan-I spacecraft, that

was launched by the Indian Space Research Organisation (ISRO) in 2008, NASA Scientists have confirmed the presence of frozen

water deposits in the darkest and coldest parts of the Moon's polar regions. Scientists used data from NASA's Moon Mineralogy

Mapper (M3) instrument aboard the Chandrayaan-1 spacecraft to identify three specific signatures that definitively prove there is

water ice at the surface of the Moon. According to the study published in the journal PNAS, the ice deposits are patchily

distributed and could possibly be ancient. The study said with enough ice sitting at the surface within the top few millimetres

water would possibly be accessible as a resource for future

expeditions to explore and even stay on the Moon.

6) Which institute has launched the 'Aditya Choubey Center

for Re-Water Research' to treat and convert sewage water into

potable water?

Ans:- IIT-Kharagpur

Explanation:- Indian Institute of Technology Kharagpur (IIT-Kharagpur) launched 'Aditya Choubey Center for Re-Water

Research' to treat and convert sewage water into potable water. Aditya Choubey Center for Re-Water Research has been set up by

a seed fund created by IIT alumni Anant Choubey and Aneesh Reddy. This has been launched to handle the issues of sewage

disposal in urban areas and access to clean potable water.

7) Home Minister assigned which Bureau to curb "sexual

violence" videos involving women and children?

Page 119: S S C E Monthly CurrentAffair(August) · SSCE 8981426494/8296260082 Downloaded from:- Page 1 S S C E Monthly CurrentAffair(August) Contact : 08981426494/08296260082 Visit us: - www

SSCE 8981426494/8296260082 Downloaded from :- www.onlinessce.com Page 119

Ans:- An important meeting was held under the head of Home

Minister Rajnath Singh to discuss recommendations on ways to curb "sexual violence" videos involving women and children.

National Crime Records Bureau (NCRB) would be the designated nodal agency for monitoring the complaints received on a

government portal that records child pornography and sexual violence videos. The NCRB would coordinate with service providers

such as Facebook, YouTube and WhatsApp to block malicious videos and contents. A notification has been issued to NCRB

under the Information Technology Act, 2000 to enable us take

action against such videos and to ask the service providers to

block the content.

8) Who become the first Indian woman to win an individual

gold in shooting at the 2018 Asian Games?

Ans:- Rahi Sarnobat

Explanation:- Indian shooter Rahi Sarnobat has become the first-ever Indian woman to win an individual shooting medal at the

2018 Jakarta Asian Games. She bagged gold in the 25m pistol final event at the Jakabaring Shooting Range in Palembang, after

prevailing in a double shoot-off at the 2018 Jakarta Asian Games. Earlier, in shooting, Saurabh Chaudhary won gold in the Men's

10m Air Pistol event. In wrestling, Bajrang Punia and Vinesh Phogat clinched gold in Men's freestyle 65 kg and Women's

freestyle 50 kg, respectively. India is currently placed 7th in the

tally with 12 medals, comprising 4 Gold, 3 Silver and 5 Bronze.

9) Who won the 24th Rajiv Gandhi National Sadbhavana Award

recently?

Ans:- Gopalkrishna Gandhi

Explanation:- Former West Bengal Governor Gopalkrishna

Gandhi was awarded the 24th Rajiv Gandhi National Sadbhavana Award on the occasion of 74th birth anniversary of former Prime

Minister Rajiv Gandhi, in New Delhi. Gopalkrishna Gandhi was awarded the 2018 Rajiv Gandhi National Sadbhavana Award for

promoting communal harmony and peace. The award includes a

citation and a cash award of Rs 10 lakh.

Page 120: S S C E Monthly CurrentAffair(August) · SSCE 8981426494/8296260082 Downloaded from:- Page 1 S S C E Monthly CurrentAffair(August) Contact : 08981426494/08296260082 Visit us: - www

SSCE 8981426494/8296260082 Downloaded from :- www.onlinessce.com Page 120

10) Who will be conferred with the 2018 UEFA President's

Award?

Ans:- David Beckham

Explanation:- Former England captain David Beckham will

receive the UEFA President's Award for his contribution to football and promoting the sport in "every corner of the planet". David

Beckham will become the third Englishman to receive the UEFA President's Award. Sir Bobby Robson (2002) and Sir Bobby

Charlton (2008) are the previous English winners.

11) The 4th BIMSTEC Summit to be held in __________.

Ans:- Kathmandu

Explanation:- The fourth Summit of Heads of State/Government

of Bay of Bengal Initiative for Multi-Sectoral Technical and Economic Cooperation (BIMSTEC) will be held at Kathmandu in

Nepal on August 30 and 31. Prime Minister Narendra Modi will participate in the summit. During the two-day Summit, leaders of

member states will review progress achieved so far in the areas of cooperation and provide guidance to the future work of the

BIMSTEC.

12) This state government will set up a befitting memorial to

former PM Atal Bihari Vajpayee.

Ans:- Maharashtra

Explanation:-Maharashtra government will also set up a study chair in the name of former prime minister Atal Bihari Vajpayee in

each of the 13 universities in the state. The state government created a corpus of `20 crore for setting up the study chair in the

name of Vajpayee in 11 state universities and two private varsities. The economy of Maharashtra is the largest state

economy in India with ?27.96 lakh crore (US$410 billion) in GDP

and a per capita GDP of ?180,000 (US$2,600).

13) Which country is the current chairperson of Bay of Bengal Initiative for Multi-Sectoral Technical and Economic

Cooperation (BIMSTEC)?

Ans:- Nepal

Page 121: S S C E Monthly CurrentAffair(August) · SSCE 8981426494/8296260082 Downloaded from:- Page 1 S S C E Monthly CurrentAffair(August) Contact : 08981426494/08296260082 Visit us: - www

SSCE 8981426494/8296260082 Downloaded from :- www.onlinessce.com Page 121

Explanation:- Bay of Bengal Initiative for Multi-Sectoral Technical

and Economic Cooperation (BIMSTEC) will be held at Kathmandu in Nepal on August 30 and 31. BIMSTEC is a regional economic

bloc comprising seven member states lying in the littoral and adjacent areas of the Bay of Bengal constituting a contiguous

regional unity. The group was formed in 1997. Bangladesh,

Bhutan, India, Nepal, Sri Lanka Myanmar and Thailand.

14) NGT directed __________ government to submit an action

plan on e-waste management.

Ans:- Uttar Pradesh

Explanation:- The National Green Tribunal (NGT) has directed the

Ministry of Environment and Forests (MoEF), the Uttar Pradesh Pollution Control Board and the Central Pollution Control Board

to submit an action plan on e-waste management within three months. NGT stated that as a measure to combat pollution and

protect the environment the scientific disposal of e-waste is an important factor. NGT, while disposing of a plea against

unauthorised recycling, collection, dismantling, burning, selling of e-waste and other solid waste on roadsides directed the State and

riverbanks in violation of Environment (Protection) Act, 1986 and E-Waste (Management) Rules, 2016. NGT said that electronic

waste accounted for 40% of lead and 70% of all heavy metal found in landfills. It also states that burning and selling of e-waste and

other solid waste has resulted in groundwater contamination and

air pollution.

15) The 2018 Men's Hockey World Cup will be hosted by;

Ans;'- Odisha

Explanation:- Odisha for the First time is Organizing the 2018

Men's Hockey World Cup, which will be the 14th edition of the Hockey World Cup field hockey tournament. It is scheduled to be

held from 28 November to 16 December 2018, at the Kalinga Stadium in Bhubaneswar, India. 16 Teams are to be participated

in the Tournament. The Hague in Netherland hosted the Previous

tournament in the year 2014,in which Australia won the title which is their third title and the Netherland became the Runner

Page 122: S S C E Monthly CurrentAffair(August) · SSCE 8981426494/8296260082 Downloaded from:- Page 1 S S C E Monthly CurrentAffair(August) Contact : 08981426494/08296260082 Visit us: - www

SSCE 8981426494/8296260082 Downloaded from :- www.onlinessce.com Page 122

Up.Only 12 Teams Participated in the Tournament. Earlier, a

nation-wide publicity campaign, 'Heartbeats for Hockey', was launched to showcase Bhubaneswar as a sporting venue and

Odisha as a tourist destination.Advertisements were put up on buses in London with the tagline of 'Odisha by Morning, Hockey

by Evening'.

Daily Current Affairs24,August,2018

1) Who has become the new brand ambassador of the Sanjay

Gandhi National Park (SGNP)?

Ans:- Raveena Tandon

Explanation:- The Maharashtra government has appointed actor Raveena Tandon as the brand ambassador of the Sanjay Gandhi

National Park (SGNP) to spread environmental awareness among citizens. The SGNP is spread over 103 Sq. Km. and is wedged

between the suburbs of Mumbai and Thane. It is home to more than 274 species of birds, 35 species of mammals, 78 species of

reptiles and amphibians, 170 species of butterflies and over a staggering 1,300 species of plants. The park is notable as one of

the major national parks existing within a metropolis limit and is one of the most visited parks in the world. It was formerly known

as Borivali National Park and was renamed SGNP in 1996, after

Sanjay Gandhi, the son of former Prime Minister Indira Gandhi.

2) CEPR and NITI Aayog's "2018 India Banking Conclave" was

held in ___________.

Ans:- New Delhi

Explanation:- Centre for Economic Policy Research (CEPR) and the government Think Tank NITI Aayog organised two-day 'India

Banking Conclave' (IBC) in New Delhi between August 23 and August 24, 2018. The main focus is On reform in the banking

sector. About Centre for Economic Policy Research (CEPR) : CEPR is network of over 1100 researchers who are based mainly in

universities throughout Europe and collaborate in research and its dissemination. It aims to enhance quality of economic policy-

making within Europe and beyond.

Page 123: S S C E Monthly CurrentAffair(August) · SSCE 8981426494/8296260082 Downloaded from:- Page 1 S S C E Monthly CurrentAffair(August) Contact : 08981426494/08296260082 Visit us: - www

SSCE 8981426494/8296260082 Downloaded from :- www.onlinessce.com Page 123

3) What is the name of the European wind survey satellite

launched from French Guiana?

Ans:- Aeolus

Explanation:- A new satellite that will use advanced laser

technology to track global winds and improve weather forecasts has been successfully put into orbit. The name of the satellite is

"Aeolus" - named after the guardian of wind in Greek mythology. The satellite "will probe the lowermost 30 kilometres of the

atmosphere in measuring winds around the Earth.

4) Name the bank which launched 'bond-i', the world's first bond to be created, allocated, transferred and managed by

using distributed ledger technology.

Ans:- World Bank

Explanation:- The World Bank launched 'bond-i', the world's first

bond to be created, allocated, transferred and managed by using distributed ledger technology. This two-year bond successfully

raised 110 million Australian dollars (USD 80.48 million). This was the first time that investors supported the World Bank's

development activities in a transaction that is fully managed using

the blockchain technology from start to finish.

5) How much amount did the European Union announce to aid

flood-hit Kerala to the Indian Red Cross Society (IRCS)?

Ans:- € 190,000

Explanation:- The European Union announced an offer of €

190,000 (Rs.1.53 crore) in aid funding to the Indian Red Cross Society (IRCS). It is to provide immediate relief to flood-affected

people in Kerala. The announcement was made by the office of the grouping that comprised 28 European countries. The funding is

part of the EU's overall contribution to the Disaster Relief Emergency Fund (DREF) of the International Federation of Red

Cross and Red Crescent Societies (IFRC). It is said the aid will directly benefit 25,000 people from some of the worst hit areas of

the state, adding a humanitarian expert from the European Civil Protection and Humanitarian Operations (ECHO) has also been

deployed to assess the situation in the state. As outbreaks of

Page 124: S S C E Monthly CurrentAffair(August) · SSCE 8981426494/8296260082 Downloaded from:- Page 1 S S C E Monthly CurrentAffair(August) Contact : 08981426494/08296260082 Visit us: - www

SSCE 8981426494/8296260082 Downloaded from :- www.onlinessce.com Page 124

vector-borne diseases such as dengue, chikungunya and malaria

are common following the flood, disease prevention and hygiene promotion activities are also being conducted. The mosquito nets

are being provided.

6) Who launched overdraft facility for medium and small

enterprises recently?

Ans:- Ujjivan SFB

Explanation:- Ujjivan Small Finance Bank announced the launch of its overdraft facility for micro and small enterprises (MSE). This

aimed at the MSEs who don't have easy access to formal credit. It would be offered to all MSEs with a turnover of Rs 50 lakh or

more. This interest rate on the overdraft is for one year with a minimum credit of Rs 11 lakh to a maximum of Rs 50 lakh

renewed on an annual basis. Ujjivan SFB also offers composite loans, where the overdraft facility can be combined with existing

MSE term loan products.

7) According to Forbes magazine, what is the position of

Akshay Kumar in the highest-paid actor list?

Ans:- 7th

Explanation:- According to Forbes magazine, Akshay Kumar has emerged as the seventh highest-paid actor in the world by earning

USD 40.5 million in 2018. Salman Khan ranked ninth place with USD 38.5 million. Hollywood star George Clooney tops the list of

'World's Highest-Paid Actors 2018' with $239 Million. Forbes list of the World's best-paid male actors 2018: George Clooney - $239m;

Dwayne Johnson - $119m; Robert Downey Jr - $79m; Chris Hemsworth - $64.5m; Jackie Chan -$45.5m; Will Smith - $42m;

Akshay Kumar -$40.5m; Adam Sandler - $39.5m; Chris Evans -

$34m; Salman Khan - $33.5m.

8) India Wins it 5th Gold in which of the following game at the

Asian Games History?

Ans:- Rowing

Explanation:- Indian rowers Sawarn Singh, Dattu Bhokanal, Om

Prakash and Sukhmeet Singh won the men's Quadruple Sculls to win India's fifth gold medal at the Asian Games 2018. The medal

Page 125: S S C E Monthly CurrentAffair(August) · SSCE 8981426494/8296260082 Downloaded from:- Page 1 S S C E Monthly CurrentAffair(August) Contact : 08981426494/08296260082 Visit us: - www

SSCE 8981426494/8296260082 Downloaded from :- www.onlinessce.com Page 125

is only the second ever gold medal for India in the rowing

competition in Asian Games history. India has now won a total of

21 medals at Asiad 2018.

9) Which company planned to launch an all-electric car

sharing service in Berlin?

Ans:- Volkswagen

Explanation:- Volkswagen is to launch an all-electric car sharing

service in Berlin. It is to serve a business serving people in big cities who don't own cars. The company planned to expand its "We

Share project to big cities" in Europe and North America by 2020. Volkswagen would deploy 2,000 of such cars in the first months of

next year. It is to build visibility and ensure that there are enough cars available that customers could always find one near them. It

selected Berlin because of its dense population. Nowadays Volkswagen and other global automakers are investing in new

business models where people use cars as something to be used

when needed rather than owned.

10) This institute launches Fabless Chip Design Incubator

(FabCI) to incubate a minimum of 50 'Make-in-India' chip

design companies.

Ans:- IIT-Hyderabad

Explanation:- IIT-Hyderabad launches First of its kind in India, Fabless Chip Design Incubator (FabCI). It offers facilities like free

software tools with leading technology partners in Electronic Design Automation (EDA). The main aim to incubate a minimum

of 50 'Make-in-India' chip design companies that can compete

globally and generate Intellectual Property Rights (IPRs).

11) The 9th edition of Mountain Echoes Literature Festival

2018 is hosted by the country of;

Ans:- Bhutan

Explanation:- The 9th edition of Mountain Echoes Literature Festival 2018 has started in Thimphu, Bhutan to serve as a

collaborative platform for knowledge sharing, leading to global

cultural potpourri of ideas. The 4-day Literature Festival began with a tribute to former Indian Prime Minister Atal Bihari

Page 126: S S C E Monthly CurrentAffair(August) · SSCE 8981426494/8296260082 Downloaded from:- Page 1 S S C E Monthly CurrentAffair(August) Contact : 08981426494/08296260082 Visit us: - www

SSCE 8981426494/8296260082 Downloaded from :- www.onlinessce.com Page 126

Vajpayee, who was remembered at the annual event for his

animated style of poetry recitations. It will celebrate 50 years of diplomatic relations between India and Bhutan. The 2018 edition

will focus on subjects such as Spirituality and Philosophy, Performing Arts, Environmental Conservation, Food, as well as

Film and Theatre, among others. The literary festival of Bhutan was established under the patronage of Her Majesty the Royal

Queen Mother Ashi Dorji Wangmo Wangchuck and is known for bringing together literary stalwarts from across the world in the

lap of the Himalayas.

12) Who clinched silver in the men's double trap shooting

event at the 2018 Asian Games?

Ans:- Shardul Vihan

Explanation:- Indian shooter Shardul Vihan (15) from Uttar Pradesh clinched silver in the men's double trap shooting event at

the Jakabaring shooting range in the 18th Asian Games 2018 on August 23. He shot 73 to claim the silver medal, while Korea's

Shin Hyunwoo clinched gold with a score of 74. Qatar's Al Marri Hamad Ali bagged the bronze after shooting 53 at the Jakabaring

shooting range. Vihan, who is coached by former Asian Championship double gold medallist Anwer Sultan, had grabbed

four gold medals at Shotgun Nationals Championship in 2017.

13) Who is appointed as head of India investments by Abu

Dhabi Investment Authority (ADIA)?

Ans:- Kabir Mathur

Explanation:- Abu Dhabi Investment Authority (ADIA), world's second largest sovereign wealth fund hired private equity veteran

Kabir Mathur to head operations in India. He is a former director

with US fund KKR & Co. L.P.

14) Name of the Indian-American scientist who has found

simple way to predict Inflammatory Bowel Disease (IBD)?

Ans:- Chander Mohan

Explanation:- Prof. Chander Mohan, an Indian-American

scientist, has identified as many as 50 protein biomarkers that can non-invasively detect Inflammatory Bowel Disease (IBD) - a

Page 127: S S C E Monthly CurrentAffair(August) · SSCE 8981426494/8296260082 Downloaded from:- Page 1 S S C E Monthly CurrentAffair(August) Contact : 08981426494/08296260082 Visit us: - www

SSCE 8981426494/8296260082 Downloaded from :- www.onlinessce.com Page 127

gut disorder that leads to diarrhoea, abdominal cramps, and

weight loss. Mohan, who is professor at University of Houston in the US, received a $ 347,490 grant from the Crohn's & Colitis

Foundation of America. Along with IBD expert Subra Kugathasan, a gastroenterologist at Emory University in the US, Mohan is

examining stool protein biomarkers that indicate the disease. IBD occurs when the body's immune system fights its intestinal cells.

Two of the most common types are Crohn's disease and ulcerative

colitis, both of which cause inflammation in the digestive tract.

15) Olympic Council of Asia (OCA) has given recognition to

__________.

Ans:- Kho-Kho

Explanation:- Olympic Council of Asia (OCA) has given

recognition to Kho-Kho, India's indigenous sport. Kho-Kho will be included as a demonstration sport in the Asian Indoor Games.

Also, it has the possibility to be included as a discipline in the next Asian Games. Rajeev Mehta is the Secretary General of

Indian Olympic Association (IOA). He is also the president of Asian

Kho-Kho Federation.

16) Scientists have developed a wireless 'in-body GPS' which

helps to track tumours inside the body. It is named as;

Ans:- ReMix

Explanation:- Scientists have developed a wireless 'in-body GPS'

system named "ReMix" that can accurately locate ingestible implants and track tumours inside the body. ReMix was able to

track implants with centimetre-level accuracy in animal tests. Similar implants can be used to deliver drugs to specific regions of

the body. The test was conducted by scientists from Massachusetts Institute of Technology (MIT) and Massachusetts

General Hospital (MGH) in the US. ReMix is not accurate enough to be implemented in clinical settings as of now. For actual

implementation, the margin of error should be closer to nearly two

millimetres.

Daily Current Affairs25,August,2018

Page 128: S S C E Monthly CurrentAffair(August) · SSCE 8981426494/8296260082 Downloaded from:- Page 1 S S C E Monthly CurrentAffair(August) Contact : 08981426494/08296260082 Visit us: - www

SSCE 8981426494/8296260082 Downloaded from :- www.onlinessce.com Page 128

1)Industry chamber Assocham appointed whom as a Secretary

General?

Ans:- Uday Kumar Varma

Explanation:- Industry chamber Assocham appointed former

bureaucrat Uday Kumar Varma as its new Secretary General. He succeeds DS Rawat who held the position for nearly 14 years.

Varma retired from the position of Secretary, Ministry of Information and Broadcasting, Government of India in 2013, and

has extensive administrative experience at the highest levels of

administration both in the state government (of Madhya Pradesh)

and central government.

2) India's largest Business Incubator Bhamashah Techno Hub

launched in __________.

Ans:- Jaipur

Explanation:- In a big step forward to strengthen the startup ecosystem in Rajasthan, the Chief Minister Vasundhara Raje

launched India's largest business incubator the Bhamashah Techno Hub in Jaipur. The incubator will provide end-to-end

support to startups, ranging from infrastructure, technology, angel funding, access to venture capitalists, mentorship, exposure to

national and international organisations and experts and much more.

3) India bagged __________ medal in Women's kabaddi at the

Asian Games 2018.

Ans:- Silver

Explanation:- India bagged silver medal in Women's kabaddi at the

Asian Games 2018 after losing to Iran in the final match. The Indian women's kabaddi team finished second for the first time at

Asiad after having won gold in the 2010 and 2014 editions.

Page 129: S S C E Monthly CurrentAffair(August) · SSCE 8981426494/8296260082 Downloaded from:- Page 1 S S C E Monthly CurrentAffair(August) Contact : 08981426494/08296260082 Visit us: - www

SSCE 8981426494/8296260082 Downloaded from :- www.onlinessce.com Page 129

4) Which state government has increased the free electricity supply to the families of the Scheduled Castes and Scheduled

Tribes for domestic purpose, recently?

Ans:- Telangana

Explanation:- Telangana State Government increased the free electricity supply to the families of the Scheduled Castes and

Scheduled Tribes for domestic purpose from the current 50 units to

101 units. This was announced by Telengana Chief Minister Chandrasekhar Rao at a review meeting in Hyderabad. This

decision has been made keeping in mind, the increasing usage of electrical appliances. The state government will pay these charges

to the distributary agencies. Telangana State Government has also increased the honorarium paid to Imams and Moazins of Masjids to

5000 rupees from the current 1500 rupees per month.

5) Which of the following pair won Gold medal in Tennis mixed

doubles at the Asian Games 2018?

Ans:- Bopanna-Sharan

Explanation:- Indian Tennis players Rohan Bopanna and Divij Sharan defeated Aleksander Bublik and Denis Yevseyev of

Kazakhstan 6-3, 6-4 in 52 minutes. India have claimed the men's doubles gold on four occasions at the Games, the last pair being

Somdev Devvarman and Sanam Singh in the 2010 Guangzhou edition.

6) How many thematic reports on Indian Himalayan Region

released by NITI Aayog ?

Ans:- 5

Explanation:- NITI Aayog Releases 5 Thematic Reports On Indian Himalayan Region. It was released by NITI Aayog Vice Chairman Dr

Page 130: S S C E Monthly CurrentAffair(August) · SSCE 8981426494/8296260082 Downloaded from:- Page 1 S S C E Monthly CurrentAffair(August) Contact : 08981426494/08296260082 Visit us: - www

SSCE 8981426494/8296260082 Downloaded from :- www.onlinessce.com Page 130

Rajiv Kumar, CEO Amitabh Kant and member Dr V K Saraswat in

New Delhi. The themes include Inventory and Revival of Springs in the Himalayas for Water Security, Sustainable Tourism, and

Transformative Approach to Shifting Cultivation, Strengthening Skill and Entrepreneurship Landscape in the Himalayas.

Himalayan Tourism is growing annually at 6.8% has created a huge challenge related to solid waste, water, traffic, loss of bio-

cultural diversity.

7) Which company won the Gold Flame Award of the Best

Energy Services Company, in the Sri Lanka National Efficiency Awards 2018 in Colombo?

Ans:- Forbes Marshall

Explanation:- The Sri Lanka National Efficiency Awards 2018

commenced in Colombo. The Sri Lanka National Efficiency Awards 2018 was instituted by Sri Lanka Sustainable Energy Authority.

Forbes Marshall won the Gold Flame Award in the Energy Efficiency Improvement category of the Best Energy Services

Companyfor its contribution to reduce energy use through energy management efforts. This award ceremony was organised annually

by the Sri Lankan Ministry of Power and Renewable Energy and Sri Lanka Sustainable Energy Authority.

8) With which country India signed the Second Protocol

amending the Comprehensive Economic Cooperation Agreement in New Delhi?

Ans:- Singapore

Explanation:- India and Singapore have signed the Second

Protocol amending the Comprehensive Economic Cooperation Agreement in New Delhi which will boost bilateral trade between

the two countries. The Protocol was signed in New Delhi by Rajneesh, Joint Secretary, Ministry of Commerce and Industry and

Francis Chong, Senior Director, Ministry of Trade and Industry, Government of Singapore.

Page 131: S S C E Monthly CurrentAffair(August) · SSCE 8981426494/8296260082 Downloaded from:- Page 1 S S C E Monthly CurrentAffair(August) Contact : 08981426494/08296260082 Visit us: - www

SSCE 8981426494/8296260082 Downloaded from :- www.onlinessce.com Page 131

9) The National Green Tribunal (NGT) has appointed this committee to decide Vedanta's plea challenging closure of its

Sterlite plant. The committee is named as;

Ans:- S J Vazifdar committee

Explanation:- The National Green Tribunal (NGT) has appointed

former Punjab and Haryana High Court Chief Justice S J Vazifdar as head of a 3-member committee to decide mining company

Vedanta Limited's plea challenging the Tamil Nadu government's order to close the Sterlite copper plant at Thoothukudi. The

committee may, if necessary, visit the site and consider the technical data and take a decision as early as found viable

preferably within 6 weeks after it assumes it's working. Recently, on 9th August 2018, the Tribunal had allowed Vedanta to access

the administrative unit inside its Sterlite copper plant at Thoothukudi in Tamil Nadu. Though, the plant would remain

closed and the company would not have access to its production unit and directed the district magistrate to ensure this. In March

2013, Sterlite's factory came into limelight when a gas leak led to the death of one person and injuries to several others, after which

then chief minister J Jayalalithaa had ordered its closure.

10) Which Indian aircraft will be fitted with a marine hydraulic system to boost the air operations of the ship?

Ans:- INS Vikramaditya

Explanation:- For the first time, India's only aircraft carrier INS

Vikramaditya will be fitted with a marine hydraulic system to boost the air operations of the ship. Hydraulics technology uses fluid

pressure to power machines. Technodinamika, a subsidiary of Russia's Rostec State Corporation, will install the system in the

Page 132: S S C E Monthly CurrentAffair(August) · SSCE 8981426494/8296260082 Downloaded from:- Page 1 S S C E Monthly CurrentAffair(August) Contact : 08981426494/08296260082 Visit us: - www

SSCE 8981426494/8296260082 Downloaded from :- www.onlinessce.com Page 132

Indian Navy ship by May 2019. The GS-1MF hydraulic system is

used in helicopters, while GS-3 is used in aircraft.

11) Shardul Vihan became the youngest Indian shooter to win a medal at the Asian Games. He belongs to the state of;

Ans:- Uttar Pradesh

Explanation:- Uttar Pradesh teenager Shardul Vihan (Fifteen-year-

old) became the youngest Indian shooter to win a medal at the

Asian Games. He shot 73 to claim the silver medal, while Korea's 34-year-old Shin Hyunwoo clinched the gold medal with a score of

74. The Union Sports Minister Rahyavardhan Singh Rathore, who won a Olympic silver in the same event at Athens 2004, tweeted his

congratulations to the young marksman

12) . Name of the Indian female tennis player has won bronze in women's singles at the 18th Asian Games 2018?

Ans;- Ankita Raina

Explanation;- Indian Tennis player, Ankita Raina has won a

bronze after losing a hard fought women's singles semifinal 4-6, 6-7 in straight sets against Shuai Zhang of China at the 2018

Jakarta Asian Games on August 23. With this, she became the 2nd Indian female tennis player after Sania Mirza to win a medal in

singles' competition at the Asian Games. For India, Mirza won silver in Doha in 2006 and a bronze in Guangzhou 2010. Ankita

from Maharashtra is currently India's number one women's singles.

13) Which state aims to be the manufacturing hub in eastern

India?

Ans:- Odisha

Explanation:- Chief Minister Naveen Patnaik reported as Odisha aims to leverage its ports. This move is a part of a push to develop

Page 133: S S C E Monthly CurrentAffair(August) · SSCE 8981426494/8296260082 Downloaded from:- Page 1 S S C E Monthly CurrentAffair(August) Contact : 08981426494/08296260082 Visit us: - www

SSCE 8981426494/8296260082 Downloaded from :- www.onlinessce.com Page 133

the State as a manufacturing hub in eastern India that could serve

the larger South Asian regional market. He inaugurated 2 manufacturing plants, a caustic soda facility and Jyoti Solar

Solutions solar power plant. He also laid the foundation stones for 13 units entailing a total investment of Rs. 2,901.20 crore. OCL

India, the flagship company of the Dalmia Group, will be investing Rs.2,000 crore, Hindustan Urban Infrastructure Ltd. Rs.389 crore,

IFFCO Rs. 63.55 crore and Jaipur Cement with Rs. 63.5 crore.

14) International water colour festival was held in __________.

Ans:- Patna

Explanation:- Five-day long 'International water colour festival' is

held in Ranchi at Audrey House, to promote the rich art and culture of Jharkhand. This is the first time in the country that

international artists will paint together. In this program 24 countries of the world, 25 states of India and 24 districts of the

state are participating. On this occasion, the artist Victoria from Ukraine and artist Uttam Karmakar from Italy did the live painting

and presented it to the Chief Minister Raghubar Das.

15) Who launched the National Electoral Transformation (NETA)

mobile application to rate Members of Parliament (MPs) and Members of Legislative Assembly (MLAs)?

Ans:- Pranab Mukherjee

Explanation:- Former President Pranab Mukherjee launched the

National Electoral Transformation (NETA) mobile application to rate Members of Parliament (MPs) and Members of Legislative Assembly

(MLAs). The Neta App aims to build political accountability and transparency among leaders. It will enable users to rate their MLAs

and MPs. It is billed as an instrument to measure voter sentiment in Indian constituencies. The Neta app has been developed by

Pratham Mittal, a 27-year-old entrepreneur. It uses Artificial Intelligence, advanced algorithms, one time password and Aadhaar

Page 134: S S C E Monthly CurrentAffair(August) · SSCE 8981426494/8296260082 Downloaded from:- Page 1 S S C E Monthly CurrentAffair(August) Contact : 08981426494/08296260082 Visit us: - www

SSCE 8981426494/8296260082 Downloaded from :- www.onlinessce.com Page 134

numbers to make sure that the users are genuine.

Daily Current Affairs26,August,2018

1)What is the India’s GDP forecast for 2018, as per Moody’s

Investors Service report “Global Macro Outlook for 2018-19”?

Ans:- 7.5%

Explanation:- Moody’s Investors Service has recently released its

Global Macro Outlook for 2018-19. In it, the Indian economy is expected to grow by about 7.5% in 2018 and 2019 as it is largely

resilient to external pressures like those from higher oil prices. However, the rating agency expects the India’s economy to grow at

a higher rate of 7.7% in the first quarter of 2018. Apart from this, Moody’s put G-20 growth at 3.3% in 2018 and 3.1% in 2019. The

advanced economies will grow 2.3% in 2018 and 2% in 2019.

2) Which state government has decided to organize youths’

conclave ‘ATAL’ in September 2018?

Ans:- Arunachal Pradesh

Explanation:- The Arunachal Pradesh government has decided to organize a youths’ conclave “ATAL – Arunachal Transformation &

Aspirational Leadership” in Itanagar on 22nd September 2018. The aim of the 3-day ATAL conclave is to engage youths in the age

group of 18 to 30 years of the state pro-actively in the areas of

public policy and governance.

3) Which country is host to the multi-nation counter-terror

exercise “SCO Peace Mission 2018”?

Ans:- Russia

Explanation:- The multi-nation counter-terror exercise Shanghai Cooperation Organization (SCO) Peace Mission 2018 has started at

Chebarkul, Russia on August 24 to provide an opportunity to the armed forces of SCO nations to train in counter terrorism

operations in urban scenario in a multinational and joint environment. The scope of the exercise includes professional

interaction, mutual understanding of drills & procedures,

establishment of joint command and control structures and

elimination of terrorist threat in urban counter terrorist scenario.

Page 135: S S C E Monthly CurrentAffair(August) · SSCE 8981426494/8296260082 Downloaded from:- Page 1 S S C E Monthly CurrentAffair(August) Contact : 08981426494/08296260082 Visit us: - www

SSCE 8981426494/8296260082 Downloaded from :- www.onlinessce.com Page 135

4) Which of the following cities is the venue of the

International Water Colour Festival 2018?

Ans:- Ranchi

Explanation:- On August 23, the International Water Colour

Festival 2018 has started at Audrey House in Ranchi, which is organized by the Jharkhand Art Culture and Tourism Department

in Audrey House. The aim of the 5-day fest is to bring local artists in touch with the international art and promote the rich art and

culture of Jharkhand. During the water colour festival, the visiting

artists will interact with school students and local artists on various themes including “nature” and “modern/abstract”. Approx

56 internationally renowned artists — from 24 countries and 25

Indian states are participating in the festival.

5) The NETA app that seeks to let Indian voters rate their elected representatives, is inspired by which country’s

approval system?

Ans:- United States

Explanation:- On August 24, the National Electoral

Transformation (NETA) mobile application has been launched by former President Pranab Mukherjee wherein voters can review and

rate their elected representatives (MPs& MLAs) as well as hold them accountable. Apart from this, the application is being seen as

an instrument to track voter sentiment across various

constituencies in India.

6) Who has been appointed new chairman of the Defence

Research and Development Organisation (DRDO)?

Ans:- G Satheesh Reddy

Explanation:- Renowned aerospace scientist Dr G Satheesh Reddy,

who is scientific adviser to the Defence minister, has been appointed new chairman of the Defence Research and Development

Organisation (DRDO) for a period of two years. He will also be secretary in the Department of Defence Research and Development

(DDR&D) for the same period. The post of DRDO chief was lying vacant since May 2018 after the then Chief Dr. S. Christopher

stepped down. Dr. Reddy is the first defence scientist working in

Page 136: S S C E Monthly CurrentAffair(August) · SSCE 8981426494/8296260082 Downloaded from:- Page 1 S S C E Monthly CurrentAffair(August) Contact : 08981426494/08296260082 Visit us: - www

SSCE 8981426494/8296260082 Downloaded from :- www.onlinessce.com Page 136

India to receive the Silver Medal of Royal Aeronautical Society, UK,

and has also been selected for the first IEI (India) and IEEE (USA)

joint award for Engineering Excellence.

7) The world’s first public blockchain bond “bond-i” has

recently launched by which International organisation?

Ans:- World Bank

Explanation:- The world’s first blockchain bond “bond-i” has

recently launched by World Bank (WB), which is to be created, allocated, transferred and managed through its life cycle using

distributed ledger technology. The “bond-I” stands for Blockchain Operated New Debt Instrument and is also has reference to

Sydney’s most famous beach ‘Bondi Beach’. The Commonwealth Bank of Australia is the sole arranger of the issuance for the bond.

It was the first time that the World Bank bond raised capital from public investors through a legally valid bond issuance that uses

blockchain from start to finish. This 2-year maturity bond has successfully raised 110 million Australian dollars (USD 80.48

million). The World Bank, whose bonds hold an AAA rating, regularly uses its borrowing power to help develop new bond

markets as well as pioneering new means for selling and trading the securities. The bond-i blockchain platform was built and

developed by the CBA Blockchain Centre of Excellence, housed in

the Sydney Innovation Lab.

8) Tejinder Pal Singh Toor, who clinched Gold in men’s Shot-

Put event at the 2018 Jakarta Asian Games, has set new Asiad

record of how much meters?

Ans:- 20.75m

Explanation:- Indian shot putter Tejinder Pal Singh Toor from Punjab has clinched Gold in men’s Shot-Put event with a record-

shattering throw at the 2018 Jakarta Asian Games on August 25. He threw the iron ball to 20.75m to win the gold and set a national

as well as Asian Games record. The previous mark of 20.57m was set by Sultan Abdulmajeed Al-Hebshi of Saudi Arabia at the 2010

Guangzhou Asian Games in China. With this, Tajinder became the first Indian to win the men’s shot put gold at the Asian Games

since Bahadur Singh Sagoo, who had thrown a best of 19.03m to

Page 137: S S C E Monthly CurrentAffair(August) · SSCE 8981426494/8296260082 Downloaded from:- Page 1 S S C E Monthly CurrentAffair(August) Contact : 08981426494/08296260082 Visit us: - www

SSCE 8981426494/8296260082 Downloaded from :- www.onlinessce.com Page 137

win the gold in Busan in 2002. This is the 8th gold medal for India

in men’s shot put in the history of the Asian Games.

9) The 2018 Nallur Kantha Swamy festival has started in which

neighbouring country of India?

Ans:- Sri Lanka

Explanation:- On August 23, the annual festival at Nallur Kandaswamy Kovil has started in Jaffna, Sri Lanka with the sacred

flag hoisting ceremony and is attracting thousands of Hindu pilgrims from within the country and abroad. During the 25-day

festival, thousands of Tamil Hindu devotees will attend the rituals in worship of Lord Murugan, Valli and Devanai Amman. The

colourful festival includes vibrant chariot processions, rhythmic drumming, dancing and acts of self-mortification. The Ther

Thiruvila (chariot festival) is the most popular of all events is very

colourful.

10) Which state government is host to the 2018 Krishi Kumbh

International Conference and Exhibition?

Ans:- Uttar Pradesh

Explanation:- The 2018 Krishi Kumbh International Conference

and Exhibition will be held at Indian Institute of Sugarcane Research (Telibagh) in Lucknow from October 26, which will be

organized by the Government of Uttar Pradesh. The aim of the 3-day conference is to provide a platform to farmers, technical

experts and entrepreneurs for exchange of knowledge and expertise in the field of agriculture sector. It also aims at increasing the

agriculture production by adopting advance technologies. In it, farmers from all 75 districts National & International agriculture

scientist, representatives from industries & policy maker from

Central and State govt. will participate.

Daily Current Affairs27,August,2018

1) Who is the author of the book "Atal Ji Ne Kaha" which has

been published by Darpan Publication?

Ans:- Brijendra Rehi

Explanation:- "Atal Ji Ne Kaha" is the latest of the books on the BJP patriarch and former Prime Minister Atal Bihari Vajpayee

Page 138: S S C E Monthly CurrentAffair(August) · SSCE 8981426494/8296260082 Downloaded from:- Page 1 S S C E Monthly CurrentAffair(August) Contact : 08981426494/08296260082 Visit us: - www

SSCE 8981426494/8296260082 Downloaded from :- www.onlinessce.com Page 138

launched with a message by Prime Minister Narendra Modi. The

book is Authored and compiled by Brijendra Rehi. Mr Rehi is noted Doordarshan producer, and senior journalist. The 320-page book is

a rich repository of the former Prime Minister's select speeches and photographs beside having three interviews by the author. The

book has been published by Darpan Publication.

2) Who will launch the second phase of FAME India scheme?

Ans:- Narendra Modi

Explanation;- Indian Prime Minister Narendra Modi will soon

launch the second phase of the FAME India scheme offering incentives for mass adoption of electric vehicles with an outlay of

Rs 5,500 crore. The Prime Minister will unveil the scheme at the

inaugural session of 'MOVE' - the global mobility summit.

3) This bank suggested its customers to switch to EVM cards.

Ans:- SBI

Explanation:- State Bank of India (SBI), the country's largest

lender, has asked its customers to replace their ATM cards for Europay, MasterCard, Visa (EMV) chip based ATM-cum-debit cards

with magnetic stripe before December 31. The Reserve Bank of India (RBI) had asked banks to issue only chip-based and Personal

Identification Number (PIN) enabled debit and credit cards to protect customers from frauds. EMV chip card protects against

counterfeit skimming (card fraud). EMV chip card and PIN protects

against both counterfeit skimming and lost and stolen card fraud.

4) Who become the first Indian badminton player to enter the

finals of Asian Games?

Ans:- PV Sindhu

Explanation:- India's PV Sindhu defeated Japan's Akane

Yamaguchi 21-17, 15-21, 21-10 in their semifinal clash at the 18th Asian Games to become the first-ever badminton player to reach

the final of Asian Games in either singles or doubles.

5) Emmerson Mnangagwa sworn in as the president of which

country?

Page 139: S S C E Monthly CurrentAffair(August) · SSCE 8981426494/8296260082 Downloaded from:- Page 1 S S C E Monthly CurrentAffair(August) Contact : 08981426494/08296260082 Visit us: - www

SSCE 8981426494/8296260082 Downloaded from :- www.onlinessce.com Page 139

Ans:- Zimbabwe

Explanation:- Emmerson Mnangagwa has taken the oath as

Zimbabwe's president in front of a stadium crowd after a divisive election, as US observers of the vote questioned the country's

democratic credentials.

6) The fourth Asian electoral stakeholders forum (AESF-IV)

begins in __________.

Ans:- Colombo

Explanation:- In Sri Lanka, the fourth Asian electoral stakeholders

forum (AESF-IV) begins in Colombo to discuss the state of elections

and democracy in the region. The two-day forum is being held for the first time in South Asia with support from the Election

Commission of Sri Lanka and the Asian network for free elections (ANFREL). The AESF will be attended by more than 250 delegates

from 45 countries. The theme is, 'Advancing Election Transparency and Integrity: Promoting and Defending Democracy Together'. India

is being represented by Chief Electoral Officer of Maharashtra

Ashwini Kumar and an NGO.

7) Who will be honoured with the Game Changer of the Year

award?

Ans:- Indra Nooyi

Explanation;- PepsiCo's India-born CEO Indra Nooyi will be honoured with the Game Changer of the Year award by a global

cultural organisation Asia Society in recognition of her business achievements, humanitarian record and advocacy for women and

girls around the world. PepsiCo's India-born CEO Indra Nooyi will be honoured with the Game Changer of the Year award by a global

cultural organisation Asia Society in recognition of her business achievements, humanitarian record and advocacy for women and

girls around the world.

8) Who won the Silver medal in 400-meter race in Asian Games

2018?

Ans:- Anjana Thamke

Explanation:- 18-year-old Hima Das won the Silver medal in 400-meter race in ongoing Asian Games 2018, Indonesia. Hima clocked

Page 140: S S C E Monthly CurrentAffair(August) · SSCE 8981426494/8296260082 Downloaded from:- Page 1 S S C E Monthly CurrentAffair(August) Contact : 08981426494/08296260082 Visit us: - www

SSCE 8981426494/8296260082 Downloaded from :- www.onlinessce.com Page 140

50.79 seconds to complete her race. Dutee Chand also bagged

silver medal in women's 100 meter race. Dutee clocked 11.32 seconds to complete her race. Muhammed Anas won Silver medal

in Men's 400m race. Anas clocked 45.69 seconds to complete his

race.

9) Which team wins the FIFA U-20 Women's World Cup for the

first time?

Ans:- Japan

Explanation:- The U-20 Japan Women's National Team won

against the U-20 Spain Women's National Team at the final of the FIFA U-20 Women's World Cup France 2018, taking down the FIFA

U-20 Women's World Cup title for the first time in Japanese football history. Japan received the Fair-play award as a team and

the FIFA U-20 Women's World Cup France 2018 has closed its

curtains.

10) Which bank has been ranked as best State-Run Bank for

Digital Transactions?

Ans:- Punjab National Bank

Explanation:- According to the PNB statement, the report card on

the banking sector's performance in digital transactions, PNB's average percentage of technical declines is only 0.83 per cent of the

total transactions. It has been rated as the sixth best in the overall

category amongst all banks in India for digital performance.

11) India's 1st Biofuel-Powered Flight lands in __________.

Ans;- Delhi

Explanation:- In a Historic day for the country's aviation and

energy sector, India's first biofuel-powered flight landed in Delhi, covering the distance between Dehradun & Delhi. The biofuel is

developed by Indian Institute of Petroleum. The flight test was successfully completed by SpiceJet. The flight was powered with a

blend of 75% air turbine fuel (ATF) and 25% biojet fuel. The advantage of using biojet fuel as compared to ATF is that it reduces

carbon emissions and enhances fuel efficiency. Made from

Jatropha crop, the fuel has been developed by the CSIR-Indian

Institute of Petroleum (IIP), Dehradun.

Page 141: S S C E Monthly CurrentAffair(August) · SSCE 8981426494/8296260082 Downloaded from:- Page 1 S S C E Monthly CurrentAffair(August) Contact : 08981426494/08296260082 Visit us: - www

SSCE 8981426494/8296260082 Downloaded from :- www.onlinessce.com Page 141

12) This compaby wins $56-million International Arbitration

Award, recently.

Ans:- Reliance Power

Explanation:- Reliance Power has announced that its wholly-

owned subsidiary Reliance Power Netherlands BV has won an international arbitration award of $56 million (Rs 390 crore)

against Prestige Capital Holdings (a Seychelles-based company) and Kokos Jiang. Kokos Jiang is the owner of Sugico Group in

Indonesia, from whom Reliance Power had acquired the entire

economic interest in three coal mining concessions in Indonesia in

2010.

13) According to the report of CERT, __________ per cent of

attacks on Indian sites are from China.

Ans:- 35%

Explanation:- According to the report, China has made the highest number of attacks on the official websites of India. While analysing

the data for April and June, CERT has found that 35% of intruding activities on Indian cyberspace are from China. The infamous

attack impacted the top five cities of India, including Kolkata,

Delhi, Bhubaneswar, Pune, and Mumbai.

14) Mizzima Media Group of Myanmar signed an MoU to realize

cooperation and collaboration in broadcasting and content

sharing with which of the Indian company?

Ans:- Prasar Bharati

Explanation:- Prasar Bharati and Mizzima Media Group of Myanmar signed an MoU to realize cooperation and collaboration

in broadcasting and content sharing. The agreement was signed in the presence of Prasar Bharati CEO, Shashi Shekhar Vempati and

Editor-in-chief and MD, Mizzima in New Delhi. The Agreement will realize cooperation and collaboration in broadcasting and envisions

content-sharing covering a wide range of genres, including culture, entertainment, education, science, news and sports along with

other areas of mutual interest.

15) The third edition of Indian Ocean Conference was held in

__________.

Page 142: S S C E Monthly CurrentAffair(August) · SSCE 8981426494/8296260082 Downloaded from:- Page 1 S S C E Monthly CurrentAffair(August) Contact : 08981426494/08296260082 Visit us: - www

SSCE 8981426494/8296260082 Downloaded from :- www.onlinessce.com Page 142

Ans:- Vietnam

Explanation:- The third edition of Indian Ocean Conference is set

to begin on August 27 at Hanoi, the capital of Vietnam. The theme of the two- day conference will focus on 'Building Regional

Architectures', particularly with regards to trade and commerce,

security and governance.

Daily Current Affairs28,August,2018

1) The Bombay Natural History Society (BNHS) will start

operating its regional centre on the campus of Wetland

Research and Training Centre in which state?

Ans:- Odisha

Explanation:- The Bombay Natural History Society (BNHS) will

start operating its regional centre on the campus of Wetland Research and Training Centre near Chilika Lake in Odisha. BNHS

is India's premier avian research institutes. The centre will carry out research on the avian disease by collecting samples and

monitor the Nalabana Bird Sanctuary.

2) Who launched the sports policy for Central Public Sector

Enterprises (CPSEs), recently?

Ans:- Chaudhary Birender Singh

Explanation:- Union Minister of Steel, Chaudhary Birender Singh,

unveiled the corporate sports policy for Central Public Sector Enterprises (CPSEs) under the Ministry of Steel. The policy will

provide a framework for promotion of sports by Steel Ministry CPSEs. Steel CPSEs will allocate a specific budget for sports

activities, as per their financial strength, and this budget will be

separate from the company's CSR funds.

3) Which Indian actor has joined Maharashtra Government's

Mission Shakti' initiative to help sportspersons with medal-

winning potential for the upcoming 2024 Paris Olympiad?

Ans:- Aamir Khan

Explanation:- Aamir Khan will be part of Maharashtra Government's Mission Shakti' initiative to groom sportspersons

Page 143: S S C E Monthly CurrentAffair(August) · SSCE 8981426494/8296260082 Downloaded from:- Page 1 S S C E Monthly CurrentAffair(August) Contact : 08981426494/08296260082 Visit us: - www

SSCE 8981426494/8296260082 Downloaded from :- www.onlinessce.com Page 143

with medal-winning potential for the upcoming 2024 Paris

Olympiad. 'Mission Shakti' will target sports like weight-lifting, archery, shooting, swimming, athletics, volleyball and gymnastics,

with excellence in physical endurance, fitness, superior stamina

and other features included in a rigorous selection test.

4) Which of the following digiLocker system did CBSE launch

to the students of flood-hit Kerala?

Ans:- Parinam Manjusha

Explanatuion:- The Central Board of Secondary Education (CBSE)

along with National e-Governance Division has developed 'Parinam Manjusha', digital academic repository, to provide digital mark

sheets, migration certificates and pass certificates to students from CBSE-affiliated schools in Kerala, whose board exam academic

documents have been lost or damaged in the recent floods. This academic repository has been integrated with DigiLocker. Over

1,300 schools are affiliated to the CBSE in Kerala.

5) This country becomes the world's first country to provide free sanitary products to all students in a bid to tackle "period

poverty".

Ans:- Scotland

Explanation:- Scotland has become the world's first country to

provide free sanitary products to all students in a bid to tackle "period poverty". Scottish government approved a £5.2 million ($6.7

million) initiative that will make sanitary products free at all schools, colleges and universities. Nearly 4 lakh pupils and

students are expected to benefit from the Scottish government's scheme. One UNESCO report estimates that one in 10 girls in sub-

Saharan Africa miss 20 percent of the school year because of their

period.

6) The G20 Digital Economy Ministerial Meeting was held in

which country?

Ans:- Argentina

Explanation:- The G20 Digital Economy Ministerial Meeting was

held in Salta, Argentina. It took place as part of Sherpa Track for 2018 G20 Leaders' Summit, which is to be hosted by Argentina by

Page 144: S S C E Monthly CurrentAffair(August) · SSCE 8981426494/8296260082 Downloaded from:- Page 1 S S C E Monthly CurrentAffair(August) Contact : 08981426494/08296260082 Visit us: - www

SSCE 8981426494/8296260082 Downloaded from :- www.onlinessce.com Page 144

end of 2018. The theme of the meeting was 'Building consensus for

fair and sustainable development'. Its three key issues for the agenda were future of work, infrastructure for development, and

sustainable food future. It concluded with the adaptation of declaration that reflects G20's commitment towards promoting

policies and actions that catalyze digital transformations.

7) Government approves commercial flying of drones in India

from;

Ans:- December 1

Explanation:- The government announced that industries including real estate, power and agriculture will be able to legally

fly remotely piloted aircraft or drones in the country for commercial purposes from December 1, when the 'Digital Sky' platform will

become operative. The Digital Sky platform will be a first national unmanned traffic management platform that implements 'no

permission, no takeoff' for remotely piloted aircraft. Initially, drones will be allowed only along visual line-of-sight during day-time with

a maximum altitude of 400 feet. The usage of drones for delivering

goods and food items has been restricted.

8) Union Water Resources Minister Nitin Gadkari signed an

MoU with how many states for the construction of the Lakhwar

multi-purpose project in the upper Yamuna basin?

Ans:- 6

Explanation:- Union Water Resources Minister Nitin Gadkari signed an MoU with six states for the construction of the Lakhwar

multi-purpose project in the upper Yamuna basin to deal with water crisis. Chief Ministers of Uttar Pradesh (Yogi Adityanath),

Rajasthan (Vasundhara Raje Scindia), Uttarakhand (Trivendra Singh Rawat), Himachal Pradesh (Jai Ram Thakur), Haryana

(Manohar Lal Khattar) and Delhi (Arvind Kejriwal) signed the MoU. Under the agreement, the 204-metres high project will be

constructed near Uttarakhand's Lohari village with live storage capacity of 330.66 million cubic metres (MCM). The total project is

worth Rs 3,966.51 crore.

9) Which country's government banned VIP protocol at airport,

recently?

Page 145: S S C E Monthly CurrentAffair(August) · SSCE 8981426494/8296260082 Downloaded from:- Page 1 S S C E Monthly CurrentAffair(August) Contact : 08981426494/08296260082 Visit us: - www

SSCE 8981426494/8296260082 Downloaded from :- www.onlinessce.com Page 145

Ans:- Pakistan

Explanation:- Pakistan government has banned the VIP protocol

given by Federal Investigation Agency (FIA) to influential people at airports. The VIP protocol was given to politicians, legislators,

senior bureaucrats, judges, military officials and journalists. The decision came into effect on 26th August. The Interior Ministry said

that the ban will be strictly implemented to provide equal opportunities to all travellers. It also warned that in case any FIA

official is caught giving protocol to a VIP, he would face strict

action.

10) Name of the former Indian cricketer who was appointed the

head coach of Zimbabwe national cricket team is;

Ans:- Lalchand Rajput

Explanation:- Former India opener Lalchand Rajput was appointed

the head coach of Zimbabwe national cricket team by the Zimbabwe Cricket (ZC). He is likely to remain in charge for the

2020 World T20 and beyond. He was earlier appointed as the interim head coach of Zimbabwe in May. He has been the manager

of the MS Dhoni-led Indian team during the T20 World Cup in

South Africa in 2007.

11) P V Sindhu won __________ medal in badminton at Asian

Games.

Ans:- Silver

Explanation:- World number three PV Sindhu lost to world number one Tai Tzu-ying (Taiwan) in the women's singles final to

bag India's first-ever silver medal in badminton at Asian Games. Sindhu, playing in her fifth final of 2018, lost the match with a

scoreline of 13-21, 16-21. Saina Nehwal had won bronze medal in

the event.

12) Integrated Defence Staff signed an MoU with which organization aims to set up telemedicine nodes for soldiers in

high-altitude areas?

Ans:- ISRO

Page 146: S S C E Monthly CurrentAffair(August) · SSCE 8981426494/8296260082 Downloaded from:- Page 1 S S C E Monthly CurrentAffair(August) Contact : 08981426494/08296260082 Visit us: - www

SSCE 8981426494/8296260082 Downloaded from :- www.onlinessce.com Page 146

Explanation:- He Integrated Defence Staff of the Defence Ministry

and the Indian Space Research Organisation (ISRO) signed a Memorandum of Understanding (MoU) to set up satellite-enabled

telemedicine nodes to improve emergency medical support to Indian soldiers posted in high-altitude areas, especially Siachen.

ISRO will establish 53 more nodes in the first phase over and above the existing 20, in various establishments of the Army, Navy and

Air Force across the country. These nodes will enable medical consultation between soldiers deployed at the Siachen glacier and

medical echelons in the rear. This joint initiative by ISRO and the

Armed Forces Medical Services will transform the reach of telemedicine to soldiers, airmen and sailors in remote and isolated

posts. One can say that the communication through these nodes will be a paradigm shift in the delivery of lifesaving health care in

adverse terrain and extreme weather conditions till the weather

clears up and movement is possible.

13) Neeraj Chopra wins at __________ medal in Javelin with a

distance of 88.06m sat the Asian Games.

Ans:- Gold

Explanation:- Neeraj Chopra became the first-ever Indian javelin thrower to win a gold medal at the Asian Games. The 20-year-old

junior world record holder hurled the javelin to a distance of

88.06m to bag the Asiad gold.

14) The 2019 Asian Youth and Junior Weightlifting

Championships will be hosted by;

Ans:- North Korea

Explanation:- North Korea announced that it would be the host for

the 2019 Asian Youth and Junior Weightlifting Championships that will commence from October 27, 2019. The Asian Weightlifting

Federation and North Korea's Sports Minister Kim Il Guk signed an agreement for the championship to be held in capital Pyongyang.

Earlier in September 2013, the Asian Cup and Interclub Junior

and Senior Weightlifting Championship was held in North Korea.

15) Who introduced the GST helpline for customers?

Ans:- NAA

Page 147: S S C E Monthly CurrentAffair(August) · SSCE 8981426494/8296260082 Downloaded from:- Page 1 S S C E Monthly CurrentAffair(August) Contact : 08981426494/08296260082 Visit us: - www

SSCE 8981426494/8296260082 Downloaded from :- www.onlinessce.com Page 147

Explanation:- The National Anti-profiteering Authority (NAA) has

introduced a helpline to encourage consumers to resolve all the queries to file complaints against the companies that are not

passing GST rate cut benefits. Since the rollout of GST on July 1, 2017, the GST Council has cut tax rates on several occasions on a

number of items. The helpline number is 011-21400643. It will guide the consumers to register their complaints, provide

information and resolve queries related to profiteering under the Goods and Services Tax (GST) law. NAA said that the consumers

can call the helpline number between 9.30 am to 1 pm and 1.30

pm to 6 pm on all working days.

16) Who became the first Indian to win an individual medal in

the equestrian event at the 2018 Asian Games?

Ans:- Fouaad Mirza

Explanation:- Fouaad Mirza became the first Indian to win an

individual medal in the equestrian event at the Asian Games in Jakarta on 26th August 2018. He won the silver medal in the

Eventing competition, while Japan won the gold medal. The last

individual medal in Eventing competition, however, came in 1982.

17) Tamil Nadu has launched a campaign to highlight the

impact of plastic pollution. Who has been appointed the brand

ambassador of the project?

Ans:- Vivek

Explanation:- Tamil Nadu Chief Minister E.Palanisamy launched an app and a website on plastic pollution free Tamilnadu. He

inaugurated a campaign to highlight the impact of plastic pollution.

He appointed actor Vivek as the brand ambassador of the project.

Daily Current Affairs29,August,2018

1) Who among the following launched 'Navlekha' for Indian

language publishers?

Ans:- Google

Explanation:- At the fourth 'Google for India' event, Google has

unveiled Project Navlekha to help Indian language publishers take

Page 148: S S C E Monthly CurrentAffair(August) · SSCE 8981426494/8296260082 Downloaded from:- Page 1 S S C E Monthly CurrentAffair(August) Contact : 08981426494/08296260082 Visit us: - www

SSCE 8981426494/8296260082 Downloaded from :- www.onlinessce.com Page 148

their content online. Navlekha is a word derived from Sanskrit

meaning 'a new way to write'. The new platform will allow 1.35 lakh Indian publishers to digitise their offline content by creating

instant web pages using scanned documents or PDFs.

2)Which bank named as the world's best bank for its digital

innovation according to Global Finance magazine?

Ans:- DBS Bank

Explanation:- Global Finance magazine named DBS Bank as the world's best bank for digital innovation while still maintaining

stable operations for its customers. It is the first Asian bank to receive the accolade. It was given to ING Bank in 2017. It also

received the Asia's best bank title.

3) Which airlines operated India's first biofuel-powered flight from Dehradun to Delhi on its Bombardier Q400 turboprop

aircraft?

Ans:- SpiceJet

Explanation:- SpiceJet would fly India's first biofuel-powered flight

from Dehradun to Delhi on its Bombardier Q400 turboprop aircraft from Dehradun's Jolly Grant airport. It has used a blend of the

aviation turbine fuel and biofuel to power the flight. The flight had 20 people, including officials from aviation regulator DGCA and

SpiceJet and was around 25 minutes. It would encourage sustainable and alternative fuels for the transportation & aviation

sector under National Biofuels Policy. With this test, India will be among the first developing nations to adopt the sustainable

alternative jet fuel.

4) Name the Indian economist who has been appointed

Assistant Secretary-General and Head of the New York Office of

the United Nations Environment Programme (UNEP)?

Ans:- Satya S Tripathi

Explanation:- Satya S Tripathi, the veteran Indian development economist and UN official, has been appointed Assistant Secretary-

General and Head of the New York Office of the United Nations

Environment Programme (UNEP). The appointment was made by UN Secretary-General Antonio Guterres. Mr. Tripathi, who will

Page 149: S S C E Monthly CurrentAffair(August) · SSCE 8981426494/8296260082 Downloaded from:- Page 1 S S C E Monthly CurrentAffair(August) Contact : 08981426494/08296260082 Visit us: - www

SSCE 8981426494/8296260082 Downloaded from :- www.onlinessce.com Page 149

succeed Elliott Harris of Trinidad and Tobago, has worked with the

UN since 1998 in Europe, Asia and Africa on sustainable development, human rights, democratic governance and legal

affairs. UN Environment is the leading agency at the world body focussed on environment. It works with governments, the private

sector, the civil society and with other UN entities and international

organisations across the world.

5) Whose birth anniversary is commemorated as National

Sports day.

Ans:- Dhyan Chand

Explanation:- The National Sports Day in India is celebrated on 29

August each year. This day marks the birthday of Dhyan Chand Singh, the hockey player who won gold medals in Olympics for

India in the years 1928, 1934 and 1936. He scored over 400 goals

in his career, from 1926 to 1948.

6) Which among the following rebranded as Google pay to offer

Instant Bank Loans?

Ans:- Tez

Explanation:- Google recently partnered with HDFC Bank Ltd.,

ICICI Bank Ltd., Kotak Mahindra Bank Ltd. and Federal Bank Ltd. to offer instant, pre-approved loans to customers. About Tez

Release date: September 19, 2017 Tez got about 8.5 million installations. Over 30 million transactions were made on the app

as of October 27, 2017.

7) NASA's OSIRIS-Rex an asteroid sampling spacecraft, has

caught its first image of which asteroid?

Ans:- Bennu

Explanation:- NASA's OSIRIS-Rex (Origins, Spectral Interpretation,

Resource Identification, Security-Regolith Explorer), an asteroid sampling spacecraft, caught its first image of asteroid Bennu, last

week after a 2-year journey. OSIRIS-Rex has started its final approach towards its target asteroid Bennu. The mission's asteroid

operations started on 17thAugust. OSIRIS-Rex's PolyCam camera

secured the image from a distance of 1.4 million miles (2.2 million

km)

Page 150: S S C E Monthly CurrentAffair(August) · SSCE 8981426494/8296260082 Downloaded from:- Page 1 S S C E Monthly CurrentAffair(August) Contact : 08981426494/08296260082 Visit us: - www

SSCE 8981426494/8296260082 Downloaded from :- www.onlinessce.com Page 150

8) Who of the following has been appointed as the next

Ambassador of India to Ireland?

Ans:- Sandeep Kumar

Explanation:- Sandeep Kumar was appointed as the next

Ambassador of India to Ireland. Sandeep Kumar is an Indian Foreign Service (IFS) Officer of the 1985 batch. Currently, he is the

Ambassador of India to Croatia. He has been appointed as the next Ambassador of India to Ireland. He will take up this assignment

soon.

9) India's first interstate connected wind power project

commissioned in __________.

Ans:- Bhuj

Explanation:- 126 MW wind power capacity, part of country's first Inter State Transmission System (ISTS) auction, is commissioned

in Bhuj, Gujarat. The energy generated from this project is being purchased by Bihar, Odisha, Jharkhand and Uttar Pradesh. This

was a 1,000 MW bid for projects to be connected on ISTS (inter-state transmission system) wherein power generated from one state

(renewable resource rich state) could be transmitted to other

renewable deficient states.

10) Abhaya Prasad Hota was appointed as an independent

director of __________.

Ans:- Indiabulls Ventures

Explanation:- Indiabulls Ventures appointed Abhaya Prasad Hota,

as an independent director on the board for a period of 3 years. Abhaya Prasad Hota is a former managing director at the National

Payments Corporation of India (NPCI). He was involved in designing the operating payments eco-system. He had worked with the

Reserve Bank of India (RBI) for 27 years. He had played a major role in launching payment systems like MICR (Magnetic Ink

Character Recognition), RTGS (Real Time Gross Settlement), NEFT (National Electronic Funds Transfer), IMPS (Immediate Payment

Service) and Aadhaar based system with UIDAI (Unique

Identification Authority of India).

11) What is the size of the nano medical robot that set a new

Page 151: S S C E Monthly CurrentAffair(August) · SSCE 8981426494/8296260082 Downloaded from:- Page 1 S S C E Monthly CurrentAffair(August) Contact : 08981426494/08296260082 Visit us: - www

SSCE 8981426494/8296260082 Downloaded from :- www.onlinessce.com Page 151

Guinness World Record?

Ans:- 120 nanometres

Explanation:- The scientists have developed the smallest medical robot and set a new Guinness World Record. The device, which is

of 120 nanometres in size, can interact with biological cells. These nanocomposite particles can assist to treat cancer and Alzheimer's

in the future and one day it might lead to huge medical advancements. The nanocomposites are made of two different types

of multifunctional oxide materials in a "core and shell"

configuration. The core is magnetic and changes shape in response to magnetic fields. The ferroelectric shell converts pressure into

electric potentials. The magneto-elasto-electric coupled effect in the nanocomposites acts as arms and legs that move the nanoparticle

around to interact with targeted biological cells.

12) Who won silver in Kurash at Asian games?

Ans:- Pincky Balhara

Explanation:- India's Pincky Balhara and Malaprabha Jadhav won silver and bronze respectively in 52 Kg category Kurash at the

Asian Games 2018. Kurash is a form of wrestling indigenous to Central Asia. It combines the skills of judo and wrestling. The 19-

year-old Pincky lost 0-10 to Gulnor Sulaymanova of Uzbekistan in

the gold medal clash to get the silver.

13) Goldman Sachs appointed whom as the managing director

and chief India economist?

Ans:- Prachi Mishra

Explanation:- Goldman Sachs appointed Prachi Mishra as the

managing director and chief India economist. Prachi Mishra will be based in Mumbai. Earlier, she was the deputy division chief of the

western hemisphere department at the International Monetary Fund (IMF). She was the head of the strategic research unit at the

Reserve Bank of India (RBI) from February 2016 to May 2017. She was also a specialist adviser at the RBI between November 2014

and February 2016. She was the first lateral recruit at the RBI. She also worked as a senior economist at IMF and the Finance Ministry

of India.

Page 152: S S C E Monthly CurrentAffair(August) · SSCE 8981426494/8296260082 Downloaded from:- Page 1 S S C E Monthly CurrentAffair(August) Contact : 08981426494/08296260082 Visit us: - www

SSCE 8981426494/8296260082 Downloaded from :- www.onlinessce.com Page 152

14) The Odisha government launched a programme to identify

young talents. It's named as;

Ans:- Mu Hero, Mu Odisha

Explanation:- The Odisha government has launched a programme

"Mu Hero, Mu Odisha" (I am Hero-I am Odisha)" to identify and to give recognition to the young achievers of the state. The initiative is

to be conducted by the Biju Yuva Vahini (BYV) members.

15) The European Union join hands with this country to

develop new Influenza vaccine, recently.

Ans:- India

Explanation:- EU-India joined hands to develop new Influenza vaccine. They have committed EUR 15 million each to fund for this

cause through the "Horizon 2020" programme. Indian government and the European Union (EU) partnered for Rs. 240 crore research

programme to develop a Next Generation Influenza Vaccine to protect people throughout the world. About EU The European

Union (EU) is a political and economic union of 28 member states that are located primarily in Europe. It has an area of 4,475,757

km2 (1,728,099 sq millions) and an estimated population of over

510 million.

16) India's tallest Film Studio was inagurated in which state?

Ans:- Tamil Nadu

Explanation:- Tamil Nadu Chief Minister Edappadi K. Palaniswami has inaugurated India's tallest Film Studio named 'MGR Centenary

Film Studio'. The 56 feet tall, 110 metres long and 100 feet wide studio was constructed using funds collected by the Federation of

Film Employees Union of South India (FEFSI), at Paiyanur. The

studio is named after former Chief Minister M.G. Ramachandran.

Daily Current Affairs30,August,2018

1) Indian women's team won __________ medal in the compound

archery team event at the 2018 Asian Games.

Ans:- Silver

Page 153: S S C E Monthly CurrentAffair(August) · SSCE 8981426494/8296260082 Downloaded from:- Page 1 S S C E Monthly CurrentAffair(August) Contact : 08981426494/08296260082 Visit us: - www

SSCE 8981426494/8296260082 Downloaded from :- www.onlinessce.com Page 153

Explanation:- Indian women's team won silver medal in the

compound archery team event at the 2018 Asian Games. The Indian team comprised Muskan Kirar, Madhumita Kumari and

Jyothi Surekha Vennam. The Indian team was defeated by South

Korea in the finals at the 18th Asian Games.

2) Who plans to outsource the production of the Polar Satellite

Launch Vehicles (PSLVs) and Small Satellite Launch Vehicles

(SSLVs) to private Industries?

Ans:- ISRO

Explanation:- The Indian Space Research Organisation (ISRO) will outsource the production of the Polar Satellite Launch Vehicles

(PSLVs) and Small Satellite Launch Vehicles (SSLVs) to private Industries. The Indian Space agency Chairman K Sivan informed

that private industries will play a major role in capacity building. The ISRO will launch Gaganyaan, India's first manned mission to

outer space in 2020. As per the announcement, the Indian Air Force will identify six persons who can be trained to be astronauts.

The assistance of the country's first astronaut Rakesh Sharma and that of foreign space agencies will be sought for the training of

astronauts.

3) Name of the Indian athelete who brokes the national record in the men's 400 metres hurdles to claim the silver medal at

the Asian Games in Jakarta.

Ans:- Dharun Ayyasamy

Explanation:- Dharun Ayyasamy broke the national record in the

men's 400 metres hurdles to claim the silver medal at the Asian Games in Jakarta. It is India's first medal in the hurdles event

since Joseph Abraham won a gold in Asian Games 2010. Qatar's

Abderrahman Samba won the gold medal at the Asian Games 2018

4) Which committee has given its approval for the umbrella scheme "Ocean Services, Technology, Observations, Resources

Modelling and Science (O-SMART)" of Ministry of Earth

Sciences?

Ans:- CCEA

Explanation:- The Cabinet Committee on Economic Affairs (CCEA)

Page 154: S S C E Monthly CurrentAffair(August) · SSCE 8981426494/8296260082 Downloaded from:- Page 1 S S C E Monthly CurrentAffair(August) Contact : 08981426494/08296260082 Visit us: - www

SSCE 8981426494/8296260082 Downloaded from :- www.onlinessce.com Page 154

has given its approval for the umbrella scheme "Ocean Services,

Technology, Observations, Resources Modelling and Science (O-SMART)" of Ministry of Earth Sciences. The scheme with an overall

cost of Rs. 1623 crore encompasses a total of 16 sub-projects addressing ocean development activities such as Services,

Technology, Resources, Observations and Science. The services rendered under the O-SMART will provide economic benefits to a

number of user communities in the coastal and ocean sectors, namely, fisheries, offshore industry, coastal states, Defence,

Shipping, Ports etc. This scheme also provides necessary scientific

and technological background required for implementation of

various aspects of Blue Economy.

5) Who clinched the silver in women's 3000m steeplechase

event in the 2018 Asian Games?

Ans:- Sudha Singh

Explanation:- Sudha Singh clinched the silver in women's 3000m steeplechase event in the Asian Games in Jakarta. Bahrain won

the gold medal, while Vietnam won the bronze medal. Sudha won a gold medal when women's 3000m steeplechase was introduced in

the Asian Games in 2010.

6) Which country using robots to teach the Kindergarten

children?

Ans:- China

Explanation:- China is using robots to teach the Kindergarten children. The 60cm high, autonomous Keeko robot is an

unconventional teacher. It covers things like storytelling and problem-solving. More than 600 kindergartens across the country

use Keeko robots. The robot makers are hoping to expand it into Greater China and Southeast Asia. Beijing has invested money and

manpower in developing artificial intelligence as part of its "Made in

China 2025" plan.

7) Which among the following became the first bank subsidiary

to enter the commodity broking business?

Ans:- Axis Securities

Explanation:- Axis Securities launched commodity broking

Page 155: S S C E Monthly CurrentAffair(August) · SSCE 8981426494/8296260082 Downloaded from:- Page 1 S S C E Monthly CurrentAffair(August) Contact : 08981426494/08296260082 Visit us: - www

SSCE 8981426494/8296260082 Downloaded from :- www.onlinessce.com Page 155

services with MCX and became the first bank subsidiary to enter

the commodity broking business. Axis Securities launched commodity broking services with MCX and became the first bank

subsidiary to enter the commodity broking business. These cover:

industrial metals, energy and agricultural segments.

8) Who launched the logo for flagship health insurance scheme,

'Ayushman Bharat' in New Delhi?

Ans:- JP Nadda

Explanation:- Union Health Minister JP Nadda launched the logo

for flagship health insurance scheme, 'Ayushman Bharat' in New Delhi. Under 'Ayushman Bharat' more than 10 crore poor &

vulnerable families will get the benefit with an insurance cover of Rs. 5 lakh per family per year. The health insurance scheme will be

launched on Septemeber 25.

9) With which bank, the Indian government signed a $375 million loan agreement to contribute to double farming

incomes in Madhya Pradesh by expanding irrigation networks

and system efficiency?

Ans:- ADB

Explanation:- The government of India and the Asian Development Bank (ADB) signed a $375 million loan agreement to contribute to

double farming incomes in Madhya Pradesh by expanding irrigation networks and system efficiency. The Madhya Pradesh

Irrigation Efficiency Improvement Project will develop 125,000 hectares of new, highly efficient and climate resilient irrigation

networks and improve water use efficiency in more than 400

villages, benefiting over 800,000 people in Madhya Pradesh.

10) The Ministry of Tourism has sanctioned two new projects under Heritage and North East Circuits of Swadesh Darshan

Scheme (SDS) in Which two states?

Ans:- Punjab and Tripura

Explanation:- The Ministry of Tourism has sanctioned two new

projects under Heritage and North East Circuits of Swadesh

Darshan Scheme (SDS) in Punjab and Tripura. 'Swadesh Darshan' is the scheme of the Ministry of Tourism for the development of

Page 156: S S C E Monthly CurrentAffair(August) · SSCE 8981426494/8296260082 Downloaded from:- Page 1 S S C E Monthly CurrentAffair(August) Contact : 08981426494/08296260082 Visit us: - www

SSCE 8981426494/8296260082 Downloaded from :- www.onlinessce.com Page 156

thematic circuits in the country. The first project under Swadesh

Darshan Scheme 'North East Circuit: Imphal and Khongjom' was

inaugurated in Manipur.

11) Researchers from this country developed a new technique

to detect early-stage cancer tumour cells in the blood through

malaria protein VAR2CSA.

Ans:- Australia

Explanation:- Australian researchers have announced that they have developed a new technique to detect early-stage cancer

tumour cells in the blood through malaria protein VAR2CSA. In this method malaria protein VAR2CSA sticks to cancer cells and

helps detect more number of cancer cells than before. This will help in detecting cancer at early stage. The findings were published in

the journal 'Nature Communications'. Researchers from the University of New South Wales (UNSW), Sydney and the University

of Copenhagen were involved in this research.

12) L&T Shipyard launched a Coast Guard offshore patrol

vessel in __________.

Ans:- Chennai

Explanation:- A Coast Guard offshore patrol vessel was launched at L&T Shipyard in Kattupalli in Chennai. It is the third of series of

the seven offshore patrol vessels under construction by L&T Shipbuilding Ltd. The length of the OPVs is 98 metre and breadth

is 14.8 metre, with gross tonnage of 2100 tonne. It will be used for surveillance along with anti-terrorist and anti-smuggling

operations in exclusive economic zone (EEZ) and for coastal

security.

13) Who is the head of the newly formed Prime Minister's

Science Technology and Innovation Council (PM-STIAC)?

Ans:- K Vijay Raghavan

Explanation:- The Government of India (GoI) has constituted a 21-member committee to advise Prime Minister Narendra Modi on

policy matters related to science, technology and innovation. The

committee, dubbed as Prime Minister's Science Technology and Innovation Council (PM-STIAC), will be headed by K Vijay

Page 157: S S C E Monthly CurrentAffair(August) · SSCE 8981426494/8296260082 Downloaded from:- Page 1 S S C E Monthly CurrentAffair(August) Contact : 08981426494/08296260082 Visit us: - www

SSCE 8981426494/8296260082 Downloaded from :- www.onlinessce.com Page 157

Raghavan, the principal scientific adviser to the GoI. It will

facilitate the formulation and implementation of policies and decisions, provide action-oriented and future preparedness advise

and assist in directing S&T to solve the socio economic problems in

the country.

14) Who become the first Indian heptathlete to win a Gold

Medal in the 2018 Asian Games?

Ans:- Swapna Barman

Explanation:- Swapna Barman created history by becoming the

first Indian heptathlete to win an Asian Games gold in Indonesia, a feat she achieved despite a toothache. The 21-year-old Barman

logged 6026 points from the seven events competed for two days. En route to the title, she won the high jump (1003 points) and

javelin throw (872 points) events and finished second-best in shot

put (707 points) and long jump (865 points).

15) For which state, the world bank provide a loan amount of

250 million USD recently?

Ans:- Rajasthan

Explanation:- The Union Government, Rajasthan and the World

Bank signed a USD 250 million Development Policy Loan (DPL) to support Rajasthan in improving the performance of its Electricity

Distribution Sector under the State's '24x7 Power for All' program. The loan from the World Bank's International Bank for

Reconstruction and Development (IBRD), has a 3 year grace period

and a maturity of 21 years.

Daily Current Affairs31,August,2018

1) Indian Railways and this organisation Sign MoU for use of

Natural Gas in Railway Workshops.

Ans:- GAIL

Explanation:- In order to replace Industrial gases like Dissolved

Acetylene, LPG, BMCG and Furnace Oil / High Speed Diesel (HSD) oil with environment friendly Natural Gas, Indian Railways has

signed a Memorandum of Understanding (MoU) with M/s GAIL

Page 158: S S C E Monthly CurrentAffair(August) · SSCE 8981426494/8296260082 Downloaded from:- Page 1 S S C E Monthly CurrentAffair(August) Contact : 08981426494/08296260082 Visit us: - www

SSCE 8981426494/8296260082 Downloaded from :- www.onlinessce.com Page 158

(India) Limited, to provide infrastructure facilities for supply of

Natural Gas to Indian Railways Workshops, Production Units and Depots at Rail Bhavan. This MoU is a broad-based, in principle

agreement, between GAIL and Indian Railways for the creation of infrastructure and supply of CNG/LNG/PNG for both industrial

and domestic purposes.

2) How much did the Cabinet approve to set up more disaster

warning systems in Indian coasts?

Ans:- Rs.1,623 crore

Explanation:- The Cabinet Committee on Economic Affairs cleared

at an outlay of Rs.1,623 crore to get more disaster warning systems and desalination plants along Indian coasts. The system will be

functioning under the programme called O-SMART (Ocean Services, Technology, Observations, Resources Modelling and

Science). O-SMART is monitored by the Union earth sciences ministry. O-SMART provides economic benefits to a number of user

communities in the coastal and ocean sectors, namely, fisheries,

offshore industry, coastal States, defence, shipping, ports, etc.

3) The first U.S.-India 2+2 dialogue will be held in which city?

Ans:- New Delhi

Explanation:- The first U.S.-India 2+2 dialogue will be happening in New Delhi on September 6. The countries will discuss regional

and strategic issues of Asia and will also have some real outcomes. Randall G. Schriver, Assistant Secretary of Defence for Asian and

Pacific Security Affairs of the United States said that China and how to respond to it will be front and centre of the dialogue. U.S.

Secretary of State Mike Pompeo and Secretary of Defense James

Mattis will be travelling to India for the dialogue.

4) Who organizes 2-Day Conference On Motivating & Attracting

Youth in Agriculture?

Ans:- ICAR

Page 159: S S C E Monthly CurrentAffair(August) · SSCE 8981426494/8296260082 Downloaded from:- Page 1 S S C E Monthly CurrentAffair(August) Contact : 08981426494/08296260082 Visit us: - www

SSCE 8981426494/8296260082 Downloaded from :- www.onlinessce.com Page 159

Ans:- In order to meet the challenge of providing sustainable

livelihoods for a rapidly growing population and to motivate and attract youth in agriculture, Dr. Trilochan Mohapatra, Secretary

(DARE) & Director General (ICAR), inaugurated a two-day conference at NASC, New Delhi on Motivating and Attracting Youth

in Agriculture (MAYA). The two-day conference provides an opportunity to all stakeholders to interact and discuss various

options and avenues for not only attracting youth to agriculture but even motivating them towards entrepreneurship in agriculture

and allied fields.

5) Which state signed a MoU with an America-based agency

'Malaria No More'to make the state malaria-free?

Ans:- Odisha

Explanation:- Odisha government signed a MoU with an America-

based agency 'Malaria No More' and Indian agency 'Malaria Elimination Trust' to make the state malaria-free. sThe two

agencies will prepare the roadmap for public awareness and will work with the various organization in the state. Earlier this year

1.13 crore mosquito nets medicated with larvicides were

distributed in the state.

6) Who is appointed as India's next High Commissioner to

United Kingdom?

Ans:- Ruchi Ghanashyam

Explanation:- Senior diplomat Ruchi Ghanashyam has been

appointed as India's next High Commissioner to the United Kingdom. Ghanashyam is a 1982 batch Indian Foreign Service

officer and is currently Secretary (West) in the Ministry of External Affairs. She succeeds Y K Sinha who had assumed the charge in

December 2016.

7) Name the racer who won the Belgian Grand Prix 2018 on

26th August 2018?

Page 160: S S C E Monthly CurrentAffair(August) · SSCE 8981426494/8296260082 Downloaded from:- Page 1 S S C E Monthly CurrentAffair(August) Contact : 08981426494/08296260082 Visit us: - www

SSCE 8981426494/8296260082 Downloaded from :- www.onlinessce.com Page 160

Ans:- Sebastian Vettel

Explanation:- Sebastian Vettel won the Belgian Grand Prix 2018 at Spa-Francorchamps Track in Belgium. Ferrari's Sebastian Vettel

beat Mercedes' Lewis Hamilton who tops the driver standings by a 17-point lead, and secured the first position. Mercedes' Lewis

Hamilton finished in the second position. He was followed by Red

Bull's Max Verstappen in the third position.

8) Which Indian Naval Ship is participating in the multilateral

regional maritime engagement exercise "KAKADU 2018"?

Ans:- INS Sahyadri

Explanation:- The 14th edition of multilateral regional maritime

engagement exercise "KAKADU 2018" has started in Darwin, Australia from August 29 and will continue till September 15. It is

hosted by the Royal Australian Navy (RAN) and supported by the Royal Australian Air Force (RAAF). Indian Naval Ship 'INS

Sahyadri', commanded by Captain Shantanu Jha, is participating in this edition of exercise. Indian Navy's participation is a

significant milestone in towards strengthening mutual confidence among regional navies and is expected to further bolster India's

contribution in ensuring peace and stability in the Indo-Pacific region. The exercise KAKADU is held biennially in Darwin and the

Northern Australian Exercise Areas (NAXA).

9) This bank changes its names and IFSC codes of its branches

recently.

Ans:- SBI

Explanation:- The State Bank of India (SBI), has changed the names and IFSC codes of 1,300 of its branches. The idea behind

the move is to rationalize SBI's merger with six associate banks and the Bhartiya Mahila Bank (BMB) which came into effect from

April 1, 2017. SBI has updated the list of branches with old and

new names and IFSC codes on its website.

10) The Indian table tennis mixed doubles pair, A.Sharath

Page 161: S S C E Monthly CurrentAffair(August) · SSCE 8981426494/8296260082 Downloaded from:- Page 1 S S C E Monthly CurrentAffair(August) Contact : 08981426494/08296260082 Visit us: - www

SSCE 8981426494/8296260082 Downloaded from :- www.onlinessce.com Page 161

Kamal and Manika Batra won __________ medal in the 18th

Asian Games 2018.

Ans:- Bronze

Explanation:- The Indian table tennis mixed doubles pair,

A.Sharath Kamal and Manika Batra won the first-ever historic bronze in the 18th Asian Games 2018. In the semifinals, Kamal

and Batra lost to China's Yingsha Sun and Wang Sun and scored 9-11, 5-11, 13-11, 4-11, 8-11. In their pre-quarterfinal, they won

against the South Korean pairing of Lee Sangsu and Jeon Jihee by

securing 11-7, 7-11, 11-8, 10-12, 11-4.

11) Which among the following become the wettest place in

India?

Ans:- Mahabaleshwar

Explanation:- Maharashtra's Mahabaleshwar has become the

wettest place in India this year, leaving behind Meghalaya's Cherrapunji. India Meteorological Department (IMD) data revealed

that the hill station in the state of Maharashtra received a staggering 5661.6 mm rainfall this season till now, much more

than Cherrapunji's 4735 mm. Cherrapunji holds two Guinness world records for receiving the maximum amount of rainfall in a

single year, 26,471mm (1,042.2 in) of rainfall between August 1860 and July 1861 and the maximum amount of rainfall in a single

month (9,300mm) in July 1861.

12) Name of the Indian athelete who won gold medal in the

men's 1500m event in the Asian Games 2018 in Jakarta,

Indonesia.

Ans:- Jinson Johnson

Explanation:- India's Jinson Johnson claimed the gold medal in the men's 1500m event in the Asian Games 2018 in Jakarta,

Indonesia. Manjit Singh, 800m champion, however, finished fourth. Johnson clocked 3:44.72 sec to claim the top honours ahead of

Iran's Amir Moradi, who turned in a timing of 3:45.62sec, his

Page 162: S S C E Monthly CurrentAffair(August) · SSCE 8981426494/8296260082 Downloaded from:- Page 1 S S C E Monthly CurrentAffair(August) Contact : 08981426494/08296260082 Visit us: - www

SSCE 8981426494/8296260082 Downloaded from :- www.onlinessce.com Page 162

season's best effort. Johnson earlier won a silver in the 800m

event.

13) India won __________ medal in the women's 4X400 metre

event.

Ans:- Gold

Explanation:- India won gold in the women's 4X400 metre event. The Indian quartet of Hima Das, Raju Poovamma, Saritaben

Laxmanbhai Gayakwad and Vismaya Koroth registered a time of 3 minutes and 28.72 seconds to finish at the top of the podium.

India clinched the silver medal in the men's 4X400 metre event.

14) Who represented India at the G20 Digital Economy

Ministerial Meeting 2018?

Ans:- Ravi Shankar Prasad

Explanation:- In Argentina, the G20 Digital Economy Ministerial

Meeting 2018 was held in Salta with theme 'Building consensus for fair and sustainable development' on August 23-24. It took place as

part of Sherpa Track for 2018 G20 Leaders' Summit, which is to be hosted by Argentina by end of 2018. India was represented by

Union Minister for Electronics and IT & Law and Justice, Ravi Shankar Prasad. The G20 Ministerial meeting allowed India to

showcase the inclusive use of digital technology by the Government to empower the people. It also allowed India to articulate it's views

on global issues relating to cyber security, data protection and innovation for growth. The key issues of the meeting were future of

work, infrastructure for development, and sustainable food future. It concluded with the adaptation of declaration that reflects G20's

commitment towards promoting policies and actions that catalyze

digital transformations.

15) The Indian women's hockey team enters Asian Games for

1st time by defeating Which country?

Ans:- China

Page 163: S S C E Monthly CurrentAffair(August) · SSCE 8981426494/8296260082 Downloaded from:- Page 1 S S C E Monthly CurrentAffair(August) Contact : 08981426494/08296260082 Visit us: - www

SSCE 8981426494/8296260082 Downloaded from :- www.onlinessce.com Page 163

Explanation:- The Indian women's hockey team defeated three-

time champions China 1-0 in the semi-final to reach their first final at Asian Games since 1998. Indian defender Gurjit Kaur scored the

only goal of the match. India will face Japan in the final match on

Friday, August 31.

16) The 6th RCEP Trade Minister's Meeting begin in which

country?

Ans:- Singapore

Explanation:- Union Minister of Commerce& Industry and Civil

Aviation, Suresh Prabhu, is leading the Indian delegation for the

6th RCEP Trade Minister's Meeting which has begun in Singapore. The Minister will participate in the meeting consisting of 10 ASEAN

countries and six ASEAN FTA partners namely, India, China,

Japan, Korea, Australia and New Zealand.

Best Regards from:- www.onlinessce.com

Page 164: S S C E Monthly CurrentAffair(August) · SSCE 8981426494/8296260082 Downloaded from:- Page 1 S S C E Monthly CurrentAffair(August) Contact : 08981426494/08296260082 Visit us: - www

SSCE 8981426494/8296260082 Downloaded from :- www.onlinessce.com Page 164

Like our facebookpage:-

www.facebook.com/onlinessce

Visit us: www.onlinessce.com

Write us:-

[email protected]

Page 165: S S C E Monthly CurrentAffair(August) · SSCE 8981426494/8296260082 Downloaded from:- Page 1 S S C E Monthly CurrentAffair(August) Contact : 08981426494/08296260082 Visit us: - www

SSCE 8981426494/8296260082 Downloaded from :- www.onlinessce.com Page 165